Medicin All Nov, Sep, Oct PDF

You might also like

Download as pdf or txt
Download as pdf or txt
You are on page 1of 140

‫ﻣﺼﺪر اﻻﺳﺎﻟﺔ ﺟﻠﻮري ﺗﯿﻢ ﻓﻘﻂ ﺟﺰاھﻢ ﷲ ﺧﯿﺮا ﻛﺜﯿﺮا ‪ ،‬وﺑﻔﻀﻞ‬

‫ﷲ وﺣﺪه ﺳﮭﻞ ﺗﺠﻤﯿﻊ اﻻﺳﺎﻟﺔ اﻟﻤﻜﺮرة وﺗﻠﺨﯿﺼﮫ ﷲ ﯾﻨﻔﻊ ﺑﮫ‬


Esophageal Disease
🌹Pt after endoscopy found tumor in stomach MALT lymphoma, and he has h pylori +ve ,
what to do ?A- Antibiotics B- surgical
GERD
🌹Smoker has chronic GERD , upper scope was done and showed high grade dysplasia
what is the management: refer for possible esophagectomy
🌹Pt heavy smoker with symptoms of GERD, upper endoscopic examination
show’s squamous cell with high-grade dysplasia, next step in management:
A. Council smoking cessation B. Endoscopic mucosal resection
🌹 58Y OLD male, came with sx of PU with weight loss. what to do?
A. Urgent referral for upper endoscopy. B. PPI for 4 wk
🌹Adult patient came to ER with Substernal pain for 1 hour not radiating no other issue,
with previous attacks 2 weeks ago last for seconds only?A. GERD B. MI C. pericarditis
🌹Pt was having GERD sxs and on PPI 40 mg. And his symptoms controlled. He
stopped the medication 4 months ago and symptoms back again but worsen.
All labs and P/E is normal. What to do?
A. Resume same dose✅ B. Give higher dose PPI 80mg C. Give ranitidine and PPI
🌹Patient with GERD being treated. What is the indicator of good therapy?
A. clinical symptoms ✅✅ B. PH monitor
🌹 with persistent smptoms of GERD despite treatment with PPI , mx ? increase dose of
PPI
🌹Dark stool with medication of GERD. What is the likely cause = Bismuth ‫🌹ﺑﺰﻣﻮت‬
، ‫اﺳم دواء ﻟﻌﻼج ﺑﻛﺗﯾرﯾﺎ اﻟﻣﻌدة‬
‫ = ﯾﺟﯾب ﺑﻛﺗﯾرﯾﺎ اﻟﻣﻌدة‬meal = ‫ = وﺟﺑﺔ اﻛل‬malt ‫ رﺑط‬. = dark stool ‫رﺑط = ﺑزﻣوت = ﺑراز‬

🌹Mobridty GERD with obese == Roux- en-Y gastric bypass surgery 🌹


🌹Patient diagnosed with GERD taking omeprazole 20mg but not improving, what to do?
A. Add ranitidine at bedtime B. Same treatment twice daily ✅
🍒🍒GERD the main risk factor esophageal carcinoma = Barret esophagus🍒🍒
🍓 Case of 70 yrs man how are heavy smoker and he came with melena and abdominal
pain and weight loos lab are unstable : A. Cancer of esophagus ✅
🍒Red Flag Age >50 Smoker Wight loss🍒
🍒elderly pt with epigastric discomfort and fullness , weight loss . Decrease appetite What
appropriate investigation: Gastroduodenoscopy
🍒 Elderly with GERD not respond to PPI what is the next step ?
A. manometry B. 24h monitoring PH✅. C. lifestyle modification. D. nissen fundoplication
• Barret's esophagus with low grade dysplasia management?
○ Surgical resection. ○Antacids. ○ Surveillance every 6-12m
○ PPI and rescope every 6-12 m ✅
🌹patient had knee pain, no trauma no fever and also have epigastric pain.
A. -ibuprofen ‫ع ﻻن ﻋﻨﺪە ﻗﺮﺣﺔ ﻣﻌﺪة‬c‫ ﻣﻤﻨ‬B. -paracetamol✅ C. -aspirin.

Patient complaining of epigastric pain, better with vomiting, worse with eating. He's
taking medications for her his joint pain. What's the diagnosis?
• Perforated peptic ulcer (Here is perforated so less likely) • Boerhaavesyndrome •
Gastritis✅
‫ اﻟﻤﻌﺪة‬ŒŽ • ‫ „ﻠﻤﺔ اﻟ† اﻧﻪ ﺷﺨﺺ ﻋﻨﺪە ﻗﺮﺣﺔ‬، ‫ﻂ ﺗ|ﺒ~•ﺎل اﻋﺮاض ﻗﺮﺣﺔ اﻟﻤﻌﺪة‬y‫ﺎﻟﻀ‬u ‫ ﻫﺬە‬gastric ‫ﻟﻤﺎ •ﻘﻮﻟﻚ اﺣﺲ‬
‫ ﻗﻤﺔ اﻟﻤﻌﺪة‬ŒŽ • ‫ ب اﻟﻢ‬epigastric pain ‫ ﻟ~ﻪ ؟؟ ﻻن ﺣﻤﺾ اﻟﻤﻌﺪة •ﻄﻠﻊ ﻣﻊ اﻟﻬﻀﻢ واﻻ™ﻞ‬، ‫و–—–ﺪ ﻣﻊ اﻻ™ﻞ‬
‫ﺎب اﻟﺮﺋ¨ﺴ~ﺔ ل ﻗﺮﺣﺔ اﻟﻤﻌﺪة ﻫﻮ اﺳﺘﻌﻤﺎل اﻟﻤﺴﻜﻨﺎت‬y‫ ﺛﺎﻧ~ﺎ اﺳ‬، ‫ﻌﺪ اﻻ™ﻞ‬u ‫ اﻟﻘﺮﺣﺔ ﻓ~ﺤﺲ ب اﻟﻢ‬¤‫ ﻋ‬¢ Ž £ ~‫ﻓﺎﻟﺤﻤﺾ ﻳ‬
‫ اﻻﻟﻢ ﻣﻦ ﻋﺎﺋﻠﺔ‬NSAID ‫ ﻣﺜﻞ‬، ‫ وﻟ¨ﺲ اﻟﺒﻨﺎدول‬Ibrufen ‫ ﻓﺎﻟﺴ|ﻨﺎر–ﻮا‬، ‫ ﻣﻌﺪة ﻓﺎﺿ~ﺔ‬¤‫–ﻠﺔ وﻋ‬c‫وﺧﺎﺻﺔ ﻣﻊ اﻟﻤﺪة اﻟﻄ‬
‫ ﻟﻮ „ﺎن ﻓ~ﻪ‬، ‫ﻌﺪ اﻻ™ﻞ و„ﺎن ´ﺴﺘﻌﻤﻞ ﻣﺴﻜﻨﺎت‬u ‫ﻂ •ﻘﻮﻟﻚ اﻟﻢ‬y‫ﺎﻟﻀ‬u Gastric ulcer ‫ ﺑ|ﻨﻤﺎ‬، ‫„ﺎن ﺑ~ﻜﻮن ﻫﻮ اﻟﺠﻮاب‬
perforater ‫ اﻟﻤﻌﺪة‬ŒŽ • ‫•ﻘﻮﻟﻚ اﻟﻢ ﺷﺪﻳ|ﻴ~ﺪ ﺟﺪا‬
sever abdominal pain
•Œ ، ‫ ﻫﺬا اول „ﻠﻤﺔ‬perforater ‫ ﺛﺎ•¸ „ﻠﻤﺔ ﻣﻬﻤﺔ ﻫﻮ‬sudden ‫ﻓﺠﺎة ﻛﺬا وﻫﻮ ﺗﻤﺎم ﻣﺎ•ﺤﺲ ب ¼»ء اﻟﻢ ﺟﺪا ﻗﻮي‬
Ž Ž Ž
‫ﺎﻟﻤﻌﺪة‬u : Sudden sever abdominal pain ‫غ دم‬À‫»ء ´ﺴﺘﻔ‬ ¼
Ž ‫ ﺛﺎﻟﺚ‬.
‫ﻟﺬﻟﻚ اﻗﺮب اﺧﺘ~ﺎر ﻫﻮ‬Gastritiis ‫ﻦ ﻻ ﺗﺨﺘﺎروا‬Ä‫ﻟ‬. ‫ﺎﺑﻬﺎ ﻣﺴﻜﻨﺎت‬y‫ وﺻﺢ اﺣﺪ اﺳ‬، ‫ﺎﻻﻋﺮاض ﻫﺬە‬u ¢ Ž £ ~‫ﻧﻌﻤﺰﻣﻤﻜﻦ ﺗ‬
perforated .
.. . . .. . .
pt with history of peptic ulcer and +ve urea breath test. You started and finished the
treatment course. When you can re-examine the urea breath test?
A. at least 4 weeks ✅ B. at least 2 weeks C.at least 1 week D.examine now ( urea = ‫ارﺑﻊ‬
‫ﺣروف = ارﺑﻊ أﺳﺎﺑﯾﻊ‬
🍒🍒urea breath test after finsh treatment course =at least 4 weeks 🍒🍒
......
🌹DM and HTN known case patient, presents with severe epigastric pain radiating to back
for 6 hours, CXR shows left air under diaphragm, most likely diagnosis: Perforated peptic
ulcer 🌹
….
🍓peptic ulcer patient with vomiting examination showed succession splash while shaking
the abdomen what acid base disorder he has:
metabolic acidosis
metabolic alkalosis compensated✅ metabolic acidosis
compensated metabolic alkalosis
n
‫ = ر`ﻂ‬alkalosis = ‫ء‬mo = ‫ﻞ‬k‫ﺲ =ا‬i‫ﺎﻟﻮز‬e‫ اﻟ‬.
Succussion splash also known as a gastric splash, is a sloshing sound heard through the
stethoscope during sudden movement of the patient on abdominal auscultation. It reflects
the presence of gas and fluid in an obstructed organ, as in gastric outlet obstruction

🍒Urine findings in metabolic alkalosis (vomiting with low serum k):


A. aciduria✅ B. alkaline urine. C. hypokalemia
🍒pt severe vomiting ECG show flat T wave what u expect in urine ?
A. aciduria✅. B. high K. C. high na. D. alkaline urine
🍒Patient with chronic use of NSAID for osteoarthritis, Present with sudden severe
abdominal pain , guarding and tenderness, What most appropriate next step: Erect chest X-
ray ( peptic ulcer perf)
🌹Pt with 2 weeks history of watery diarrhea, vitals were stable What is the expected
acid-base abnormality?
A. Metabolic acidosis B. Metabolic alkalosis
C. Compensated metabolic acidosis D. Compensated metabolic alkalosis

🌹Pt with US and have diarrhea what electrolyte find :


A.metabolic acidosis B.metabolic alkalosis C. recompense metabolic acidosis
..
🌹Burning chest pain for 6 months increased at night and unpleasant taste when lifting
heavy objects:
A. Esophagitis✅ B. Acute gastritis C. Boerhaave syndrome ( its do blood vomting) D.
Perforated peptic ulcer
.....
🌹An adult patient k/c of severe peptic ulcer disease presents with recurrent vomiting;
imaging shows a severely narrowed pylorus with ulceration at many.points. What would
you most likely find in this patient?
A. hypokalemia B. Metabolic acidosis C. Two other options
🍒Pt had 2 weeks of excessive vomiting after investigation he has chronic PUD leads to
sever narrowing of pylorus so what you wil find in the labs?
A-Resp alkalosis B-Metabolic acidosis C-Hypokalemia ✅ D-Increase urinary potassium
.....
🌹 Which food consider low potassium diet ? ‫ „ﻤ~ﺔ ﻗﻠﻴ|~ﻠﺔ ﻣﻦ‬¤‫ ﻃﻌﺎم •ﺤﺘﻮي ﻋ‬k
A. Bananas B. Avocado C. Berries‫ ﺗﻮت‬D. Strawberry

🌹MALT treatment: = H.pylori eradication

🌹Confirm eradication of H.pylori ? Urea breath test 🌹
......... . .
🌹 How to follow patient after ttt of H.pylori ? Urea breath test 🌹

Male asymptomatic , they did endoscope and found multiple ulcers , after biopsy they
found +ve pylori and MALT lymphoma
A. H.pylori eradication B. surgical C. Chemo
...
🌹Pt had h.pylori and treated for 10 days with ppi, clarithro and azithro. Now having same
ssx, what to do?
A. give the same regimen for 14 days
B. give doxycycline and metronidazole
C. give tetracycline with metronidazole
then change to bismuth
‫ﺴﺘﺨﺪم‬ž ، ‫ﺎدة‬w‫ ﻳﻮم ز‬١٤ ‫‚ ﻣﺎﻳﻨﺠﺢ ﻣﻌﻬﻢ اﻟﻌﻼج ﻓ}ﺤﺘﺎﺟﻮا ا‘ اﻧﻬﻢ “”ﻤﻠﻮا اﻟ–ﻮرس‬ƒ‫ {ﺎﻟﻤ}ﺔ ﻣﻦ اﻟﻤﺮ‬٢٠ ‫ﺎ‬xwv‫ﺗﻘ‬
‫ ﻓﻴﻬﻢ‬protin pump inhiptor ‘o‫ ا‬²§‫ اﻻﻗﻞ “ﻜﻮن ﻏ‬-‫ واﺣﺪ ﻣﻨﻬﻢ ﻋ‬، ‫¦ ﻣﻀﺎد ﺣﻴﻮي‬ ƒ § ‫{ﺎﻻﺿﺎﻓﺔ ا‘ اﺛﻨ‬
‫ ف‬، ‫ اﺳﺘﺨﺪﻣﻬﻢ ﺳﺎ{ﻘﺎ‬tetracyclin ‫ و‬metro ‫‘ ﺗﻌﺎﻟﺞ‬o‫ﺔ ا‬w‫ ﻣﻦ اﻻدو‬h.pylo ‫ﻢ ﺳﺎ{ﻘﺎ‬¹‫‘ اﺧﺬ‬o‫وﺗﺨﺘﻠﻒ ﻋﻦ ا‬
.....
🌹Patient develop black tarry stool after H.pylori treatment drugs , which drug cause it ?
bismuth
…..
🌹Patient is known to have chronic gastric ulcers at pylorus, presented with vomiting and
abdominal pain. UGI Endoscopy: narrowing at the pylorus end due to chronic ulcers, What
abnormality are likely present?
A. increase urinary potassium
B. metabolic acidosis
C. hypokalemia ✅
Metabolic alkalosis if not hypokalemia .
........
🌹 woman have epigastric pain and shortness of breath for many years . Realeve by vomit
only What to do ?
A. upper gi endoscopy ✅✅( gastric ulcer) B. barium swallow C. CT abdominal D. plain x ray
abdominal
.....
🌹Patient her father has gastric ulcer, c13 urea breath test done for her shows +ve result.
What is the organism? H.pylori

🌹pt after endoscopy found tumor in stomach, and he has h pylori +ve , do ?
A. H pylori eradication therapy. B. refer for possible gastrectomy
‫ ال‬H.pylory ‫ اذا ﻣﺎ ﻋﺎﻟﺠﻨﺎﻫﺎ راح ﺗﻌﻤﻞ‬reccurnt ulcer Ð‫ ﻣﻤﻜﻦ ﻳﺘﺤﻮل ا‬ÐŽ‫ﺎﻟﺘﺎ‬Ï‫ و‬cancer ‫ ﺗﻤﺎﻣﺎ‬Ñ •
Ž ‫ﻓﻤﻬﻢ ﺟﺪا ﻧﻘ‬
‫–ﺎ اﻻول‬ÒÓ‫ﻜ‬y‫ ﻫﺬە اﻟ‬¤‫ ﻋ‬.
.......
Child with Abdominal pain, +ve urea breath test. Given tripple therapy. How to know
improvement? A- pH 24 hours monitoring. B- Clinical. C- Urea breath test .
‫ﻞ و {ﻌﺪ اﻟﻌﻼج‬x‫ﺎ اﻟﻤﻌﺪﯨﺪة ﻧﻌﻤﻠﻪ ﻗ‬w²§‫ﺎر {ﻜﺘ‬x‫اﺧﺘ‬

🌹Patient with h pylori has cancer What will do


A. Observe B. H pylori eradicating C. Gasteroctomy

🌹 Pt with history of PUD and he has joint pain which analgesic to give ?
A. Codeine. B. Paracetamol✅. C. Aspirin ‫ﺎﻟﻤﻤﻌﺪة‬u ‫ع ﻻن ﻋﻨﺪە ﻗﺮﺣﺔ‬c‫ﻣﻤﻨ‬

🍓Pt complete 2week of h pylori treatment today and come to ask you for test
eradication when to do it?After 4 weeks✅
....
🌹Regarding ppi after endoscopy: A. take for 24 hr B. 36 hr C. 48 hr D. 72 hr
How does PPI Help GI bleed?
PPIs reduce gastric acid secretion for up to 36 hours,41 thereby promoting healing of ulcers
and erosions as well as stabilizing thrombi and decreasing rates of GI bleeding in patients .
🌹pt with acut upper GI bleeding due to PUD=IV PPI for 72 hours🌷
...
🌹IV medication before upper endoscopy in upper gi bleeding patient? A. Vit K B.
Octreotide
...
🌹 Stomach varicocele before endoscopy what to give? Octeoride

🌹 pt did graham patch for gastric perforation, then he had esophageal bleeding, and ask
about the next appropriate step in management:
A.endoscopy stent ✅✅ B. Laparotomy C. Ct scan
..........
Pt with melena and labs show anemia , Normal upper and lower endoscopy . next step ?
capsule
....
🍓Pt with upper Gi bleeding, after initial ER management, what to give before endoscopy:
A. propranolol B. octreotide✅ C. VIT K D. vasopressin
........
🌹Highest diagnostic value test for achalasia ?
A. lower esophageal manometry B. UG Endoscopy
‫ ﻫﺬا‬manometry ‫ •ﻘ¨ﺲ‬:
measures muscle contractions
....
patient with achalasia. Repeated attempt of balloon expansion relapses again.What is the
management?- myotomy✅
‫¸ ﻟﻠﻀﻴﻖ ﻫﻮ ﻗﺺ ﻋﻀﻠﺔ اﻟﻤﺮيءو‬Ž • ‫ﺎﻟﻮن ﻓﺎﻟﺤﻞ اﻟﺜﺎ‬y‫ﻟﻼن ﻣﻤﺎ ﻧﻔﻊ ﻣﻌﻪ اﻟ‬

..............
Pt alcoholic presented with hemoptysis and vomiting and abdominal pain:
A. Esophageal varices ✅✅ B. Acute pancreatitis C. Chronic pancreatitis D. Pancreatic
pseudocyst
Alcoholic cirrhosis ) prot HTN ) esophageal varices
........
-Patient presenting with Esophageal perforation post dilatation for achalasia, how will
you manage?
A-Surgical drainage and anastomosis ✅✅ B--Esophagectomty
🌹Esophageal perforation post dilatation for achalasia =Surgical drainage and anastomosis
🌹
.... ....
A 32 years-old alcoholic male patient is brought to the emergency department with the
history of vomiting large amount of bright red blood. Physical examination revealed
splenomegaly and ascites. Which of the following is the most likely source of bleeding? -
Esophageal varices.✅
... ..... ...... . ..
• Esophageal carcinoma management = ○Referral for possible esophageal resection ✅ ○
Antacids
‫ﻪ اﻟﻮرم‬u‫ اﺻﺎ‬ÐŽ‫ﺌﺼﺎل اﻟﺠﺰء ا‬Û‫~ﺪ ﻻزم ﺟﺮاﺣﺔ او اﺳ‬Ù‫ اﺻﺎب اﻟﻤﺮيء ا‬ÐŽ‫ ﻫﻨﺎ ﻋﻼج اﻟﻮرم ا‬.
-- ----- ----- -----
30)_ Forceful vomiting followed by severe central chest pain and nausea, subcutaneous
emphysema dx: esophageal rupture. ✅✅ ‫ اﻟﺼﺪر‬ŒŽ • ‫ء‬à á £ ‫ﺐ اﻟﻢ ﺷﺪ•ﺪ و ﻣﻔﺎ‬Þ‫ﺳ‬
……
Coffee floor emesis: A- GERD. B- H. Pylori C- Gastritis
‫ﺴﻤﻌﻮا „ﻠﻤﺔ‬â ‫ ( ﻟﻤﺎ‬coffe ground emesis ) ‫ او اﺣﻤﺮ ﺟﺪا‬، ‫غ ﻛﺘﻞ دم ﻟﻮﻧﻬﺎ ﻣﻘﺎرب ﻟﻼﺳﻮد‬À‫ﻣﻌﻨﺎﻫﺎ ﺟﺎﻟﺲ ´ﺴﺘﻔ‬
‫ ﻻن ﻣﻌﻨﺎﻫﺎ ﻓ~ﻪ ﻧ—–ﻒ ﻣﻦ اﻟﻤﻌﺪة او‬، ‫ ﻏﺎﻣﻖ وﻫﻨﺎ ﻧﺨﺎف‬esophages .
coffee ground vomitus, means that upper gastrointestinal bleeding is more acute or more
severe, for example due to Mallory–Weiss tear, gastric ulcer or Dieulafoy's lesion, or
esophageal varices,
gastritis.
...........
esophageal carcinoma? A.smoking B. barret esophagus ✅
......
male complaining of reflux and post prandial discomfort this a main complain in his family
What is the risk factor for cancer esophagus? Barrette esophagus ‫ﺻﺢ‬
‫ﻪ‬yé‫ﺴ‬â ‫ﻃﺎن‬è= ‫ة‬Òç‫ﺎر =ﺑ‬u
...... ...... ......

🍓Pt vomit blood I think he has Varicose vein what to give him for decrease risk of
bleeding : B blocker

Long-term treatment to decrease risk of recurrent bleed `-blocker (e.g. nadolol)✅
• Repeat EVL/sclerotherapy • Nitrates • Follow-up
….
🍒 Elderly pt with Dysphagia to solid and liquid PR examination change in stool color .=
Esophageal ca

🍒 Smoker and On biopsy there is metaplasia of the esophagus what’s the management:
A. Refer for possible esophageal resection✅ B. Smoke stopping plan
🌹metaplasia mean pre.cancer = Barrett's esophagus = high grad dysplasia = endoscopic
ablation/resection, or esophagectomy + if low grade dysplasia, both surveillance and
endoscopic ablation/resection are satisfactory options.
.........
🍒To prevent esophageal bleeding in liver cirrhosis: BB🌹

🌹Alcoholic with negative Hx, vomits blood for 1 day, no epigastric tenderness Dx?
A. Esophageal varices B. gastric ulcer C. acute pancreatitis D. chronic pancreatitis
....
🌹Pt with esophageal bleeding, How to prevent? A. Nadolol B. Aspirin
‫ﻠﻮك ﻣﺜﻞ ﻣﺎ اﻧﻪ •ﻘﻠﻞ ﺿﻐﻂ اﻟﺪم ﻛﺬﻟﻚ •ﻘﻠﻞ‬u ‫ﺑ|ﺘﺎ‬
decrease portal pressure in patients with cirrhosis
‫ ﻣﻦ‬ïŽ Ó –‫ و‬rebleeding prevention .
....
Old man + smoking + esophageal show squamous cell with highly displasia next step?
A. Stop smoking B. surgical resection
†¼Û‫ﻞ ﻻ ﻳﻨ‬y‫ ﻃﻮل اﺷ~ﻞ اﻟ†ﻃﺎن ﻗ‬¤‫ ﻋ‬ñ • ç ‫ﻃﺎن ﻣﺎ ﻳﻨﻔﻊ اﻗﻮﻟﻪ وﻗﻒ ﺗﺪﺧ‬è ‫ ﺧﻼص ﺳﺎر ﻋﻨﺪە‬.
....
🌹Pt presented with many bouts of hematemesis after prolonged vomiting, Dx? A. Mallory
weis B. Esophageal varices
....
🌹Pt chronic alcoholic, complains of progressive dysphagia, he’s cachectic on Digital Rectal
Exsam there’s clots of blood with stool. Dx? A. Esophageal ca B. Acidic peptic disease C.
Pancreatitis
......
🌹A 40 year old man underwent endoscopic esophageal dilatation and was cleared to go
home post op day 2. He returns post op day 8 with retrosternal pain and fever (39°).
Esophageal perforation > endoscopic stent if mild
‫ ﻋﻤﻞ‬، ‫ واﺿﺤﺔ‬endoscopic esophageal ‫ = وﻣﻦ ﻣﻀﺎﻋﻔﺎﺗﻬﺎ‬Esophageal perforation
....
🌹Elderly with severe epigastric pain increased at night, with metallic taste
A. esophagitis B. Esophageal cancer C. Boerhave..
‫ﻂ =اﻟﻤﺮيء •ﺪﺧﻞ ﻓ~ﻪ اﻻ™ﻞ ﻃﻌﻤﻪ‬Ï‫ =ر‬metallic
...
🌹Most common/important risk factor for esophageal cancer is: A. Heavy smoking B.
Barrett's esophagus
....
🌹child with GERD symptoms on metronidazole, omeprazole which drug to add?
Clarithromycin ✅🌹
‫–ﺎ اﻟﻤﻌﺪة‬Òç‫ﻜﺘ‬u ‫–ﺾ ﻋﻨﺪە‬À‫ •ﻠﻤﺢ اﻧﻪ اﻟﻤ‬H.pylory ‫ وﻋﻤﻞ ﻟﻪ‬GERD ، ‫–ﻖ اﻟﺪزاء‬À‫–ﺎ اﻟﻤﻌﺪة ﻋﻦ ﻃ‬Òç‫ﻜﺘ‬u ‫ﻓﻼزم ﻧﻌﺎﻟﺞ‬
¸Ž ô ‫ اﻟﺜﻼ‬.
.........
The pt already on metronidazole and PPI what should you add?
A. clarithromycin✅ B. tetracycline C. sucralfate
Triple Rx :amoxicillin + clarithro + ppi
Quadruple Rx: metronidazole + tetracycline + bisthmus + PPI
.......
🌹Pt take 20 mg omeprazole no improvement of symptoms, urea breath
test negative : A. Increase dose of omeprazole ✔ B. Add clarithromycin
usually dose is 20 mg OD for 4 week if the Pt not fully healed after 4 weeks,treatment may
continue for another 4 weeks.
......
🌹What symptom makes you suspect eosinophilic esophagitis the most?
prolonged chewing of food
....
Case about Esophagitis in children = A_Worse night symptom B Excessive chewing✅✅
🌹Esophagitis in children =Excessive chewing 🌹
…..
In patients with reflux symptoms despite standard dose once daily PPI (eg, omeprazole 40
mg once daily), options include splitting the dose (eg, omeprazole 20 mg twice daily),
doubling the PPI dose (eg, omeprazole 40 mg twice daily), or switching to another PPI (eg,
lansoprazole 30 mg once daily)
....
Duodenal perforation treatment?
simple closure with omental patch “Graham omental patch” ✅ ✔
.. . ..

Cardiology
🍓 Patient Post MI with 2:1 AV block IV atropine given but did not work what to
give? pace maker

🌹 Elderly pt with CHF on ACE, spironolactone, furosemide, admitted for some reason I
forgot, upon admission she was on fluids? Labs show she’s
hypotensive with increased urea and creatinine and FENa 0.6 What to do?
A. Increase furosemide. B. Increase spironolactone C. Increase fluids carefully

🌹 pt asymptotic with severely aortic stenosis what you will do?


A. follow up. B. aortic valves replacement
🌹 Pt with atrial fibrillation, what is the most common cause of death
A. MI. B. sudden cardiac death
🌹 Pt on renal disease and decreased GFR need to dialysis in next year What risk of death
in this pt!!!??? A. Renal failure B. Coagulopathy. C. Cardiovascular disease
🌹 Risk factor of stroke= A. HTN B. DM. C. hyperlipidemia. D. obesity
🍒pic of ECG show lead 2 and 3 and Avf elevated :
A. anterior st elevation MI. B. posterior st elevation MI c- Inferior wall MI
🌹Pt have high BP and mild BPH what to give? A. ccb. B. bb. No alpha blocker in answers.
🌷 Scenario, develop AV block m = Temporary pacemaker
🌷 Pr has HF due to cardiomyopathy and has chronic Afib, how to control heart rate:
A. BB. B. Digoxin. C. Deltizimab
🌷 CHF + dialated cardiomyopathy A-Ace+ diueretic. B-Bb+ dieretic
🌹before allowing vigorous exercise what condition need to be excluded
first ? hypertrophic cardiomyopathy
🌹 Adult girl with heart block mobitz type 1, future prognosis?
A. Need pacemaker. B. Cardiomyopathy or MI. C. Reassure
🌹 30 y old male came with dyspnea chest pain PND orthopnea OE pansystolic murmur
Gallop s3 with displaced apex, no any significant medical history Most appropriate next
step: A. Chest X ray and ECG B. Echo
🌹Patient HTN has EF 55% and LVH. Cause of HF in this case ?
A. Systolic dysfunction. B. Diastolic dysfunctions. C. HCM
🌹 Case of STMI and center doesn't have PCI what you will give this pt
A. ASA streptokinase heparin BB B. ASA streptokinase nitroglycerin BB
C. Other choices I exclude BC there was no streptokinase
🌹 Case of 60 years old male medically free came for routine check up every thing is normal
he doesn't complain about anything, when u calculate his risk for 10 years risk of MI it was
6.9 by score i don't remember what its name, what you will do for this patient :
A. CT angio B. MRI cardiac. C. Stress Echo
🌹(Picture of pericaditis) what sign do espect to find in examination = friction rub🌹
🌹Which one of the following is a primordial prevention method
A-Preventing risk factors of MI✅✅ B-Modifying existing risk factors of MI
C-Preventing MI complications

🌹Patient with motor vehicle accident he have aortic injure and spleen laceration while
you preparing patient for transfer his blood pressure drops what to do
A-Transfer to center they have vascular surgeon
‫ﺎت‬S‫ﻪ ﻟﻠﻌﻤﻠ‬:‫ﻒ ﻓﻮرا ﻧﻮد‬CF% % ‫ ﻻزم ﻧﻮﻗﻒ اﻟ‬، ‫ ﻣﺴﺘﻘﺮة ﻧﺰل اﻟﻀﻐﻂ‬FG‫ﺾ ﻏ‬CB‫ﻤﻮت ﻻن ﺣﺎﻟﺔ اﻟﻤ‬: ‫ راح‬، 6& % ‫ وﻗﺖ اﻗﻮم اﻧﻘﻠﻪ ل ﻣﺮﻛﺰ ﺛﺎ‬$& % ‫ ﻣﺎ‬.
B-Transfer to OR✅✅C-To ICU

🌹A patient with HTN that has been persistent despite antihypertensives.


Several investigations= Urine catecholamine very high. Which drug will you use for
management? A.CCB. B.ACE1. C.alpha blocker✅✅ D.b blocker
🌷Urine catecholamine very high +persistent despite antihypertensives Treat by = alpha
blocker🌹

🌹pulmonary hypertention investigation: A-echo✅. B- Ct

‫ وﻟﻣﺎ ﯾﺣﺻل ﻓﺷل‬right ventrical failler ‫ ھو اﻧﮫ ﺣﺻل‬pulmonary hypertenstion ‫ﻣن اﺣد اﻻﺳﺑﺎب اﻟﻲ ﺗﻌﻣل‬
‫ ﻣﺎ ﯾطﻠﻊ اﻻ ﻓﻘط ﻟﻣﺎ ﯾﻛون ﺣﺻل ﻓﻲ‬ventrical ‫ ھذا اﻻﻧزﯾم ﯾﺧرج ﻣن‬، BNP ‫ﻓﻲ اﻟﺑطﯾن اﻻﯾﻣن ﯾرﺗﻔﻊ اﻧزﯾم اﺳﻣﮫ‬
Echo ‫ ﻻزم اﻋﻣل ﻟﮫ اﯾﻛوا‬pulmonary hypertention ‫ ﻓﻠﻣﺎ ﻧﺷﺧص ﻣرﯾض اﻧﮫ ﻋﻧده‬، ‫اﻟﺑطﯾن ﻓﺷل ﻓﻲ وظﺎﺋﻔﮭﺎ‬
right vent failler . ‫ﺣﺗﻰ اﺳﺗﺑﻌد وﺟود‬
The most important serum markers for pulmonary hypertension are brain natriuretic
peptide (BNP) and NT-proBNP; strongly elevated values may suggest right ventricular
failure. The most important non-invasive diagnostic tool is echocardiography.

🌹 Which of the following is the most preventable cause of Dyslipidemia and Coronary
heart disease in women? A)Smoking. B)Diet✅. C)Physical inactivity
If no dyslipidemia answer is smoking With dyslipidemia Diet more important Avoid
saturated fat.
🌹Patient with mitral stenosis getting pregnant, what is the physiological change that
Happen in pregnancy affecting or leading to heart failure I can’t remember:?
A. Increased RBC B.Increased stroke volume✅✅
🍬stroke volume is the volume of blood pumped from the left ventricle per beat.🍬
‫ ﯾﺳﺑب ﺿﻌف ﻓﻲ ﻋﺿﻠﺔ اﻟﻘﻠب‬stroke volum ‫ف ﻣﻊ ﺗﺿﯾق اﻟﺻﻣﺎم و زﯾﺎدة‬
🌹patient with A fib. For 4 h and vitally unstable (HR= 140, with something else) how to
manage: A- amiodaron. B- electrical cardiversion ✅✅. C- digoxi

🌹 pt with 3month hx of left leg non pitting edema no history of surgery or trauma and
there is thickening skin and dark color ask about next investgo?A- venous duplex✅
‫ ھذه ﺣﺎﻟﺔ‬lymphedema ‫ ﻣﺛل ﻣﺎ ﻗﻠت ﺳﺎﺑﻘﺎ اذا ﺷوﻓﺗوا ﻛﻠﻣﺔ‬non pitting ‫ اول ﺷﻲء ﻧﺳﺗﻌﻣل‬، ‫ﻓﻛروا ﻟﯾﻣف اﯾدﯾﻣﺎ‬
venous douplex ‫ ﻻﺳﺗﺑﻌﺎد وﺟود‬DVT ‫ وﻟﺗﺷﺧﯾص‬lymphedema ‫ ف اﻓﺿل ﺷﻲء ھو‬Lymphgraphy

🌹Aoertic valve stenosis what is indications for aortic valve replacement ?


Severe AS = AVA ( aortic valve area) if less than 1 cm ✅ ✔
‫ ﺳﻢ‬١ ‫ ااذا „ﺎن ﻓﺘﺤﺔ اﻟﺼﻤﺎم وﻗﻄﺮﻫﺎ اﻗﻞ ﻣﻦ‬، ‫ ﻟﻤﺎ •ﻜﻮن ﺣﺼﻞ ﻓ~ﻪ ﺿﻴﻖ‬û‫ﺪال ﺻﻤﺎم اﻻور‬yÛ‫ ﻋﻤﻠ~ﺔ اﺳ‬Ð‫ اﻟﺘﺠﺄ ا‬ýÓ‫ﻣ‬
‫ ﺳﻢ‬2 ‫ ﻣﻦ‬ÒôÙ‫ ا‬û •
Ž ‫ اﺻﺢ ﻗﻄﺮ اﻟﺼﻤﺎم اﻻور‬ý‫ﻤﻌ‬u ‫ او‬þŽ~‫ ﻻن اﻟﻔﺘﺤﺔ اﻟﻄﺒ‬، ‫ﺪﻟﻪ‬yÛ‫ﻫﻨﺎ ﻻاازم اﺳ‬
🌹What is the hard sign of vascular surgery: A-Weak pulse. B-Skin color change ( Ischemia ).
C-bruit ✅ ✔
‫ﺑروﯾت =ﻣﺑروك =ﻛﻠﻣﺔ ﻣﺑروك ﻣﺎ ﻧﻘوﻟﮭﺎ ل اﺣد اﻻ اذا ﻣر ﺑﺻﻌوﺑﺎت ﻛﺛﯾرة وﻧﺟﺢ ﻓﯾﮭﺎ‬bruit = ، ‫ = ﺻﻌب‬hard = ‫رﺑط‬
🌹Asymptotic pt with severe AS , EF I think 40 How to manage? valve replacement
🌹Heart failler diagnosed and on ttt, now comes with SOB otherwise every things are
normal, what you want to add? Frusemide.(lasix)
‫ ﺳﺑب ھذه اﻟﻛﺗﻣﺔ ھﻲ ﺗراﻛم اﻟﺳواﺋل ﻓﻲ اﻟرﺋﺔ ﻓﻧﺣﺗﺎج دواء ﯾطﻠﻊ ھذه‬، ‫ وﯾﺷﺗﻛﻲ ﻣن ﺿﯾﻘﺔ ﻧﻔس‬heart failler ‫طﯾب ھﻧﺎ اﻟﻧرﯾض ﻋﻧده‬
.‫اﻟﺳواﺋل اﻟﻣﺗﺟﻣﻌﺔ ف دواء ﻻزﯾﻛس ھو اﻓﺿل ﺷﻲء ﻋﺎﻟﻣﻲ ﻟﺗﺧرﯾﺞ اﻟﺳواﺋل اﻟﻣﺗﺟﻣﻌﺔ ﺳواء ﻓﻲ اﻟرﺋﺔ او اي ﻣﻛﺎن ﺛﺎﻧﻲ‬
🌹Scenario of different BP between upper and lower limb in a child, what is the
diagnosis? Coarctation of Aorta ✅ ✔
‫ ﯾﺧﺗﻠف ﻋن اﻟﺿﻐط اﻟﻲ ﻓﻲ‬upper limo ‫اي ﻣرة ﺗﯾﺟوا ﺗﻘﯾﺳوا اﻟﺿﻐط ﻟﻠﻣرﯾض وﻟﻘﯾﺗوا ﻓﯾﮫ اﺧﺗﻼف ﺑﯾن اﻟﺿﻐط ﻓﻲ‬
Coarctation of aorta ‫ﻻ ﺗﻔﻛرون ب ﺷﻲ ﻏﯾر‬lower limp
‫ ف اذا ﻛﺎن ﻓﯾﮫ اﺧﺗﻼف ﻓﻲ اﻟﺿﻐط‬، femoral ‫ﻻن اﻟطﺑﯾﻌﻲ ﻟﻣﺎ اﻗﯾس اﻟﺿﻐط ﺑﺎﻟﯾد ﻣﺎﯾﺧﺗﻠف ﻛﺛﯾر ﻋن ﻟﻣﺎ اﻗﯾﺳﮫ ﻓﻲ‬
‫ ﻛﻠﻣﺔ ھذه‬coraction ‫ﻣﻌﻧﺎھﺎ ھذا ﻣﺋﺔ ﺑﺎﻟﻣﺋﺔ ﻋﻧده ﻣﺷﻛﻠﺔ ﺑﺎﻟﻘﻠب ﻋﻣﻠت ھذا اﻻﺧﺗﻼف واﻟﻣﺷﻛﻠﺔ ھﻲ‬mm 10 ‫اﻛﺛر ﻣن‬
‫ﯾﻌﻧﻲ ﺣﺻل ﺿﯾق ﻓﻲ اﻻورطﻰ ﻓﺎﻟدم ﻣﺎﯾوﺻل ﺑﻧﻔس اﻟﺳرﻋﺔ ﻟﻠطرف اﻟﻌﻠوي واﻟﺳﻔﻠﻲ ﻻن ﻓﯾﮫ ﺿﯾق‬
) ⚽⚾ )) ‫ ﻻزم ﻋﺷﺎن ﻧﻠﻌب ﻛورة ﻻزم ﯾﻛون ﻓﯾﮫ اﺛﻧﯾن ﻓرﯾق )ﻣﺧﺗﻠﻔﯾن‬، ‫ (( =ﻛورة‬Coarctation = ‫رﺑط‬

🌹HTN, DM female, on medications of CHF, now admitted to be undergo hysterectomy,


she is now in Gyne department, she now developed SOB. What to do?
A- Frusemide ✅ B- Chest and andomen

🌹STEMI pic, what is your action: PCI


🌹Pt female with worsening symptoms of dyspnea, bilateral basal crackles, she have
severe mitral regurg, but symptoms controlled with medication Your management:
A. follow up after 6 months B. refer for mitral valve replacement
🌹Patient with vasovagal syncope after examination everything was normal wat
will u doA. Reassure and send home B. Holter C. Tilt table

🌹CHF s/s with dilated both atria . Asking highly diagnostic value :
A. coronary angio B. echo C. cardiac ct
🌹Decrease mortality in HF: Enalapril
🌹Old pt DM- HTn cardiac dis, long senario. And all treatment given and stable on
thrombolytic, What is the most common cause of death during his hospital stay:
A. bleeding B. stroke C. PE D. MI
🌹Anterolateral MI, management? (Aspirin, BB, analgesia, nitroglycerin)
🌹 Supraventricular tachycardia= Carotid massage
🌹Inferior MI, What is next step? right lead
🌹best diagnostic test to detect ischemia :
A. stress Echo ( Approved)✅ B. CK
🌹Female in her fifties develops MI and she was asking about the best way to prevent a
reinfarction:asprine
🌹 coarctation of the Aorta (absence and delay of femoral pulse) confirmatory test:
A. ECho B. X ray C. CT angio

🌹- 57 ys old with atenolol 100 1st visit 160/100 2nd visit 158/95 then 160/99 ?
A. stop atenolol & start hydrochlorothiazide
B. Dec atenolol & start hydrochlorothiazide
C.Dec atenolol & start CCB D.Stop atenolol & start ACE
🌹 splinter hemorrhages below his nails= Infective endocarditis

🌹Post MI pt what to do: A. Open within 6 weeksB. Lap within 6 weeks C. Same within 6
months ‫ اﺳﺎﺑﻴﻊ‬٦ ‫ ﺷﻬﻮر واﻻﺧﺮى ﻗﺎﻟﺘﻮ‬٦‫{ﻌﺾ اﻟ–ﺘﺐ ﻗﺎﻟﺘﻮ‬
🌹Eledeely female with inferior MI, JVP raised, what is the cause
A.rt Ventricular infarction B.left V infarctionC. tamponade D. complete heart
🌹 primordial prevention? Risk factors prevention like in cardiac diseases.
• ç ‫ ﻣﺜﻞ اﻟﺘﺪﺧ‬، ‫ﺪاﺋ~ﺔ =ﺣﻤﺎ•ﺔ اﻟﻨﺎس ﻣﻦ اي ر–ﺰك ﻳﺆدي ل اﻣﺮاض اﻟﻘﻠﺐ‬y‫ اﻟﻮﻗﺎ•ﺔ اﻟ‬¤‫ﻣﺜﺎل ﻋ‬
‫اﻟﺦ‬.. ‫ اﻟﺴﻤﻨﺔ‬، ñ
🌹Patient K/C of HTN on medication, with prostatic enlargement, no sign of malignancy,
Vitaly stable:A. Alpha blocker B. surgical

🌹 Left ventricular hypertrophy caused by :


A. systolic dysfunction. B. diastolic dysfunction C. septal defect !
....

🌹Role of furosemide in CHF= A. Hypokalemia✅. B. Hypoglycemia C. Bronchospasm


🌹50 y.o DM HTN CHF on DM med and HTN med and BB was admitted to cardiac care unit
as NSTEMI. What will you add to improve his condition?
A. Digoxin B. CCB (forgot the name) C. Aspirin D. Anticoagulant
🌹65y female K/C of DM, heart disease HTN, history of fever, severe vomiting and
diarrhea, examination she looks dehydrated, JVP: 1 cm above sternal angle, central
pressure 3 cm of H2O, Na fraction excretion: 0.6. What is the most appropriate next step?
A.IV dupatamine. B.IV Furosemide C.IV bolus N saline with precaution✅✅
D.IV spironolactone.
🌹Patient with inferior MI treated at hospital then 2 days later come complain of SOB,
bilateral basal lung crepitation: A- Rt side heart failure B- papillary muscle rupture

🌹Man is concerned about cardiovascular risk, the most important risk factor for
CVD= A-waist circumference 103cm. B. blood glucose 8 C. BMI 31

🍓Infective endocarditis Echo view: TEE = Trans esophagiat echoÖ!@


🌹Adult known Asthmatic and Heart disease, presented to you with increasing SOB 3 weeks
ago, + Ejection Systolic Murmur, grade 4/6. that confirms the diagnosisL: transthoracic
echocardiogram (TTE)
🌹 pt medically free and asymptomatic have screen and discover have diastolic murmur
2/6 what to do ? A. Transthoracic echo ✅ B. Transesophageal echo
🌹What is the most appropriate investigation for MS:
A. Transesophageal Echo. B. Transthoracic Echo. C. CT
🌷Bronchogenic carcinoma of the lung = Trans- tracheal endoscopy 🌷

🍒Elderly pt have DM and HTN on anti hypertensive drugs he developed a productive cough
with greenish sputum and SOB ,What is the Cause induce this type of cough ?
A. hypertension B. medications C. heart problems✅
🍒Treatment of MI in none equipped hospital = : mona + thrombolytic
🍓VSD 2mm: = Observe🍓
🍓 Post MI = best for secondary prevention :“ without history “
A. bv physical exercise B. lower lipid C. smoking cessation.✅✅‫ء ﻻزم ﻳﻮﻗﻔﻪ‬Ô Õ
o ‫ﻬﻬﻬﻢ‬¹‫ﺬا ا‬¹ .
🌹Most preventable cause of DLP and CAD in female?
A. physical inactivity B. diet. C. poor quality sleep D. smoking✅
🍓Pt admitted as case of cardiac disease, during hospital course, he developed
something related to urinary catheter, what you will do to prevent that:
daily assessment of needs of urine catheter✅
🌹Elderly conscious, alert, oriented, receiving nsaids and codeine on urinary catheter, after
2 days he developed confusion and agitation, the nurse noticed cloudy urine, labs:T: 38.9
RR: 18 HR:99, What’s the most important thing to prevent this condition?
A. avoid codeine in elderly B. assess the daily requirements of urinary cath. ✅
🌹Old man fall down and has hx so they put for him FC due to immobility Given
paracetamol and Codeine for pain Then he develop fever And nurse noticed cloudy urine
Mx? A-Review the need for Foly cath✅
B-Request for member of the family turning the pt continuously
C-Codeine not good for this age group

🍓MI chest pain STEMI and no pci and you will transfer him, what to give prior to transfer :
A. give asprin , thrombolytic , bb , Nitroglyrcrn✅
B. give asprin , nnitroglycerin , bb.
......
🍓Chest pain after new exercise program? reassurance and re evaluate after I week
🍓patient with cannon A waves and raised JVP asking about the mx ?
A. ICD B. Permanent pacemaker ✅ pac ‫–ﻌﺔ‬è= ‫ﻌﺔ „ﺎﻧﻮن‬u‫ﻂ =„ﺎﻧﻮن =ﻃﺎ‬Ï‫ر‬
🌹second heart block mobliz II = A. – syncope B. - permanent pacemaker✅
• ç ‫ رﻗﻢ اﺛﻨ‬II ñ
ñ • ç ‫ = ﻣﻊ „ﻠﻤﺘ‬Permaent ‫ و‬pace mark
🌹what is the most common case of death in pt with dialysis = IHD🌹

🌹Modifiable risk factor for htn?


A. BMI ✅ ‫ﻦ اﻟﺴﻤﻨﺔ اﻫﻢ‬Ä‫ ﻟ‬، ñ• ç ‫ اﻟﺘﺪﺧ‬ýÓ‫ ﺣ‬B. Smoking C. Family hx D. Age
🍓 60 years old male with history of DM and HTN and smoking , low compliance on his
medication , he presenting with pain in his right limb , absent of pulse , and cold on
examination What is most common causes will lead to death in this case:
A. MI✅✅ B. Stroke
🌹Which of the following is the most significant risk factor for heart disease?
A. BMI 31. B. HTN 140/90. C. Waist 103 cm D. Two readings of fasting glucose ( 8.2, 8.3) ✅
🌹Which of the following is considered a risk for htn? Childhood obesity 🌹

🌹Case of MI with hypotension just what to give? 🌹


A. Dobutamine B. IV Fluids ✅✅ C. Others
🌹Case of MI with hypotension + Bradycardia +Cardiogenic shock just what to give? =
atropin

Prophylax Antibiotic
🌹 Patient going for elective hernia surgery. Known to have bicuspid + aortic valve and
prosthetic heart valve since 4 years. Best antibiotic prophylaxis?
○ Cefuroxime ○Amoxicillin ○ No prophylax is required✅
bicuspid aortic valve = no need to prophlax
🌹 Pt w/t mitral valve prolapse and she will undergo for thyroidectomy ..
A- Give amoxicillin to prevent infective endocarditis B- No need for prophylaxis ✅✅
🌹 62 yrs male idiopathic subvalvular hypertrophic aortic stenosis going for
dental extraction :
-Risk of endocarditis is 50% Ö-Risk of endocarditis is 12%
-Prophylactic AB not needed -After procedure AB is sufficient
‫ﺤﺘﺎج ل‬7 56‫ ا‬34‫ اﻟﻤﺮ‬، ‫ﺐ ﻧﺮﻛﺰ )ﺎﻟﺴﺆوال‬#‫ ﻃ‬prophlaxsis antibiotic )$ % ‫ﺪال ﺻﻤﺎم اﻟﻘﻠﺐ ب ﺻﻤﺎم (ﺻﻨﺎ‬IY‫ ﻋﻤﻠﻮا اﺳ‬56‫ء )ﺎﻻﺳﻨﺎن ﻫﻢ ا‬N
O
6 ‫ﻞ اﺟﺮاء اي‬I‫ﻗ‬
، ‫ﺔ‬#‫ﺔ ﻋﺎﻟ‬I‫ﺴ‬k‫ﺠﻴﻬﻢ ﺑ‬7 ‫ﺔ ﺧﻠﻊ )ﺎﻻﺳﻨﺎن اﻧﻪ‬#‫ اذا ﻋﻤﻠﻮا اي ﻋﻤﻠ‬a6 ‫ﺪﻟﻮا ﺻﻤﺎم اﻟﻘﻠﺐ ب ﺻﻨﺎ‬IY‫ اﺳ‬56‫ ﻻن وﺟﺪوا ا‬infective endocarditis ‫ﻦ ﻫﻨﺎ اﻟﺴﺆوال ﻣﺎ‬p‫ﻟ‬
4
‫ﺎﺧﺬ ﻣﻀﺎد‬7 ‫ ﻣﻔﺮوض‬56‫„ ﻣﻨﻬﻢ ﻫﺬا اﻟﻤﺮض ا‬6 ‫ﺪ ﻻﻳﻦ ﻣﺎ‬7‫ف ﻋ• ﺣﺴﺐ اﻟﺠﺎ‬. ‫ ﺛﺎل ﻋﻨﺪە ﺿﻴﻖ )ﺎﻟﺼﻤﺎم اﻻور| وﺗﻀﺨﻢ‬، a 6 ‫ﻗﺎل اﻧﻪ اﻟﺸﺨﺺ ﻋﻨﺪە ﺻﻤﺎم ﺻﻨﺎ‬
4
‫‹ا ﺗﻄﻠﻌﻮا ﻋﻠﻴﻬﻢ‬Ž‫ﺎﺧﺬ ﻣﻀﺎد ﺣﻴﻮي اذا ﺗﺤ‬7 ‫ ﻻزم‬a6 ‫‹ اﻟﺼﻤﺎم اﻟﺼﻨﺎ‬Œ‫ﺔ ﻏ‬#‫ﻪ اﻣﺮاض ﺛﺎﻧ‬#‫ﺐ ﻓ‬#‫ ﻃ‬، ‫ﺔ اﻟﺨﻠﻊ‬#‫ﻞ ﻋﻤﻠ‬#‫ ﺣﻴﻮي ﻗ‬:
🌷idiopathic subvalvular hypertrophic aortic stenosis = no need to antibiotic Prophylactic
before dental Extraction 🌷

🌹Patient with multi drugs use, with hyperkalemia 6.5, what drug to stop:
A. BB B. Aspirin C. ACE
🌹 Best drug to treat Pulmonary ht , without cardiac causes?
A. Furosemide B. Spironolactone C. Bb D. ACEI Answer is : CCB
🌹Decrease mortality in inferior-lateral MI hospital stay ? A. BB B. Warfarin C. Thizaid
...
🌹patient with CHF and DM and HTN not tolerating ACEi what to switch it
with? ARBS
🌹Patient with HV and LV dysfunction giv? A. BB. B. Digoxin. C. ARB
🌹She said the full scenario Is Young fainting with exertion Dilated cardiomyopathy No EF
mentioned New presentation no comorbidities: A. Options. B. CCB. C. BB. D. Diuretics
🌹CHF on acei and many medication for dm now stable and asymptomatic :
A. Add bb ✅ B. Add furudmide C. Add digixon
🍒Dm HTN and renal Failure on CCB, ACEI and thiazide and now hypertensive what to do
A. change thi to fursemid B. increase doses C. add BB ✅

🌹Man in early 30s with HTN, his parents and sister also have HTN, what’s the
best antihypertensive for him? A. BB B. ACEI C. CCB
🌹 Elderly women in her 70s with DM & HTN, normal renal profile, which antihypertensive
should be started? A. thiazide B. ACEI C. BB D. CCB
🌹Male came for presport screening asymptomatic Found to have LV enlarged , EF 40 , ECG
normal : A. Frusemide B. Follow up after 6 months C- ACEi

🌹Patient after MI treated with thrombolysis and discharged. Came back with tongue and
facial swelling, what drug causes this? A. Statin B. Atenolol C. ACEI
....
#Patient with HV and LV dysfunction give: A. BB B. Digoxin. C. ARB
# If HF + Afib and how to control rate? Digoxin
🌹 Pt with Afib and chf what's the appropriate drug to add?
A. digoxin. B. Adenosine. C. Lidocaine
#If EF < 40= ACE
…..
● 🌹Patient on ACE develop cough? Switch to losartan
….
induced angioedema: ACEI
….
🌹How to reduce protein in urine in nephrotic patients? ACEI
🍓 drugs used in a heart failure pt = Start by ACEI as the first line 🍓
🍓 pt known to have DM and HTN with proteinurea what is the most appropriate:
A. decrease protein in take B. give him ACEI✅
🌹Pre employment screening, pt has cardiomegaly on CXR, echo > left ventricle
enlargement and EF40%. What is the management ? ACEI as he has systolic HF
🍒Young man in pre employee clinic, medically free, echo done and showed EF = 40% but
no signs or symptoms of heart failure, what to do:
A. Give BB. B. Give ACE inhibitor✅ C. Repeat echo after 6 months
. .. . . .. .
🌹77 yo male known diabetic with history of MI now has HTN 170/110 what will u give
a. ACEI b. BB c. CCB d. Thiazides
...
🌹 Patient with diabetes and HTN and CHF was prescribed warfarin for presumed DVT now
has edema and investigations given. K=6.5 aptt high pt normal. He’s on ACEI , insulin and
furosemide .which med will u stop ? a. warfarin b. ACEI c. Diuretic d. Insulin
🌹77 yo male known diabetic with history of MI now has HTN 170/110 what
will u give? A. ACEI B. BB C. CCB D. Thiazides
🌹Pt has Heart failler and co morbidties and on multiple drugs.. he now stable only K+ = 6 ..
which drug you should stop now : A.BB B. ACEi ✅ C.Digoxin D. Heparin
🌹 Drugs decrease mortality in HF A. ACEI. B. Digoxin
🌹patient take ACEI, CCBs, thiazide , develop LL edma mainly , other Normal but in
investigation all RFT test deteriorated, what you will do
A- Stop ACEI B- Change thazide to frismide C- No change
ACEI not contraindicated in CKD
But suddenly elevated BUN and Creatinine > stop ACEI ,Because we don’t know about the
cause could be bilateral renal artery stenosis
......
🌹Patient with diabetes and HTN and CHF was prescribed warfarin fornpresumed DVT now
has edema and investigations given. K=6.5 aptt high pt normal. He’s on ACEI , insulin and
furosemide .which med will u stop ? A. warfarin B. ACEI C. Diuretic D. Insulin
‫ „ﻠﻴﺘﻪ‬¤‫ •ﺤﺎﻓﻆ ﻋ‬ýÓ‫ ﺣ‬. acei ‫–ﺾ ﺳﻜﺮ وﺿﻐﻂ ﻧﻌﻄ~ﻪ ﻋﻼج اﻟﻀﻐﻂ‬À‫ﻣ‬
🌹50 years male take antihypertensive : (ARBs, CCBs) , RFTs= Normal Which drug could be
taken in this case without C/I : A_ACEI B- Atenolol C_Thiazadin
.....
🌹two qs about prevention of re infarction : A- Anti platelet B- Surgery
🌹Female in her fifties develops MI and she was asking about the best way to prevent a
reinfarction: A-Use of aspirin for short duration B-Use of Calcium channel blocker
indefinitely C-Use of ACEI if she develops heart failure to prevent cardiac changes
...

….
🌹Patient k/c of heart failure, presented to ER with image of acute CHF (SOB,
orthopnea, PND, abdominal swelling, hepatomgaly and LL edema) Echo was done, showed
EF 40, what’s your management: A. Start spironolactone
B. Start Furosemide ✅✅ ‫ اﻟﺮﺋﺔ‬ŒŽ • ‫ ﻣﺘﺠﻤﻌﺔ‬ÐŽ‫ ﻣﺪرات ﻋﺸﺎن اﻟﺴﻮاﺋﻞ ا‬C. ACEI D. Nifedipine
.....
🌹HF pt on ( ACEI , bb , furosemid ) what to add ?
A. spironolactone✅ B. isosorbide dinitrate
🌹patient presented with sweating; dyspnea, palpitations and headache. She is
a known case of HTN and despite taking meds it’s not controlled. Imaging reveals
suprarenal mass. Which of the following meds will you give to control her HTN?
A. CCB B. alpha block C. BB D. ACEI
....

🌹Postural hypotension that caused by Beta block , what does prevent this side effect ?
A. Avoid airplane for for 2 w ✅ B.Avoid airplane for 4 w ñ • ç ‫ﻠﻮك =„ﻠﻤﺘ‬u ‫ =ﺑ|ﺘﺎ‬ñ
• ç ‫ﻂ =اﺳﺒﻮﻋ‬Ï‫ ر‬.
🌹Female hypertensive and diabetic, on Acei, insulin, Metformin, she decided to get
pregnant soon. Labs: heavy proteinuria, Hga1c: 8 What’s your most appropriate advice for
her regarding diabetic control and fetal congenital malformations?
A.stop snacks and start 3 heavy meals a day.
B.switch ACEI to ARB’s
C.try to control her hga1c to normal or near normal as possible before pregnancy. ✅

🌹 WPW syndrome treatment procainamide ✅🌹


‫ﻂ =ﺑﺮوك‬Ï‫ ر‬proc = ‫وك =واو‬Ò£‫ ﻣ‬waw . 🎁🎁
🌹Wpw syndrome unresponsive to atenolol. Next:
A. Increase the dose. B. Give amiodarone
. . . . . .
🌹 CHF, DM,HTN patient and on thiazide, atrovastatin, ramipril, and amlodipine, presented
with progressive +3 lower limb edema, cardio and pulmonary exams are normal, his lab
tests are also normal, what is the management:
A. Change the thiazide to furosemide B. No change in his medications
C.Add bisoprolol ✅ 🌹(beta block) .
<
‫&ﺾ ﻋﻨﺪە‬%‫ اي ﻣ‬CHF ‫ﺪا‬2‫ﻠﻮك ﺗﺤﺪ‬B ‫ﺲ اي دواء ﺑ?ﺘﺎ‬J‫ﻠﻮك وﻟ‬B ‫ دواء ﺑ?ﺘﺎ‬9‫ﺎﻻﺿﺎﻓﺔ ا‬B ‫; اﻟﺴ?ﻨﺎر&ﻮا‬: 9:‫ﺎﺧﺬ اﻻدو&ﺔ ا‬2 ‫ ﻻزم‬bisoprolol ‫ او‬metaprlol
<
.;: ‫اﻫﻢ اﻫﻢ دواء‬. ^_‫ ﻣﻦ ﻛﺘﺎب ﻛﻮرﻧ‬E Inhibitors and ARBs.
These agents should be given to all patients with systolic dysfunction at any stage

🍒Elderly KC of DM and uncontrolled hypertensive, history of chest tightness with


exertion, he is on insulin, ACE, nitroglycerin, what to add?
A.metoprolol ✅. B.diltiazem. C.nifedipine. D.Amlodipine
🌹left ventrical failler whats the best test to diagnosis ?? Brain natriurtic peptide
‫ ﻋﺎﻟﯾﺔ اﻋرﻓوا ﻓﯾﮫ ﻣﺷﻛﻠﺔ‬BNP ‫ اول ﻣﺎ ﺗﺷوﻓون‬، ‫ او اي ﻣﺻﯾﺑﺔ ﻓﯾﮭﺎ ﺳواء ﺟﻠطﺔ او ﺗﺿﺧم‬left ventrical ‫ ﻻاا ﯾﺧرج اﻻ ﻓﻘط ﻓس ﺣﺻول ﻓﺷل ﻓﻲ‬BNP ‫ھذا‬
. Left ventrical ‫ﺑﺎل‬
🌹73 years old male with high BP recorded at home for 1 week and in hospital before his
surgery 200/190 . He is on 3 antihypertensive medication what is the cause of his high BP
A. NSAID B. Pseudoephedrine 🌻If high K >> NSAID. 🌻If low K >> pseudo.
🌹 Elderly patient with Ibuprofen use, Came with High Reading BP 160/99,
with electrolyte disturbance (can't remember) no sign of headache,
visual disturb mentionA. NSAID induced. B. Essential HTN C. Pheochromocytoma
‫ ﻣﻨﮭﺎ ارﺗﻔﺎع اﻟﺒﻮﺗﺎﺳﯿﻮم وﯾﺴﺒﺐ ارﺗﻔﺎع ﺑﺼﻐﻂ اﻟﺪم وﺧﻠﻞ‬various electrolyte ‫ ﯾﺴﺒﺐ‬NSAD ‫ادوﯾﺔ‬
‫ﺑﻮظﺎﺋﻒ اﻟﻜﻠﻰ‬
🌹🌹case about what the management of chronic condition of supraventrical tachy?
*SVT management* ‫ﻣﻦ ﺟﻠﻮري‬
*First* :
Vagal valsalva maneuvers(carotid massage) Then IV adenosine
*If ineffective:* IV bb, diltiazem or Verapamil.
*If all ineffective or unstable:* Cardioversion 🌹🌹

🍒 hypotension , raised Jvp, muffled heart sounds = Cardiac tamponade🍒


..
🍒 What important investigation to pt have LVH ( left ventrical hylertrophy )? Echo for EF

male with epigastric fullness. On examination, there is pulsatile abdominal mass. What
investigation you would order? A. bilirubin B. Amylase* ✅✅ Its case of (AAA)
🌹Same scenario Pulsatile abd mass Ab x ray showed no air level What test :amylase
🍒 Wolff-Parkinson-White syndrome syndrome case not respond to Tx what is next step
is : Radiofrequent ablation
….
🍒hx of DM , heart failure and admitted for hysterectomy and received Normal Salin cuz
poor oral intake, 4 d nurse noticed decreases in Sat and SOB , o/e : crackles
How to prevent this complication ? A- cardiopulmonary consult B- daily fluid assessment✅
....
🍒Elderly on 3 antihypertensives, but still uncontrolled, what is the reason ?
Ask if he is use Pusedoephedrine drug for decongestants nasal Bez its rise blood presser
and its contraindicated in patients with type 2 diabetes and Hypertensive 🍒

🌹Chronic liver patient ,with shistosomiasis with right sided heart failure , what is
complications developed? Constrictive pericarditis🌹
... .
SOB upon activity and relieved with rest, no chest pain no pressure on chest. What’s it?
a) musculoskeletal ‫ﻣﺎﻳ—–ﺪ ﻣﻊ اﻟﻤﺠﻬﻮد‬
‫ﺎﻟﻌﻀﻠﺔ‬u ‫ ﻻن اﻟﺘﻬﺎب‬، ‫ﺎﻟﻤﺠﻬﻮد‬u ‫ اﻟﺼﺪر ﻣﺎﻟﻪ ﻋﻼﻗﺔ‬ŒŽ • ‫ﺎﻟﻢ‬u ‫ ﺳﺎﻋﺔ ﺗﺤﺲ‬٢٤ ‫ •ﺠ~ﻚ‬.
b) angina-equivalent ✅ ýŽ • ‫ و–ﺨﻒ ﻣﻊ اﻟﺮاﺣﺔ •ﻌ‬، ‫ اﻟﺤﺰن اﻟﺸﺪ•ﺪ‬، ‫ اﻟ†– ـ ـﻊ‬/ ¼
Ž ‫ ﻣﺜﻞ اﻟﻤ‬، ‫ﻳ—–ﺪ ﻣﻊ اي ﻣﺠﻬﻮر‬
/ ¼ ¼
Ž ‫ واذا ارﺗﺤﺖ ﻳﺮوح اﻻﻟﻢ وا™ﻤﻞ ﻣ‬، ‫ واﺣﺲ ب اﻟﻢ‬/ Ž ‫اﻣ‬
C) MI ‫ﺪا ﻣﻊ اﻟﺮاﺣﺔ‬u‫ﻣﺎ•ﺨﻒ ا‬
d) unstable angina ‫ﻣﺎ•ﺨﻒ ﻣﻊ اﻟﺮاﺣﺔ‬
. . . .. . . .
🌹Pt with chest pain during activty and relive by Rest , there is no chest tightnes =
Stable angina 🌹
‫ ﻣﻊ‬/ ¼ ¼ •Ó
Ž ‫ ﻣﻊ ﺳﺘﺎ•ﻞ اﻧﺠﻴﻨﺎ ﻫﺬا ﻣﺎ •ﻤ‬/Ž ¨‫ اﻫﻢ „ﻠﻤﺔ اﻧﻪ ﻳﺮوح ﻣﻊ اﻟﺮاﺣﺔ ﻫﺬا ﺗﻤﺎﻣﺎ •ﻤ‬muscle pain ‫ ﻣﺎﻟﻪ‬ñ‫اﻻم ﺗﻠﻌﻀﻠ‬
‫ﺎﻟﺮاﺣﺔ‬u ‫ واﻧﻪ •ﺨﻒ‬، ‫ﺸﺎط‬2‫ﺎﻟ‬u ‫ﻋﻼﻗﺔ‬
.. . .
male pt with retrosternal pain radiates to the left shoulder at rest , positive
cardiac enzymes , ECG show t wave inversion in lead V2 - V5 , what’s the dx :
A. stable angina B. unstable angina C. STEMI D. Non-STEMI✅✅
.....
🌹Old pt with chest pain respond well to nitroglycerin = A. unstable angina B. MI C. stable
angina
....
🌹Old + hx of chest pain increased in intensity last 3 weeks even at rest what is the type of
angina:
A-Exertion B-prinzmetal C-unstable
‫ „ﻠﻤﺔ‬even at rest ‫ ﺣﺪدت‬ÐŽ‫ ا‬4 Ž unstable
...
🌹Patient k/c of DM HTN came with unstable angina treated with statin , aspirin , b
blocker , heparin , nitrate What to add ? A-Candesartan B-Clopidogrel C- CCB
...
🌹 Old pt present with retrosternal chest pain with exertion relieves by rest, all normal
except with S4, on ECG there is T wave inversion in lead V2-V5= Stable angina.
‫ ﻣﻊ‬Þ Õ
o ‫ﺒ}”ﺎل “ﻤ‬Û‫ﺬا ﺗ‬¹= ‫ﺮوح ﻣﻊ اﻟﺮاﺣﺔ‬w‫ﺎﺿﺔ و‬wv‫ﺪ ﻣﻊ اﻟ‬wÙ‫ ﻗﺎﻟﻚ اﻟﻢ ﻳ‬stable .
....
🌹21 Diabetic patient with central chest pain occur after exercise and last 15 minutes
relieve by rest ECG was given with depressed st segment in lead v1 & v3 ? Case of stable
angina.
A.urgent echo B.refer to cardiologist✅ C.ask for cardiac enzymes and come back again
next day D.tradmil ECG
….
🌹Pt presented with unstable angina ECG: NSTEMI. He’s managed with aspirin, bisoprolol,
enoxaparin, atorvastatin, what medication should be added to his medical management?
A. Candesartan B. Clopidogril
....
🌹 51 yrs old diagnosed of unstable angina he’s on aspirin, statin ( other medication) but
still has sx what to add:
A. bb ( name of the drug) B. nifedipine C. ccb ( name of drug)
....
symptoms of Classic Stable Angina (gradual onset chest pain with exertion? Answer: Stable
angina ✅ ✔

stable angina defined as: Chest pain reileved by rest or nitroglycerine last for short
duration exacerbated by emotion excersie .
….
Patient with chest pain during exercise, normal resting ECG, what to order? Exercise Stress
ECG ✅

question about Drugs lower mortality in MI patients?Aspirin, B blocker, Tpa, PCI
Nitrates = No mortality benefit . Asprin = strong mortality benefit
. . . ..
question about symptoms of Unstable angina for 3 weeks (chronic angina increase in
frequency and duration and intensity..? Unstable angina ✅ ✔
.....
ñ• ç ‫ﺎﺧﺘﺼﺎر ﺑ‬u ‫ اﻟﻔﺮق‬stable and unstable angina:
‫–ﻖ ﺑ|ﻨﻬﻢ‬À‫ اﻟﺘﻔ‬ŒŽ • ‫ اﻫﻢ اﻫﻢ „ﻠﻤﺔ‬:
‫ ان‬stable angina ‫ ﻟﻤﺎ •ﺎﺧﺬ‬nitrats ))ïŽ • ‫اﻻﻟﻢ ((•ﺨﺘ‬
‫ ان‬unstable angina ‫ •ﺎﺧﺬ‬nitrats ‫ﺔ ﺛﺎﻧ~ﺔ „ﻤﺎن اﻻﻟﻢ ﻣﻮﺟﻮد ´ﺴﺘﻤﺮ اﻻﻟﻢ ل‬y‫واﻻﻟﻢ ((ﻣﻮﺟﻮد ))ﻳﺮﺟﻊ •ﺎﺧﺬ ﺣ‬
‫ﺎﻟﺴﺆوال‬u ¢ ¼ • ô
Ž £ • ‫ ﻻزم‬ÐŽ‫ﺲ ﻫﺬا اﻟ† اﻟﻤﻬﻢ ا‬7 ‫ ﻓ~ﻪ اﺷ~ﺎء ﺛﺎﻧ~ﺔ ﺗﻔﺮق‬، ‫»ء‬Ž ‫ اﻫﻢ‬4 Ž ñç‫ﻠﻤﺘ‬6‫ﻫﺬە اﻟ‬. ‫ ﻣﻦ ﻧﺺ ﺳﺎﻋﺔ‬ÒÙ‫ ا‬.
.........
ECG pic with case of inferior MI, how to manage? IVF then MONA.
/2‫ﻼم ﻋﺸﺎن ﻣﺎ ﻧ‬6‫ اﻟ‬ŒŽ • ‫ ﻧﺮﻛﺰ‬:
‫* اوﻻ اذا ﺷﻮﻓﻨﺎ‬inferior MI* ‫ﺎ •ﻜﻮن ﻣﻌﺎە‬y‫ ﻏﺎااﻟ‬:
((Right ventricular infarction))
‫ع ﻣﻦ‬c‫ وﻣﺸ•ﻠﺔ ﻫﺬا اﻟﻨ‬MI ‫ﺐ‬Þ‫ﺴ‬7 ‫ﻞ‬u ‫ •ﻤﻮﺗﻮن ﻣﻨﻪ ﻟ¨ﺲ ﻋﺸﺎن اﻟﺠﻠﻄﺔ‬8• ‫* اﻟﻤﺮ‬hypotension* ، ‫ﺸﻮﻓﻮن‬â ‫اول ﻣﺎ‬
‫ „ﻠﻤﺔ‬inferior mi ‫ ﻣﻊ‬right ventrical ‫ﻢ „ﻠﻤﺔ‬6‫ﺎﻟ‬u ŒŽ • ‫ اﻟﺨﻮف ﻣﻦ (( ﺣﻄﻮا‬hypotenstion )) , ‫ﻟﺬﻟﻚ اول ﻣﺎ‬
‫–ﺾ ﻋﻨﺪە‬À‫ ﻧﻌﺮف ﻫﺬا اﻟﻤ‬inferior mi ‫†ﻋﺔ ب‬7 ‫( راح اﻟﺤﻖ ﻋﻠ~ﻪ‬I.v fluid ) ‫ ارﻓﻊ اﻟﻀﻐﻂ *ﺛﻢ‬ýÓ‫*اﻋﻄ~ﻪ ﺳﻮاﺋﻞ ﺣ‬
‫ اﺧﺘﺼﺎرە‬ÐŽ‫ﺎﻟﻌﻼج ﺣﻘﻨﺎ ا‬u ‫ اﻋﺎﻟﺞ اﻟﺠﻠﻄﺔ‬MONA .
‫( ﻓﺎﻟﺠﻮاب ﻫﻨﺎ ﻧﻌﻄ~ﻪ اوﻻ‬i.v fluid ) ‫ﺛﻢ ﻋﻼج اﻟﺠﻠﻄﺔ‬.
*Inferior MI + Right ventricular infarction = hypotenstion = treat by ((I.V Fluid ))*
Right ventricular infarction should always be considered in any patient who has inferior wall
myocardial infarction and associated hypotension ,fluids should be given to the RVI patient
to maintain cardiac output, blood pressures and coronary artery filling pressures. Normal
saline up to 40 ml/kg is the preferred fluid.
..... ...
scenario about Mobitz type 2, how to manage? Pacemaker ✅ ✔
Mobitz 1 no ttt unless symptomatic, Mobitz 2 always pacemaker.
..... ....
patient with HTN Emergency, How to manage? Answer: Na Nitroprusside ✅ ✔
‫ اﻗﻮل ﻣﺼﻄﻠﺢ‬ýÓ‫ اوﻻ ﻣ‬Hypertnstion emergency ? ‫ ﻟﻤﺎ •ﻜﻮن اﻟﻀﻐﻂ ﻣﺴﺎوي او ﻓﻮق‬:
180/120
‫ ﻟﻮ اﻗﻞ ﻣﻦ ﻛﺬا ﻻ اﻃﻠﻖ ﻋﻠ~ﻪ‬HTN emergency ,
N.B: HTN emergency definition :- equal or more than 180/120 + end organ damage, without
organ damage is urgency.
‫ ا´ﺶ اﻋﻄ~ﻪ ؟‬١٢٠ /١٨٠ ‫¸ واﺣﺪ ﺿﻐﻄﻪ‬Ž • ‫ﻃ~ﺐ اذا ﺟﺎ‬
*Na Nitroprusside*
‫ واﻷوردة‬ñ • ç ‫¸ ﻗﻮي ﻟﻠ¼†اﻳ‬á‫ و ﻣﻮﺳﻊ وﻋﺎ‬Òç‫ﺸ•ﻞ ﺧﻄ‬7 ‫وا ﺑﺮوﺳ~ﺪ ﻫﻮ ﻟﻌﻼج ﺿﻐﻂ اﻟﺪم اﻟﻤﺮﺗﻔﻊ‬ÒÓ‫ﻧﻴ‬
Ž
*HTN emergency = Na Nitroprusside*
.. . . . . .
🌹 Case of AAA ( abdominal aortic aneurysm) what electrolyte abnormality you will find?
hypomagnesemia ✅ ✔ ýŽ • ‫ﻄ‬u ŒŽ • ‫ =ﻋﻨﺪي ﻣﻐﺺ‬، ‫ﻂ =ﻣﻐﻨ¨ﺴﻴﻮم =ﻣﻐﺺ‬Ï‫🤕 ر‬

🌹Pulsatile abdominal mass, mild tenderness on palpation, most appropriate


diagnostic test? A. US B. CTA C. MRA

🌹Patient on parenteral feeding developed weakness and convulsions what cause these
symptoms : A. hypokalemia B. hypomagnesemia
….
🌹Patient on (Total parenteral nutrition) devoloped weakness in the lower limps and
tremors what is the problem? A-Hyperkalima B-Hypomagnisemia✅ C-Hypoglycemia
((Possible complications associated with TPN include: Dehydration and electrolyte
Imbalances. Thrombosis (blood clots)
Mg ‫ﻓﻤﻦﻓﻣن ﻣﺿﺎﻋﻔﺎت اﻟﻐذاء ھذا اﻧﮫ ﯾﺣﺻل ﺧﻠل ﻓﻲ اﻻﻛﻠﺗروﻻﯾت اي ﻧوع ﻣن اﻻﻟﻛﺗروﻻﯾت ھذا؟‬
mg‫ ﻧﻔﻛر ﺑﺎل‬tremor ‫ ﻓﻠﻣﺎ ﯾﺳﯾر ﺿﻌف و‬mg , ‫ ﺑﺳﺑب اﻧﺧﻔﺎض‬convultion ‫ﻻن ﻣﺛل ﻣﺎ ﻗﻠﻧﺎ واﺣد ﺳﺎر ﻟﮫ‬
......

🌹weakness in the lower limps and tremors what is the problem?


A-Hyperkalima B-Hypomagnisemia✅
‫ و اﻟﻣﻌﻧﯾﺳﯾوم اذا ﻧزل ﯾﻌﻣل ﺿﻌف وﺗوﺗر ﻓﻲ‬hypo ‫ ﺑﻣﺎ اﻧﮫ ﺿﻌف ﯾﻌﻧﻲ‬، ‫ﻧرﺑطﮭﺎ = ﺿﻌف ﻓﻲ اﻟﺟزء اﻟﺳﻔﻠﻲ‬
‫ ﻧﻘﺻﺎن اﻟﻣﻐﻧﺳﯾوم ﯾﺎﺛر ﻋﻠﻰ اﻟﻌﺿﻼت وﯾﺿﻌف ﺣرﻛﺗﮭﺎ ﻣﯾﺣﺻل ﺗرﯾﻣور‬، Muscle ‫ ﻣﻊ‬Mg ‫ ﻓﻣن اﺳﻣﮭﺎ‬، ‫اﻟﻌﺿﻼت‬

🌹pt in 20s c/o sharp pain centrally after activity for 3 weeks .. what next:
A- Reassure and follow up 1 week B- Nitrate C- Ibuprofen✅✅
‫ﻧﺎ‬ÒÓ‫( ﻟ~ﻪ ﻣﺎ اﺧ‬A) , ‫ ﻻن •ﻘﻮﻟﻚ ﻋﻨﺪە‬sharp pain ‫»ء‬ ¼
Ž ‫ ﺛﺎﻧ~ﺎ ﻟ~ﻪ ﻣﺎﻗﻠﻨﺎ اﻧﻪ‬،! ‫ﺪون ﻣﺴﻜﻦ‬u ‫اﻟﻢ ﺷﺪ•ﺪ ﻣﺎ ﻳﻨﻔﻊ اﻃﻠﻌﻪ‬
angina ‫ ﻣﺎﻋﻄﻴﻨﺎە‬ýŽ • ‫ﺎﻟﻘﻠﺐ •ﻌ‬u nitrate ‫ﺤﺔ‬u‫ اﻟﻢ اﻟﻘﻠﺐ او اﻟﺬ‬، ‫ اﺳﺎﺑﻴﻊ ﻣﺘﻮاﺻﻠﻪ‬٣ ‫ ﺛﺎﻧ~ﺎ •ﻘﻮﻟﻚ‬، Òç‫ﻻن اوﻻ ﻋﻤﺮە ﺻﻐ‬
angina ‫ﻌﺪ اﻟﺮاﺣﺔ‬u ‫ دﻗﺎﻳﻖ و–ﺮوح اﻻﻟﻢ‬٥ ، ‫ﺎﻟﻤﺪة ﻫﺬە‬u ‫ﻣﺎ´ﺴﺘﻤﺮ‬.
🌹 pt in 20s c/o sharp pain centrally after activity for 3 weeks = Ibuprofen🌹
........
75 y/o male k/c of HTN, DM, Hx of TIA presented with palpitation. ECG demonstrated 75
bpm, irregular. What is the best next step? give anticoagulation (( susbect Atriall
fibbralantion ))
.............
female came complaining from sudden episodes of palpitations each 10-15 min that are
unrelated to any activity. You did ECG which was unremarkable. What is your next step?
A. Echo B. Stress ECG C. Holter monitoring ✅✅

‫ﺎت اﻟﻘﻠﺐ ﻣﺜﻞ‬Ï@ ‫ ﻫﺬا ﺟﻬﺎز •ﻌﻤﻞ ﻗ~ﺎس ل ﻋﺪم اﻧﺘﻈﺎم‬ECG ‫ ﺳﺎﻋﺔ‬24 ‫ﻦ •ﻜﻮن ﺷﻐﺎل و–ﻘ¨ﺲ ﻟﻤﺪة‬Ä‫ﻂ ﻟ‬y‫ﺎﻟﻀ‬u
ñ• ç ‫ ﺳﺎﻋﺔ ﻣﺘﻮاﺻﻠ‬٤٨ ‫ﻌﻀﻬﺎ‬Ï‫ و‬.
🍒 complaining from sudden episodes of palpitations each 10-15 min that are unrelated to
any activity ==Holter monitoring🍒🍒
.....
patient with heart failure. Which of the following medication improve mortality?
A. ACEI (enalapril)* ✅✅ B. Digoxin C. Diuretic
..........
patient with chest pain for 2 hours. ECG showed ST elevation in V2-V4. Patient started on
Aspirin, Nitroglycerin, and O2. What is the definitive management?
A. Thrombolysis B. Angioplasty ✅✅
(PCI), also known as coronary angioplasty
🍒🍒chest pain for 2 hours. ECG showed ST elevation in V2-V4=PCI (angioplasty )
.... ......
patient with raised JVP, Hypotension, unclear heart sound, clear lung sounds. How to
confirm the Dx?
A. Echo ✅✅ B. CXR C. ECG
🍒raised JVP, Hypotension, unclear heart sound == coniform diagnosis = Echo🍒

🍒🍒epigastric pain and pulsatile abdominal mass = to confirm the diagnosis= CT with
contrast*🍒🍒
......
🍒🍒 pulsatile abdominal mass. What investigation you would order = Amylase🍒

🍒3rd block : ECG asking about management == pacemaker 🍒

🌹ECG show atrial Fib + lower limp edema and pain = LMWH
‫ وﻫﻨﺎ ﺣﺼﻞ‬، ‫–|ﻴﻊ‬è ‫ﺎر–ﻦ ﻣﻔﻌﻮﻟﻪ‬y‫–ﻦ ؟؟ ﻻن اﻟﻬﻴ‬À‫ﻧﺎ وارﻓ‬ÒÓ‫ ﻟ~ﻪ ﻣﺎ اﺧ‬DVT ‫وﻧﺨﺎف ﻓﻮرا ﺗﺘﺤﺮك اﻟﺠﻠﻄﺔ وﺗﺮوح ﻟﻠﺮﺋﺔ ف‬
‫ •ﺠ~ﺐ اﻟﻤﻔﻌﻮل‬ýÓ‫ ﺧﻤﺲ ا•ﺎم ﺣ‬Ð‫ﻌﺔ ا‬Ï‫–ﻦ •ﺤﺘﺎج ار‬À‫اﻟﻮارﻓ‬. ‫– ـ ـﻊ‬è ‫ﺎر–ﻦ ﻣﻔﻌﻮﻟﻪ‬y‫ﻫﻴ‬
…..
🌹ECG with Cannon a waves = pacemarker permenant 🌹

🌹Pt Elderly with hypertenstion + on routin cheak up and on cardiac exsam we found
there is (sever aortic stenosis ) and ( left ventrical failler ) with normal EF = PT
Asymptomatic = Aortic valve replacment
(Another Wronf choice = ACEI , beta block ; follow up 6 month = ‫ﻠﻬﺎ ﺧﻄﺎاا‬á ) .. 🌹
. . . ..
🌹Most important factor to do surgery in aortic stenosis? A.Pt symptoms ✅ B.Lt ventricle
hypertrophy
.............
🌹Aortic stenosis with left ventrical hypertrophy came 2 times once with medication name
and one time with out ?
A. control hypertension B. vasodilators C. valve plasty if symptomatic or low EF > AVR
....
🌹AS with LVH asymptomatic EF40 = A. Follow up in 6 months m B. AV Replacement

🌹-Case with finding of S1Q3T3 on ECG ” :- pulmonary embolism



🌹Pt with pluritc chest pain and ECG pic of diffuse elavated st . mx ? NSAID 🌹

🍓ECG of pericarditis (diffuse elavated st )management of pericarditis? NSAID as


lbuorofen 🍓

🌹Patient with chest pain, ECG done shows diffuse ST segment elevation in all leads .
What is the treatment?
A. NSAID B. corticosteroid
....
🌹ECG picture of tall t wave , mx? Ca gluconate (antidote for hyperkalemia)
.. .....
🌹ECG of supraventricular tachychardia what treatment ? adenosine 🌹
N.B: 1st line: valsalva maneuver (carotid massage) and adenosine
If ineffective > BB,CCB
If ineffective or hemodynamically unstable> Cardioversion.
🌹 50 years Pt with htn start to have new onset of palpitations.Irregular pulse AND HR is
170. Treatment? A. amiodarone. B. adenosine ✅. C. cardioversion. D. observation

🍓pt came with yellow vision , their is ECG : digoxin toxicity


...............
🌹Patient had fatigue, dyspnea, elevated JVP, ascites and positive shifting dullness. The JVP
doesn't fall with inspiration. (I think it is constrictive pericarditis). What has the highest
diagnostic value?
A. Holter monitor B. Cardiac CT = constrictive pericarditis C. Coronary angiogram D. stress
ECG
.......
🌹Patient with recurrent episodes of palpitations. Normal cardiovascular exam. What is the
most appropriate investigation?
A. holter monitor✅ B. echo C. stress ecg
.....
🌹 Middle age diabetic male presented with persistent neck pain, ECG done showed non
specific changes, Trop is negative What is the best next step ? A. Repeat trop after 6 h✅ B.
Aspirin and nitrate C. Do right leads ECG
........
🍒ECG 2nd heart block with pic, ttt is ? A. surgery B. diuretic C. pacemaker ✅
....
🌹 Picture of ECG showing SVT, what to do now? ‫ﺐ‬ã‫ﺗ‬²n‫ {ﺎﻟ‬Þ Õ
o ‫ﻠﻬﻢ ﺻﺢ {ﺎﻻﺧﺘ}ﺎرﺗﺖ ﻟ–ﻦ ﻧﻤ‬á
A. carotid massage✅✅ B. iv adenosine C. iv another drug D. cardioversion
.....
🌹 NSTEMI management? A. Clopidogrel✅✅ B. Streptokinase
........
🌹Case of unstable MI ( ECG of S.t depression) Cardiac enzymes normal , on aspirin, statin ,
anticoagulant whats recommended to add: A. Clopidigril ✅ B. Spironolactone C. 2
unrelated
.......
🌹ECG heart block (drop of QRS complex) then I understand from the scenario that the
patient is Marfan? , he was > 95th percentile, normal weight and irregularly irregular pulse.
What is the prognosis ?
A. he will develop syncopal attacks B. he will need pacemaker✅ C. normal life
......
🌹 ECG not clear but history was about yellow vision= digoxin toxicity

🌹Patient with typical chest pain ecg shows St depressed and troponin high dx ? Non
STeMi
....
🌹Pt diagnosed with wolff parkinson syndrome and he is on BB but still
uncontrols what to do : A. increase the dose of BB B. add CCB
....
🌹Middle age male presented to ER with retrosternal chest pain. ECG shows inferior wall
MI. He was put on morphine and sublingual glycerin infusion until finish prep. To move him
to cath lab. However pt became worse and deteriorated. What is the cause of pt
deterioration?
A. Rt ventricular ischemia B. Rupture of papillary muscles C. Cardiac tamponade
.....
🌹Picture of ECG: ( sinus node dysfunction) and long scenario , asking what you will need
for management later in complication or prognosis: pacemaker

🌹Old pt on captopril , ecg shows tall T wave .best next step? check k level
..
🌹Patient presents with vague symptoms ECG was done and it shows third degree heart
block What is the DEFINITIVE management for this patient?
A. Permanent pacemaker B. Implantable cardioverter defibrillator C. Oral theophylline
....
🌹Male patient loss his vision on left eye for 20 minutes then return to vision . In
history it was DM . What the Dx ?
A. Multiple Sclerosis.B. Retinal detachment C. Conversion disorder D. Transient ischemic
attack
...
🌹66 years old patient come with progressive difficulty breath . In history he is.being
treated for bronchogenic carcinoma . In P/E : JVP elevated , lung clear and heart sound
very quiet. What’s the confirmatory investigations ?
A. CXR B. Echo C. ECG DABG
...
🌹50 years old come with sudden severe retrosternal pain radiating to back. Within the
minutes he come unconscious . He is a smoker and history of HTN . What’s Dx ?
A. vasovagal attack B. PE C. tear of aortic intima D. acute myocardial infarction
‫ال‬aortic dissection ‫ ﻫﻮ اﻧﻪ •ﺤﺼﻞ‬tear ‫ﻘﺔ‬y‫ ﻃ‬ŒŽ • intima ، ‫ اﻟﻌﻤﺮ‬Ò£‫ﺐ ارﺗﻔﺎع اﻟﻀﻐﻂ وﻣﻊ ﻛ‬Þ‫ﺴ‬7 ‫ •ﺤﺼﻞ‬،
‫ و–ﻤﺘﺪ ﻟﻠﻈﻬﺮ ﻣﻦ ورا‬à£‫ﻂ ﻧﺜﻞ اﻟﺴ|ﻨﺎر–ﻮا اﻟﻢ ﺷﺪﻳ|~ﺪ وﻣﻔﺎ‬y‫ﺎﻟﻀ‬u ‫–ﺾ‬À‫•ﺠﺴﻚ اﻟﻤ‬
...
🌹drugs that increase survival in CHF=ACE
...
🌹Young pt c/o syncope attack during macturation and cough Examination and ECG normal
?? A. Holter monitor
B. Reassure and education
....
🌹Accurate test for LT ventricular dysfunction??
A. CRP B. CKMP C. Trop l D. BNP
....
● Elderly with hypertension on furosemide and ACEI, complains of substernal chest pain.
ECG confirms heart failure and left ventricular dysfunction. On examination his chest is
clear and no lower limb edema. His blood pressure is well controlled and heart rate is
normal on the lower side. the mostappropriate management?
A. add digoxin B. Add bisoprolol C. No need to add any medications D. Add ARBs
....
🌹Patient with dyslipidemia was prescribed a medication, after 10 days presented
complaining of face flushing and redness. Which medications he was prescribed?
A. niacin B. Cholestyramine C. Statin
D. Fibrate
‫ﻧﺎ´ﺲ =وﺟﻬﻬﺎ اﺣﻤﺮ‬
...
🌹 Pt have lead 2,3 avf and avl st elevation ask about next to order ? A_ R side lead ecg to
rule out posterior MI
.....
🌹Clear hx of acute pericarditis and with ecg tx? ibuprofen
….
🌹Pt with paroxysms of palpitations and other symptoms, ecg normal in the clinic, the next
step :
A. Holter B. stress ecg
....
102-Patient with pervious viral infection then pleuritic chest pain ecg shows diffuse ST
elevation and PR depression what to expect on chest exam? Pericardial rub ✅ ‫ﻻن ﻫﺬە ﺣﺎﻟﺔ‬
pericarditis

ECG with tall T wave and potassium level 6.5 immediate action?
A-calcium gloconate✅ B- insulin C-dialysis D-sodium bicarbonate ((Hyperkalemia ===
treat by : ca ‫)) „ﺎﻟﺴﻴﻮم‬
‫ﺑﻮﺗﺎﺳﻴﻮم =ﺑﻮﺗﺎﺳﻴﻮم و„ﺎﻟﺴﻴﻮم اﺻﺪﻗﺎء‬
…..
Elderly 60 female, palpitations, ECG normal. What to do? A- Stress ecg✅ B- Echo
...... ...... ....
Pt Pt has pleuritic chest pain on left side, P/E reveals a pleuritic friction rub. What is the
next step?
A. 12 leads ECG B. CT chest C. CXR✅✅ D. ECHO, refer to cardio
N.B: Pluritc friction rub indicate pleurisity or pluritis Commonly in pneumonia , " pleuritic
friction rub" not cardiac.

ECG with tall T wave and potassium level 6.5 immediate action?
A-calcium gloconate✅ B- insulin C-dialysis D-sodium bicarbonate
#hyperkalemia ECG give = Ca gluconate.#
........
ECG shows inferior MI( Lead II + III + AVF ))

🍓Pt with chest pain , murmur, splinter hemorrhage “ Infective endocarditis??” what’s the
TTT:
A. ceftriaxone B. gentamicin C. ceftriaxone + vancomycin✅
.......
🌹 Something month old baby presented with fever, dyspnea, chest retraction O/E: there is
bilateral crepitations Apart from his post natal examination he found to have Pansystolic
murmur Xray: Cardiomegaly, bilateral lung infiltration What is the best next step ?
A. Surgical referral B. Diarutics ✅✅ C. Antibiotics then re-evaluation
.........
🍓 TB with chest pain for 2month. he has distended JVP, increases with inspiration. No
murmur
A. Constructive pericarditis✅ B. Cardiac tamponade
.....
🍒Best drug decrease mortality in HF = A. Digoxin B. Enalapril✅✅ C. Hydolizen

🍒Young male presented with fever had history of rheumatic heart disease, before this
admission he had a history of teeth extraction. On examination: murmur and splenomegaly
What is the Dx ?
A. Recurrent rheumatic fever B. Infective endocarditis ✅✅
….
🌹diffuse ST Elevation ,will find in exam: (pericarditis case)= A- murmur B- pericardial
rub C- canon a wave D- 4th heart sound
….
....
🌹 45 ys old lady last visit from 6 month was normal now C/o dyspnea 'LL oedema ascites
no murmur no fever distended jugular v with picture n curve ? A. pulmonary hypertension
B. tricuspid regurg C. restrictive pericarditis
Pulmonary HTN > core-pulmonale > signs of right-sided heart failure
🌹 Pt with signs and symptoms of dyspnea and pulmonary congestion, mentioned
the pulmonary artery pressure 35 mmhg, most likely cause:
A. constrictive pericarditis. B. pulmonary HTN (normal 8-20)

….
acute congestive heart failure management 1-oxygen 2-furosemide 3-ACEI 4-BB
....
🌹 Elderly with ejection systolic murmur, asymptomatic. His pressure gradient is 40 mmhg.
What’s the appropriate management?
A. balloon valvuloplasty B. Valve replacement
C. Evaluate by echo or exercise test (follow-up) ‫ {ﺎﻟﻌﻤﺮ ﻣﺎراح ﻳﺘﺤﻤﻞ اﻟﺠﺮاﺧﺎت‬²§‫ﻻن ﺧﻼص ﻛﺒ‬
‫ة‬²§‫اﻟ–ﺒ‬
....
🌹Elderly with ejection systolic murmur and asymptomatic. indicated surgical intervention?
A. patient’s symptoms B. Low pulse pressure C. Increase intensity of the murmur
...
🌹 Pt 16 years old male with faint/LOC after exercise, and ejection systolic murmur, echo
shows mildly dilated LV walls, most likely the cause of his symptoms/ or DX: A. AS B.
Hypertrophic/dilated cardiomyopathy
....
Just like the above case what is the right action to treat this murmur?
A-cathlap B-refer to cardiology C-valvotomy D-treat the the cause then reexaminations
✅✅
🌹Pt k/c of DM and HTN use ACEI and BB , CCB , Statin , then develop dry cough
for 3 month what drug stop = ACEI
...
🌹Atriall fibrillation which drug for sinus rhythm= Amedaron 🌷

2 weeks newborn with history of SOB and sweating in forehead what cardiac anomaly he
has?
a- PDA. b- VSD✅✅. c- TGA. d- ASD
Symptoms of PDA and VSD are similar, could be distinguished by physical examination but
the most common type of heart defect is ventricular septal defect (VSD) in USA.
🌹2 weeks newborn with history of SOB and sweating in forehead what cardiac anomaly he
has = VSD 🌷٠
..... ..... .....
A known hypertensive on different medications.He was recently diagnosed of Asthma.He
came in the Clinic for review of his drugs.Which of the following medication will likely be
stopped?
A.CCB. B.B blocker✅✅. C.Aspirin. D.Diuretics
🌹hypertensive recently diagnosed of Asthma = what medicantion will stopped = B
blocker🌷
* Beta blcoker Do bronchoconstriction*
.... .... ...
🌹pt has excacerbated by exercise what to give to maintain = inhalation steroid twise
with short acting when needed 🌹

Patient with old MI and HTN and asthma came due to acute onset since two
days, which made him stop his medications, his wife noticed he was feverish
and somnolent, physical exam showed diminished JVP, no basal crackles, No
abnormal heart sounds, ECG showed sinus tachycardia with a waves
indicating old MI ( as per the question) what is the most appropriate thing to do
next? A. 1- IV normal saline bolus B. 2- Atropine C. 3- dobutamine D. 4- IV beta blockers
…..
🌷 Wood worker ‫ﻋﺎﻣل ﺧﺷب‬has symptoms of asthma? serial test at work and home🌷

🍒Diagnosis of occupational asthma?
A-Skin prick test B-Specific lgE C-Peak flow rate at home and work ✅
..
🌹 Patients who take steroids at 12:00 - 3:00 - 6:00 complain that she cant
take it on this time for -some reason related to sleep- what is the management:
A. take at the same time, regardless of her sleep
B. take it after waking, 3 and 6 hours after that✅
🌹 Female taking steroids wants to change the medication so not to counteract with her
work: A. take same time B. no need to take it C. take it at wake up time, 3 hr then 6 hrs

🌹Elderly on med(forget name) taking at bedtime at the morning he feels dizzy:


A. switch to SSRI B. take 10mg / 3 times per day
‫~ﺔ ﻟﻬﺎ‬é‫اات ﻣﺘﻘﻄﻌﺔ •ﻘﻞ اﻻﻋﺮاض اﻟﺠﺎﻧ‬ÒÓ‫ ﻓ‬¤‫ ﻟﻤﺎ ﻧﺎﺧﺬ اﻟﺪواء ﻋ‬.
....
🍒Patient with lower limb edema, dyspnea, Decreased heart sound, raisedJVP?
- Cor-pulmonale✅🍒

‫ﻒ ﻧﻔﺲ‬w‫&ﺺ ﺻ‬%‫ﺎﻟﺮﺋﺔ وﺳ‚ﺐ ﻟﻠﻤ‬B ‫ﻄﻠﻊ |ﻞ اﻟﺪم ﻓ}^ا~ﻢ‬2 ‫ﻒ وﻣﺎﻗﺪر‬w‫ ﻓﺴﺎر اﻟﻘﻠﺐ ﺿﻌ‬، ‫ﻤﻦ‬2‫; اﻟﻘﻠﺐ اﻻ‬: < ‫&ﺾ ﺣﺼﻞ ﻟﻪ ﻓﺸﻞ‬%‫ﻠﻤﺔ ﻣﻌﻨﺎﻫﺎ ﻣ‬l‫ﻫﺬە اﻟ‬
‫ﻪ ﻓﺮﺟﻊ ﻟﻠﺠﺴﻢ‬w‫ﻒ وﺗﺮا~ﻢ اﻟﺪم ﻓ‬w‫ﻤﻦ ﺳﺎر ﺿﻌ‬2‫ وﻻن اﻟﻘﻠﺐ اﻻ‬، ‫•ﺪﻟﺖ ب |ﻠﻬﺎ ﺳﻮاﺋﻞ‬Œ‫ﺎﻧﺔ ﻫﻮاء ﻟ‹ﻦ اﺳ‬w‫ﻠﻮا اﻟﺮﺋﺔ ﻣﻔﺮوض ﺗﻜﻮن ﻣﻠ‬w‫ﺪ ﻻن ﺗﺨ‬2‫ﺷﺪ‬
‫ واﺻﺒﺢ اﻟﺠﺴﻢ |ﻠﻪ ﻣﻨﻔﺦ رﺟﻠﻪ |ﻠﻬﺎ‬edema ‫ﻄﻨﻪ‬B ‫ و‬ascitis ‫ و‬liver ‫ و‬spleen ‫ﻪ ﺗﻀﺨﻢ‬w‫ ﺳﺎر ﻓ‬.
Cor-pulmonale= abnormal enlargement of the right side of the heart as a result of disease of the lungs or the
pulmonary blood vessels.
….
# Ckd with hyperkalemia of 6.6
A. – hemodialysis B. - Iv phosphate C. – insulin D- Iv Ca gluconate
….
🌹Elderly patient presented with SOB, placed on 100% O2 but still not improving, failed NS
challenge, vitally hypotensive and no fever, CXR showing bilateral lung infiltrates, what is
the cause of her SOB?
A. PE
B. Cardiogenic cause of lung edema ‫ﺲ راح ﺗﺠﻤﻊ ﺳﻮاﺋﻞ و ﻋﻨﻈﻪ اﻧﺨﻔﺎض {ﺎﻟﻀﻐﻂ‬é‫ﻻن ﻗﺎل ﻓ}ﻪ {ﺎﻻ‬
C. Pulmonary pneumonia‫ﻣﺎﻋﻨﺪە ﺣﺮارة‬
D. Volume overload ‫ﺎﻟﻀﻐﻂ‬u ‫ﺎ ﻣﺎ•ﻌﻤﻞ اﻧﺨﻔﺎض‬y‫ﻏﺎﻟ‬
....

🌹 indicate adequate systemic perfusion?


A. Cardiac index B. Mixed venous oxygen saturation

🌹Mitral stenosis is critical when the valve area is reduced to = 1 cm
‫ﻂ =„ﻢ ﺻﻤﺎم ﻋﻨﺪﻧﺎ‬Ï‫ ر‬mitral = ‫واﺣﺪ‬
...
- Newborn has cyanotic spells XRAY shows egg on side, Ddx? TGA (Transposition of the
great arteries ).
🌹Newborn has cyanotic + XRAY shows egg on side = Transposition of the great arteries 🌹
..... ..... ....
🌹Lateral MI pic best intervention‫ ؟‬PCI🌹
🍓HOCM treatment? A. B blocker ✅ B. nifedipine
ƒ
‫ ر`ﻂ ﻣﺮض‬hocm ‫ﺴﺪاد ={ﻠﻮك‬ï‫ و ا‬²§‫ﻴ‬Û‫ اﻟﻘﻠﺐ ﺗﻀﺨﻢ ﻛﺒ‬mo ‫ ﺣﺎﺻﻞ‬îo ƒ ‫ “ﻌ‬block . = ‫ﺘﺎ {ﻠﻮك‬Û‫ﺑ‬
🍓Female want to start a sport , she had a brother died suddenly during doing a kind of
sports ( i forgot ) , you want to exclude which disease before starting :
dilated cardiomyopathy answer is HOCM✅✅✅
🌹During sport he became unconscious which CHD he may have? HOCM

🌹 how to exclude cardiac ischemia? coronary angiography


🍓cardiac tamponade definitive investigation = ECHO 🍓

Murmur
🌹Pt on pre sport screening had wide fixed splitting of S2 and was diagnosed with ASD.
What to do = Reassure and discharge
( Not surgical intervention = Bez he is stable not complain of cyanosis, SOB ..etc )🌹

-Mid systolic ejection murmur , diagnosis? Pulmonary stenosis + Aortic stenosis.
….
🍒 23 year old female patient, history of ASD when she was 3 Years, now he has
decrescendo diastolic murmur, 2/6, on the left sternum, what is your diagnosis? A. Mitral
stenosis B. Aortic regurge C. Tricuspid regurg
.....

🌹case of cardio have ejection systolic murmur what the diagnosis? A- MR B- AR C-
Tricuspid stenosis D- pulmonary stenosis
....
60 y//o male with valvular disease, radiate to carotid? Aortic stenosis ✅
‫ﺸﻮﻓﻮن‬â ‫ﺪا‬u‫ داﺋﻤﺎ وا‬: murmur radiate to carotid
‫ ﻃﻮل اﺧﺘﺎروا‬¤‫ ﻋ‬aortic stenosis .
¼ ¼
‫–ﺎن‬è( ‫ﻄﻬﺎ‬ÏÀ‫ ﻃ~ﺐ ﻧ‬aorta ‫–ﺎن‬è ‫–ﺎن ﺿﺨﻢ •ﻐﺬي „ﻞ اﻟﺠﺴﻢ وﻛﺬﻟﻚ‬è carotid ñ ¼ • ç ‫ﺿﺨﻢ •ﻐﺬي „ﻞ اﻟﻤﺦ ﻓﺎﻻﺛﻨ‬
ŒŽ • ‫ ﺣﺼﻞ ﺿﻴﻖ‬، ‫ﺎر‬y‫ ﻛ‬ñ
• ç ‫–~ﺎﻧﻴ‬è¼ aorta Œ• ‫ ﻟﺬﻟﻚ راح ´ﺴﻤﻊ‬carotid ‫ﺎر‬y‫ اﺳﺎﺳ~ﺔ وﻛ‬ñ
Ž
• ç ‫اﻳ‬è¼ ñ
• ç ‫) ﻻن ﻫﻤﺎ اﻻﺛﻨ‬
...... .. . . .
Child with Continuous murmur, both systolic and diastolic? PDA ✅ ✔
PDA = Patent duct arterios
ƒ
‫ ﻟﻤﺎ •ﻘﻮﻟﻚ‬Continuous murmur ‫ اﻻ‬mo ‫ ﻣﺎ‬PDA , ‫ﺸﻮﻓﻮن‬ò ‫ اول ﻣﺎ‬، ‫ ﻣﺴﺘﻮاﻧﺎ‬-‫ﺲ ﺧﻠﻴﻨﺎ ﻋ‬ñ ‫ﻓ}ﻪ اﺷ}ﺎء ﺛﺎﻧ}ﺔ‬
ƒ
Continuous murmur ‫ ﻃﻮل اﺧﺘﺎروا‬-‫ﺴﺘﻮل ود“ﺎﺳﺘﻮل ﻋ‬ã‫ اﻟﺴ‬mo ‫ ﻣﺎرﻣﺮ ﻣﺴﺘﻤﺮ‬îo ƒ ‫ “ﻌ‬PDA .
‫ ( = ر`ﻂ‬PDA ) = ‫ = {ﺪء‬contiunous = ‫ﻣﺴﺘﻤﺮ‬
Õ Õ
‫`ﺎت‬ö‫ﺴ÷ﺴﻠﻤﻮا ﻟﻠﺼﻌ‬ò ‫ﺴﺘﻤﺮوا )ﻓ}ﻪ وﻻ‬ò( ‫ء ﻧﺎﺟﺢ ﻻزم‬Ô o ‫ﻜﻮن‬w‫ﺪﺋﻮن )ﻓ}ﻪ و‬x‫ء (ﺗ‬Ô o ‫👌 اي‬
........
🌹Patient 40 yo, complaining of sudden loss of consciousness at rest and activity in the
gym. P/E shows left para sternal ejection systolic murmur that increase on standing and
bearing forward with no radiation. ECG shows st and t wave changes. What is the dx? A.
Hypertrophic cardiomyopathy B. aortic stenosis
‫ ﻟﻤﺎ •ﻘﻮﻟﻚ‬murmur ‫ ﻳ—–ﺪ ﻣﻊ اﻟﻮﻗﻮف‬standing = ‫ ﻧﺨﺘﺎر‬hypertrophic cardio ‫ﻻن ﻣﻊ اﻟﻮﻗﻮف •ﻘﻞ ة‬venus
return ‫ •ﻘﻞ‬ÐŽ‫ﺎﻟﺘﺎ‬Ï‫ و‬left ventricull fillin ‫ﺎﻟﺪم ﻳ—–ﺪ اﻟﺤﻤﻞ ﻋﻠ~ﻪ وﺻﻮت‬u †´‫ اﻻ‬ñ • ç ‫ﻄ‬y‫ وﻟﻤﺎ •ﻘﻞ اﻣﺘﻼء اﻟ‬murmur
‫ﺎل‬u ‫ دم „ﺎﻓ~ﺔ‬ŒŽ • ‫ راح •ﻜﻮن ﺟﺪا واﺿﺢ ﻻن ﻣﺎ‬ventrical ‫ •ﺤﺎول ﻳﺰود „ﻤ~ﺔ اﻟﺪم‬ýÓ‫ﺾ ﺣ‬y‫ﺬل ﻣﺠﻬﻮد و–—–ﺪ اﻟﻨ‬y‫ﻓﺮاح ﻳ‬
‫ﻟﺬﻟﻚ ﺻﻮت اﻟﻤﺎرﻣﺮ ﻫ~ﻜﻮن واﺿﺢ‬.
....
🌹 ECG shows (LVH) what clinical findings you can see ?
A. murmur B. a wave C. fourth heart sound D. friction rub
‫ﻂ‬Ï‫ =ر‬LVH = 4= ‫ = ﺛﻼث ﺣﺮوف‬FORTH
....
Normal asymptomatic pt came for check up .. stage 2 mid diastolic murmur:
A- Echo B- Antistrept O titer C- Other choices were investigation also
• ¼
‫ ﺻﻤﺎم اﻟﻘﻠﺐ‬ŒŽ ‫ﺸﻮﻓﻮا ﻣﺸ•ﻠﺔ‬â ‫ اي ﻣﺮة‬valve ‫»ء ﻫﻮ ا•ﻜﻮوووا‬ Ž ‫ اﺧﺘﺎروا اول اول‬ECHO .
🌹pt with stage 2 mid diastolic murmur for check up = Echo🌹
. . . .. . . . . .
🍒male presented with macro-orchidism, long face, wide ears, joint laxity, mid diastolic
murmur best heard at apex area. What is the Dx? Fragile X syndrome 🍒
.......
🍒Aourtic stenosis = systolic murmur radiated to carotid🍒

🌹HCOM: murmur increases with standing or valsalva + causes syncope, most common
cause of sudden death in young athletic

🌹 Mitral regurgitation (Pansystolic murmur) + splinter hemorrhage, the most likely
organism: strept viridins
‫ﻂ‬Ï‫ = ر‬viri = ‫ ﻓ~ﺪﻳﻮا‬vidio ‫ﺸﻮف ﻓﻴﻬﺎ‬D mitral = ‫¸ ودم‬Ó‫ ﻣ~ﺖ ﻣﻮ‬hemohhrage
............
🍓Murmur increase with finger grasp =
Mitral regurge + Aortic regurge ✅
ƒ
‫ﺪ ﺻﻮت‬wÙ‫ اي او`ﺠ}ﻜﺖ او اﻻﺻﺒﻊ اﻟﻤﻬﻢ ﺿﻤﺔ اﻟ}ﺪ {ﻘﻮة ﻳ‬-‫ﺾ “ﺪي {ﻘﻮة ﻋ‬x‫ ﻟﻤﺎ اﻗ‬îo ƒ ‫ “ﻌ‬murmur ‫ اﻟﻘﻠﺐ‬mo
‫ وﺟﻮد =ﻣﻊ‬-‫ﺬا ﻋﻼﻣﺔ ﻋ‬¹‫ و‬regurge ‫ ﺳﻮاء‬mitral or aortic
Grasp = regurge . = increase .
...............
🍓Murmur decreased with finger grasp :
Aortic stenosis. + HOCM AND MVP.
‫ﺾ •ﺪي ﺻﻮت ال‬y‫ ﻫﻨﺎ اﻟﻌﻜﺲ ﻟﻤﺎ اﻗ‬murum ‫ •ﻘﻞ وﻋﻜﺲ‬regure ‫ ﻫﻮ‬stenosis .
ƒ
‫ اﻟﺼﻤﺎم‬mo ) ‫ﻘﻮة ﺻﻮت اﻟﻤﺮﻣﺮ (“ﻘﻞ )ﻣﻊ وﺟﻮد (ﺿﻴﻖ‬u ‫ﺾ •ﺪي‬y‫ ﻟﻤﺎ اﻗ‬ýŽ • ‫ اﻟﻌﻼﻗﺔ ﻋﻜﺴ~ﺔ •ﻌ‬.
.........
🍓pt with mid diastolic rumbling murmur , what’s the cause: A. mitral stenosis✅ B. aortic
stenosis C. tricuspid stenosis
‫ اذا ﺷﻮﻓﺘﻮا‬diastolic ‫ ﻣﻌﻨﺎﻫﺎ‬stenosis ‫ واذا ﺷﻮﻓﺘﻮا‬systolic ‫ ﻣﻌﻨﺎﻫﺎ‬regurge . ŒŽ • ‫ ﺣﺴﺐ اﻟﻤ•ﺎن ﻟﻮ ﻗﺎل‬¤‫ﺛﻢ ﻋ‬
apex ‫ ﻣﻌﻨﺎﻫﺎ‬mitral ‫ ﻟﻮ ﻗﺎل‬righ scond intercostal ‫ ﻣﻌﻨﺎﻫﺎ‬aorta .
....
🍓pt with murmur radiating to the carotid artery, what’s the cause:
A. mitral stenosis B. aortic stenosis✅✅ C. tricuspid stenosis
‫ اذا ﺷﻮﻓﺘﻮا „ﻠﻤﺔ‬carotid ‫ „ﻠﻤﺔ‬Òç‫ ﻏ‬ŒŽ • ‫ ﻻ ﺗﻔﻜﺮوا‬aorta ‫ﻌﺾ‬u ‫ﺎر ﻣﻊ‬y‫ ﻛ‬ñ
• ç ‫–ﺎﻧ‬è¼ .
......
🍓Case of murmur increase with supine postion? Innoncent murmur.

🍒elderly 2 murmur diastolic rumble murmur and diastolic decrescendo murmur what's
most likely dx?
aortic regurge✅
Decrescendo diastolic -> AR mid diastolic -> MS
.....
🍒2 day (Im not sure about day may be earlier) Post Mi complication with systolic murmur
that radiates to the axillary:
A. Papillary rupture ✅ B. Myocarditis

🌹20s age with Hx of TOF repair 12 years ago, presented with diastolic decrescendo
murmur best heard in left parasternal area and increased with inspiration? A-Tricuspid
regurgitation. B-pulmonary regurgitation✅ C-Mitral stenosis

🌹+Post MI come with harsh systolic murmur what's dx= Papillary rupture
🌻rupture= harsh
....
🌹Child noticed having cyanosis with feeding+ Holosystolic murmur was noted:
TOF ( Not VSD ).
‫ = ر`ﻂ‬tof ‫ﻮﻟﻮا‬¹= ‫ﺎت‬¹‫ﺸﻮ‬ò ‫ﻠﻪ ﻓ}ﻪ‬á ‫ “ﻜﻮن اﻟﻘﻠﺐ‬holo = ‫ﻠﻪ‬á .
.....
-Mid diastolic murmur heard on the right sternal border,which valve diseased? Aortic
valve regurgitation (diastolic murmur )
....
🌹 Pt with femoral pistol shots , what is DX:= Aortic Regurgitation
û‫–ﺎن اﻻور‬è¼= ‫ﻂ‬Ï‫ ﻓ~ﻤﻮرال ارﺗﺮي ر‬¤‫ﺴﻤﻌﻪ ﻟﻤﺎ ﻧﻀﻊ اﻟﺴﻤﺎﻋﺔ ﻋ‬D ‫ ﺻﻮت‬ýŽ • ‫ﺴﺘﺎل •ﻌ‬Ec‫ „ﻠﻤﺔ ﺑ‬shot ‫اﻟﺪم ﻟﻠﺠﺴﻢ‬
...
Diastolic murmur with pistol shot sound in the femoral artery?
A-aortic stenosis B-Mitral stenosis C-aortic regerg ✅✅ D-mitral regerg
• ç ‫ ﻓﺎﻻﺛﻨ‬Òç‫ﺎﻟﺮﺟﻞ ﻣﺮة ﻣﺮة ﻛﺒ‬u Ð‫ =واﻟ¼†–ﺎن ا‬û‫اﻻور‬
• ç ‫ﻄ‬y‫ ﻣﺮﺗ‬ñ
‫ﺎﻟﻤﻨﻄﻖ اﻟﺪم ﻟﻤﺎ ﻳﺮوح‬u ‫ ﺛﺎﻧ~ﺎ د•ﺎﺳﺘﻮﻟ~ﻚ‬، ‫ﻌﺾ‬y‫ ﺑ‬ñ Ž
‫ح ؟ ﻣﻦ اﺳﻤﻬﺎ‬c‫ ﻣﻔﺘ‬û‫ ﻣﺶ ﻻزم •ﻜﻮن اﻻور‬û‫ ﻟﻼور‬.
.....

Name of murmur that occurs when is severely sick and previously normal CVS exam: A-
Innocent murmur ✅

heard , the child CVS and chest exams were previously normal (prior visits) what is the
management of murmur?
A-Urgent echo B-Reexamine after these symptoms subsides ✅ C-Refer to cardio
pediatrician
‫ﺴﻤﻊ ﺻﻮت ﻣﺎرﻣﺎر ﻣﻦ اﻟﻘﻠﺐ‬ï ú
o }‫ ﻟﻤﺎ اﻟﻄﻔﻞ “ﻜﻮن ﻋﻨﺪە ﺣﺮارة ﻃﺒ‬:
Normal” is another word to describe these murmurs. A normal murmur is just a noise
caused by blood flowing through a normal heart. ... For example, when the body's
temperature goes up, the blood flows faster. Doctors often hear heart murmurs when they
check children who have a fever.
.....
Long scenario about child have systolic murmur and laps with obvious anemia what is
that murmur?
A-innocen✅ B-Mitral regerg C-aortic stenosis D-pulmonary stenosis
... ........ ........ ......
Name of murmur that occurs when is severely sick and previously normal CVS exam: A-
Innocent murmur ✅
….

Early systolic murmur = pulomonary stenosis


‫ﻂ‬Ï‫ﺎض اﻟﻘﻠﺐ =ر‬y‫ اﻧﻘ‬ŒŽ • ‫ ﻃﻮل‬¤‫ﺿﻴﻖ اﻟﺮﺋﺔ =•ﻌﻤﻞ ﺿﻴﻖ ﻋ‬
......
mid diastolic murmur+ increases with standing, age 14 years? A- VSD B- AS C- Mitral
valve prolapse or HOCM ✅
Mid = mitral , prolaps = ‫زﺣﻠﻘﺔ ﻟﻤﺎ اﻟﺸﺨﺺ ﻳﻮﻗﻒ‬
.... ...... ......
pediatric with spells of cyanosis when crying, examination showd ejection murmur, what is
the dx:
A- aortic stenosis B- Tetralogy of fallot) ✅ C- TGA
**ejection murmur == Tetralogy of fallot**
..... ..... ....
- patient has history of MI suddenly he became breathlessness and harsh systolic murmur
heard what the cause:
A.PE. B.rupture aortic cusp C.tricuspid regurge. D. rupture of papillary muscle >>MR
🌹history of MI suddenly he became breathlessness and harsh systolic murmur heard =
rupture of papillary muscle 🌹
... .... ...
🌹case with poorly controlled HTN , cardiac complication ? hypertrophic left ventricle (
distolic hf )🌹
........
🌹Rheumatic heart disease? mitral stenosis 🌹
......
# Mid to late diastolic murmur at apex typical came with= Mitral stenosis
…….
🍓 case with flat T wave what you will find in the pt the : hyporkalemia
🍓 case with elevated T wave and a lot of hx then what to give the : ( hyperkalemia) = CA
gluconate

● Picture of ventricular tachycardia?

Picture of supra-ventricular tachycardia?

.....

Asthma
Bronchial asthma*
FEV1 low FVC low FEV1/FVC ratio low DLCO normal (or high) TLC normal (or high)
🌹Patient presenting with symptoms of obstructive lung disease. PFTs show decreased vital
capacity and decreased FEV1/FVC ratio.what is associated with this condition?
A. Increased lung compliance✅ B. Decreased lung compliance C. Weak respiratory muscles

🌹Female with 3 wks hs of productive cough With decrease both FEV and ratio
What other finding? A. Increase lung compliance B. Decrease lung compliance
🌹 Asthma medications in pregnancy, what to take and what to stop? Continue the same
‫ﺴﺘﺨﺪﻣﻬﺎ اﻟﺤﺎﻣﻞ آﻣﻦ ﻋﻠﻴﻬﺎ‬â ‫ادو–ﺔ اﻻزﻣﺔ ﻋﺎدي‬
🍒 Elderly patient with hx of excessive smoking presented with 2 weeks history of dry
cough, nocturnal Spirometry showed decreased ratio FVC/FEF and decreased vital capacity.
What else would be associated?
A. increased Lung Capacity✅. B. Decreased Lung capacity C. Increase airway resistance
🍒 asthmatic patient, last OPD peak flow was 600, when to say it is acute sever asthma:
A- peak flow of less 250 ✅ B- O2 saturation 94%

🍒What indicates severe asthma? A. RR >25✅ B. HR >100. C. PEFR <300. D. O2sat <95%
🍒 patient’s PEF was 600, What is the indicator?
A. RR >20. B. HR >100. C. PEF <200✅. D. O2 sat <95%
🍒 Asthma salbutamol before exercise Not control ?add ICS🍒

🌹Patient using many drugs but he came with decreased K = salbutamol . 🌹

🌷Severe asthma (daily use of SABA and more than 2-3times night symptoms) on LABA and
ICS what to add? theophylline🌹
….
🌹Asthma management: The best initial test in an acute exacerbation: peak expiratory
flow🌹
.....
🌹The most accurate diagnostic test is pulmonary function testing ( Spirometry)🌹will
show a decrease in the ratio of forced expiratory volume in 1 second (FEV1) to forced vital
capacity (FVC).

🌹Asthma exacerbation after pneumonia tx? A. Ventolin + Antibiotic ✅
B. Ventolin + systemic steroid C. Systemic steroid + hydration D. Systemic
steroid + AB
‫ ﻧﻠﺘﺠﺎ ل اﺳﺘﺨﺪام‬systimic steroid 5 ¤‫ ﺣﺎﻟﺔ وﺣﺪة اذا اﺳﺘﻌﻤﻠﻨﺎ ﺟﻤﻴ|ﻴﻊ ادو–ﺔ اﻻزﻣﺔ وﻣﺸ|ﻨﺎ ﻋ‬ŒŽ • steps ‫–ﺐ‬ÒÓ‫ﺎﻟ‬u

‫ اي‬ŒŽ ‫وﻣﻊ ﻫﺬا „ﻠﻪ ﻣﺎ‬
N.B: Empirical abx not recommended in acute asthma, but it’s used in suspected bacterial
sinusitis or pneumonia.
.............
🌹Asthma severe cant complete one sentence management? A.Cpap ✅ 🌹 B.Mechanical
ventilation
‫ﺎب ﻣﻦ اﻟﺸﻬ~ﺔ ﻣﺎ ﻧﻘﺪر ﻧﺘ•ﻠﻢ وﻧﻘﻮل ﺟﻤﻠﺔ وﺣﺪة „ﺎﻣﻠﺔ‬y‫ ﻧﺎ™ﻞ ﻛ‬¢ Ž £ ~‫ﺎب =ﻟﻤﺎ ﻧ‬x‫ﻂ =ﻛ‬Ï‫ ر‬.
.........
Best treatment for obstructive sleep apnea? CPAP

🌹 sleep apnea what is the treatment ? A. home o2 B. nasal cpap
..
🌹 He has HF and came with asthma attack then in ER they manag him with salbutamol and
he has decreased K what is the cause? Salbutamol 🌹 ‫ﻂ‬Ï‫ = ر‬salb = ÐŽ‫ = ﺳﻠﺐ ﺟﻤﻴﻊ اﻣﻮا‬K = ‫„ﻠﺐ‬
، ‫اق‬è.
............
🌹Copd patient not responding to Salbutamol what’s the next drug? Ipratropium
‫ = ر`ﻂ‬Ipra ‫ة =ﻣﺮض‬²ÿ‫ اﺑﺮا =ﻋ‬COPD ¦ ƒ § ‫ة )ﻳﻮﻗﻔﻮا ﺗﺪﺧ‬²ÿ‫¦ =ﻟﻌﻠﻪ (ﻋ‬
ƒ § ‫ﺴ!ﺐ اﻟﺘﺪﺧ‬ñ ‫ ﻣﺎ وﺟﺪ اﻻ‬.
N.B: Salbutamol + ICS in asthma Salbutamol + Ipratropium in COPD
.......
🌹A child with worsening asthma father won’t quit smoking what should u do?
A. consult ethics B. refer to other doctor C. report it. ✅✅
.......
Male smoker since 3-4 years has dyspnea.Fev1/fvc is less than 0.7,Dlco is
low,FEV1/FVC mildly improved after administration of beta blocker
What is the likely dx: A. Chronic bronchitis. B. Asthma. C. Emphysema
….
🌹Child was having severe asthma exacerbation father won’t quit smoking =
A.treat the child and ignore father B.call child protection Answer is A then B
...........
🍓 Case about moderat asthma on saba and ics what add : Laba.( long acting ...
.........
🍓Case of copd it is sever and full secretion : invasive MV .
.........
🌹 smoker with recurrent respiratory infection and productive cough ?
chronic bronchitis ‫ﺲ ﻛﺤﺔ ﺟﺎﻓﺔ‬ã‫ﻻن ﻗﺎﻟﻚ ﻛﺤﺔ ﻣﻊ {ﻠﻐﻢ وﻟ‬

🍓pregnant women with asthma what tocolytics that C/I for her? A. Oxytocin B.
Misoprostol✅
‫ﺎ‬¹‫ع “ﺎﺧﺬ‬ö‫ ﻓﻤﻤﻨ‬، ‫وا “ﻌﻤﻞ ازﻣﺔ‬²ƒ § ‫وا =ازﻣﺔ =ﻣﻦ اﺳﻤﻬﺎ ﻣ‬²ƒ § ‫ ﻣ‬.
.........
🌹 Young female came to the clinic complaining of episodes of SOB, wheeze and cough for
the last 6 months not related to certain provoker. She says that these episodes comes
suddenly and resolve suddenly. She used ventolin but improved slightly. Upon physical
exam she has equal air entry with no wheezing. She is not allergic to anything and no one in
her family has the same problem.All test were normal and spirometry is normal. What is
the dx ? A.foreign body B.vocal cord dysfunction C.asthma
🍒Patient came with Symptoms of asthma but( little improve with bronchodilator) dx:
Vocal cord dysfunction
🌹Female 20-something y/o with dyspnea intermittently resolves suddenly. Minimal
improvement with b agonist. Dx:
A. asthma B. Vocal cord paralysis
🌹. Acute sever asthma criteria
B. RR 20 ( more than 25 to say sever). C. HR 100 ( more than 110 to say sever)
D. PFR 200 (should to say sever belew 100). E. Inability to complete one sentence in one
breath ✅
.......
🌹 Sever asthma exacerbation ( in distress )how to manage :
A. theophylline B. Magnesium sulphate ✅
C. Intubation = ( we use intupantion when O2 less than 50 and co2 more than 50 ) D. Non
invasive MV
Magnesium sulfate for treating exacerbations of acute asthma in the emergency
department. In an asthma attack, the airways (passages to the lungs) narrow from muscle
spasms and swelling .
......
🌹case of acute asthma exacerbation what study is most accurate to monitor
improvement? A. ABGs B. PFTs

🌹 pt known case of asthma in-compliant on his medication came with sx of status
asthmaticus , his last PEF was 600 , whats of the following indication of severity “in this pt”
: PEF 250

- in case of acute asthma exacerbation what study is most accurate to monitor
improvement=Pulmonary function tests
. .....
🌹How treat moderate asthma : A. laba + ics B. ics + saba C. ipratropium
...
🌹Patient known case of asthma on albuterol , diagnosed as moderate persistent asthma
what you will add ? A. ICS. B. ICS and LABA as needed. C. Theophylline D. Montelukast
....
Q about asthma exacerbation: ventoline and steroid ✅ ✔
During an asthma attack, also called an asthma exacerbation give both short acting
(ventoline or albuterol) and steroid
..... .... ....
Q about Exercise induced asthma, management? Salbutamol pre exercise ✅ ✔
......

Q about asthma increased symptoms in the last month? add ICS ( Inhaled corticosteroids )
✅✔
🌹Asthmatic patient on SABA + steroid, free of symptoms for 1 year, next?
A. continue same management B. stop both medication
C. decrease steroid to half dose + SABA✅ D. add montelukast

🌹Pt pregnant with bronchial asthma have SOB and whezzes and some times have night
attacks of asthma : SABA AND ICS ✅ ✔.

243- A 55yr old chronic smoker with chronic non productive cough presented with stable
vitals. X ray done is normal.whats the likely diagnosis?
A.COPD✅✅ (chronic bronchitis) B.Idiopathic Pulmonary Fibrosis. C.Pulmonary fibrosis
🌷chronic smoker + chronic non productive cough + x.ray normal = COPD (chronic
bronchitis) 🌹
.... ....
A child presented with sinusitis and recurrent lower respiratory tract infections.Mcs of
bronchial aspirate had pseudomonas aeruginosa.Whats the diagnosis?
A.Cystic Fibrosis✅✅. B.Primary ciliary dyskinesia. C.Kartagener's syndrome
🌷sinusitis and recurrent lower respiratory tract infections + bronchial aspirate had
pseudomonas aeruginosa =Cystic Fibrosis
..... ......
🌹Pt known to have emphysema, undergo some GIT operation (lap choly i think),
then pt is complaining of progressive dyspnea. ABG: high CO2 and high O2
Your management: A. intubation and hyperventilation B. give high oxygen
….
🌹FEV/FVC < 70 and low DLCO, what is the diagnosis?
A-Bronchiectasis. B-Emphysema✅✅. C-COPD
🌹Bronchial asthma =
FEV1 low and FVC low FEV1/FVC ratio low DLCO normal or high and TLC normal or high🌹
🌹COPD FEV1 low FVC low FEV1/FVC ratio low (DLCO low in emphysema, normal or high in
chronic bronchitis ) TLC normal (or high) in both🌹
‫ ﺣﻔﻈﻬﺎ‬ŒŽ • G ‫اﺳﺎﻋﺪ„ﻢ ان ﺷﺎء‬
ƒ ƒ n
‫اﻻزﻣﺔ و‬: ‫ اوﻻ‬COPD ‫ﺶ؟‬ž‫ ا‬mo ¦ § ‫ﻛ‬²‫ﻠﻬﻢ ﻣﺸ‬á
Asthma and COPD :
💎FEV1 + FVC FEV1/FVC ratio = LOW
💎 TLC = normal (or high) in both
• •
ŒŽ ‫ﺴﺪاد‬D‫ ﻻن „ﻠﻬﻢ ﻋﻨﺪﻫﻢ ا‬þŽ~‫ ﻃﺒ‬، ‫ ؟‬ÐŽ‫ اول ﺛﺎﻧ~ﺔ ﻫﻞ ﻗﻠ~ﻠﺔ وﻻ ﻋﺎ‬ŒŽ ‫ ﺗﺪﺧﻞ اﻟﺮﺋﺔ‬ÐŽ‫ﺎﺧﺘﺼﺎر „ﻤ~ﺔ اﻟﻬﻮاء ا‬u ‫ﻫﺬا ﻣﻌﻨﺎە‬
‫ ﻟﺬﻟﻚ ﻗﻠﻨﺎ‬، ‫ داﺧﻞ راح •ﻘﻞ‬ÐŽ‫ اﻟﺸﻌﺐ ﻓﺎﻟﻬﻮاء ا‬Fev = low . ‫اذا‬
FEV1 + FVC FEV1/FVC ratio = LOW
‫ و „ﻠﻤﺔ‬TLC ‫ ﻣﻌﻨﺎﻫﺎ‬totall lung capicity ‫ داﺧﻞ اﻟﺮﺋﺔ او ﻗﺪ ا´ﺶ اﻟﺮﺋﺔ ﻓﻴﻬﺎ ﻫﻮاء وﺣﺎﺻﻞ ﻟﻬﺎ‬ÐŽ‫ „ﻤ~ﺔ اﻟﻬﻮاء ا‬ýŽ • ‫•ﻌ‬
inflation ‫ اﻻزﻣﺔ وال‬ŒŽ • ، ‫ ﻣﻠ~ﺎﻧﺔ ﻫﻮاء‬ýŽ • ‫ •ﻌ‬copd ‫ﺐ‬Þ‫ﺴ‬7) ‫ﺗ—–ﺪ ﺣﺠﻢ اﻟﺮﺋﺔ ﻟ~ﻪ ؟؟ ﻻن ﻣﻠ~ﺎﻧﺔ ﻫﻮاء (ﻣﺤﺒﻮس‬
‫اذا‬. ‫ ﻓﺎﻟﺮﺋﺔ ﺣﺠﻤﻬﺎ زاﺋﺪ‬، ‫ ﻣﻮ ﻋﺎرف •ﻄﻠﻊ ﺑﺮة‬Òç‫ﺴﺪاد اﻟﻬﻮاء ﻛﺜ‬D‫ اﻻ‬:
TLC = normal (or high) in both
ñ• ç ‫ ف اﺗﻔﻘﻨﺎ ان اﻟﻤﺮﺿ‬copd and asthma ‫ •ﻜﻮن ﻋﻨﺪﻫﻢ‬ñ • ç ‫ اﻻﺛﻨ‬FVC ‫ ﻗﻠ~ﻠﺔ و‬TCL þ~‫ ﻋﺎﻟ~ﺔ او ﻃﺒ‬.
Ž
...
ŒŽ • ‫( ﻃ~ﺐ اﻻﺧﺘﻼف و–ﻦ‬DLCO) ‫ﻠﻤﺔ اﺧﺘﺼﺎرل‬6‫ﻫﺬە اﻟ‬:
(Diffusing capacity for carbon monoxide)
its mean (( is the extent to which oxygen passes from the air sacs of the lungs into the
blood. )).
ý•‫ ف ﻣﻌ‬DLCO : ‫–ﺼﻼت اﻟﺮﺋﺔ‬c‫ ﺣ‬ŒŽ • ‫ ﻣﻮﺟﻮدة‬ÐŽ‫ ا‬ñ • ç ‫ „ﻤ~ﺔ اﻻوﻛﺴﺠ‬4‫ ﻣﺎ‬ý•‫(•ﻌ‬Alveoli) ‫ ودﺧﻠﺖ ﻟﻠﺪم ؟‬،
Ž Ž
‫–ﺼﻼت‬c‫ ﺗﻄﻠﻊ ﻣﻦ ﺣ‬ÐŽ‫ ا‬ñ • ç ‫ „ﻤ~ﺔ اﻻوﻛﺴﺠ‬ýÓ‫( ﻃ~ﺐ ﻣ‬alveoli) ‫ﻟﻠﺪم ﺗﻘﻞ ؟؟‬
‫ ﻟﻤﺎ ﺗﺨﺮب‬، ‫–ﺼﻼت ﻫﺬە‬c‫ اﻟﺤ‬ŒŽ • ) Òç‫ ﻟﻤﺎ •ﻜﻮن ﻓ~ﻪ ﻣﺮض او (ﺗﺪﻣ‬alveoli ‫ •ﻘﻞ‬ÐŽ‫ﺎﻟﺘﺎ‬y‫ ﻟﻠﺪم ﻓ‬ñ • ç ‫ ﻣﺎ ﻳﻮﺻﻞ اوﻛﺴﺠ‬DLCO
، ‫ ﻋﻨﺪﻫﻢ ﻫﺬا‬ŒŽ Ó ‫ ﻧﻼ‬ÐŽ‫ﺎﻟﺘﺎ‬Ï‫ و–ﺨﺮب اﺑﻮ ﺷ•ﻠﻬﺎ و‬Òç‫–ﺼﻼت ﻫﺬە ﺗﺪﻣ‬c‫ ﻃ~ﺐ ﻓ~ﻪ ﻣﺮض واﺣﺪ •ﺪﻣﺮ اﻟﺤ‬DLCO ‫ﻣﻨﺨﻔﺾ‬
‫ ﻣﺎﻫﻮ ﻫﺬا اﻟﻤﺮض ؟ اﺳﻤﻪ‬:
Emphysema
‫ﻮ ﻛﺬا ﻣﺮض ﻣﻨﻬﺎ‬¹ ، ‫ﺲ ﻣﺮض واﺣﺪ ﻓﻘﻂ‬ã‫ﻮ ﻟ‬¹ ‫ﻌﺎ‬x‫ ﻃ‬fibrosis ‫ﺎر ﺣﻘﻨﺎ‬x‫ ﻗﺪ اﻟﺴﺆوال واﻻﺧﺘ‬-‫ﺲ ﺧﻠﻴﻨﺎ ﻋ‬ñ .
‫ ﻧﻌ}ﺪ ﻣﺮة ﺛﺎﻧ}ﺔ‬: DLCO ‫•ﻜﻮن ﻗﻠ~ﻞ ﻣﻊ اي ﻣﺮض ؟‬......
Emphysema
• •
‫–ﺼﻼت واﻟﺪم‬c‫ اﻟﺤ‬ñç‫ ﺑ‬ñç‫ﺎدل ﻟﻼوﻛﺴﺠ‬y‫ ﻟﻦ •ﺤﺼﻞ ﻋﻤﻠ~ﺔ اﻟﺘ‬ÐŽ‫ﺎﻟﺘﺎ‬Ï‫–ﺼﻼت و‬c‫ﻟ~ﻪ ؟ ﻻن ﻫﺬا اﻟﻤﺮض دﻣﺮ اﻟﺤ‬
‫ ﻃ}ﺐ‬DLCO ‫–ﺔ ﻣﻊ اي ﻣﺮض؟‬c‫ او ﻣﺮﺗﻔﻊ ﺷ‬ú o }‫•ﻜﻮن ﻃﺒ‬....
Asthma and chronin bronchlitis
‫ اﻟﺸﻌﺐ اﻟﻬﻮاﺋ~ﺔ‬¤‫ﻫﺎ •ﻜﻮن ﻋ‬Òç‫ ؟ ﻻن ﻣﻦ اﺳﻤﻬﺎ اﻻزﻣﺔ واﻟﺘﻬﺎب اﻟﺸﻌﺐ ﺗﺄﺛ‬þŽ~‫ ﻟﻴ|~ﻪ •ﻜﻮن ﻃﺒ‬Bronchi ¤‫وﻟ¨ﺲ ﻋ‬
‫–ﺼﻼت‬c‫( اﻟﺤ‬Alveoli) ...

‫ع ﻣﺮة واﺳﻊ‬c‫ﺎﺧﺘﺼﺎااار واﻻ اﻟﻤﻮﺿ‬u ‫ ﻫﺬا‬.


‫ﺲ اذﻛﺮ„ﻢ ﻓﻴﻬﺎ‬7 ‫ﻌﺪ اذﻧ•ﻢ ﻃﻮﻟﻨﺎ‬u ‫ ﻓ~ﻪ ﻣﻌﻠﻮﻣﺔ‬.
‫ اﺳﻤﻪ‬ÐŽ‫ ( ﻣﺮض ا‬COPD ) ‫ﻫﻮ اﺧﺘﺼﺎر ل‬
Chronic obstructive pulmonary disease
‫ ))ا´ﺶ ﻫﻤﺎ؟؟‬ñ • ç ‫ﻫﺬا اﻟﻤﺼﻄﻠﺢ ´ﺸﻤﻞ ((ﻣﺮﺿ‬
1_ Emphysema 2_ chronic bronchlitis
‫»ء اﺳﻤﻪ دﺧﺎن‬ ¼ • •
Ž ŒŽ ‫ )ﻟﻮ ﻣﺎ‬ñç‫ ﻟﻠﻤﺮض ﻫﻮ (اﻟﺘﺪﺧﻴﻴ‬/ Ž ¨‫ﺐ اﻻول واﻟﺮﺋ‬Þ‫ اﻟﺴ‬smoker ‫ ﻟﻤﺎ „ﺎن ﻟﻬﺬا اﻟﻤﺮض وﺟﻮدا‬.
‫ وﻗﺪرە‬G ‫ﻌﺪ ﻗﻀﺎء‬u. ‫ وﻫﺬا ﺣﻘ~ﻘﺔ‬.
‫ﺲ ﺣﺒ~ﺖ اذﻛﺮ„ﻢ ﻓﻴﻬﺎ‬7
. . . . . . .. . . .
Patient with obstructive about Pulmonary function testing , what is true?
A-Increased lung capacity ✅
B-Decreased lung capacity
C-Airway resistance
‫ ﻳ—–ﺪ ﻣﻦ ﺣﺠﻢ اﻟﺮﺋﺔ‬ÐŽ‫ﺎﻟﺘﺎ‬y‫ﺲ داﺧﻞ اﻟﺮﺋﺔ وﻣﺎ•ﻄﻠﻊ ﺑﺮة ﻓ‬Þ‫ﺎﻟﺮﺋﺔ اﻟﻬﻮاء ﻳﻨﺤ‬u ‫ﺴﺪاد‬D‫ﺣﻨﺎ ﻟﻤﺎ •ﺤﺼﻞ ا‬è¼ ‫ ﻣﺜﻞ ﻣﺎ‬.
.. . .. . .
🌹Bronchial asthma =
FEV1 low and FVC low FEV1/FVC ratio low DLCO normal or high and TLC normal or high🌹
🌹COPD FEV1 low FVC low FEV1/FVC ratio low
= DLCO low in emphysema,
=DLCO normal or high in chronic bronchitis = TLC normal (or high) in both🌹
‫–ﻞ‬c‫ﻂ =„ﻠﻤﺔ اﻟﻄ‬Ï‫ ر‬chronic bronchitis ‫ ﻣﻌﻨﺎﻫﺎ ﻣﺮﺗﻔﻌﺔ‬DLCO ، ‫–ﻠﺔ =•ﻜﻮن‬c‫ ﻓ•ﻠﻤﺔ ﻃ‬DLCO ‫ز–ﺎدة‬.
.....
Pt known case of COPD or asthma non compliant to medications with recurrent
admissions:
A. Keep in hospital since he is non compliant B. Tell his next kin to give the medications
….
Patient have anxiety for 4 years and have asthma she noticed when she use salbutamol
symptoms get worse, give: A. BB. B. sertraline. C. burbion
….
#● Management of moderate asthma with night symptoms 6/ month
A. LDICS. B. LDICS+ LABA
……
# A case of poorly controlled asthma ? Check the use of inhalers
………
-Patient with social anxiety and asthmatic, how to treat?
A-Propanalol. B-Bupropion c _ psychotherapy CBT if not mentioned SSRI.✅
... ..... ...
🌹COPD patient on salmeterol and albuterol, still not controlled as he needs to
use albuterol more often now and he had multiple exacerbations in the last 6 months, what
to give him for maintenance?
A. Prednisolone B. Tiotropium
....
🍒Smoker, SOB, cough yellowish sputum, fever, examination wheezing, febrile, Dx?
A.community-acquired pneumonia ✅ B.Acute bronchitis C.COPD. D.TB
...
🍒Best diagnostic test for asthma?
A- Spirometry✅✅
B- B- Methacholine test ( if asymptomatic or normal spiro do methacholine test. 🍒
🌹Patient presented with SOB only exertional, did spirometry and was normal:
A- repeat spirometry B- methacholine challenge test
🌹Pregnant lady presents with cough of white sputum then cough increased in severity
and she developed SOB she is allergic to dust, mites and pollens what is the most imp
investigation? Spirometry ✅ ✔
🌷accurate test for Asthma = Spirometry🌷
🌹How to monitor asthmatic response to treatment : A. spirometery✅. B. PEF
🌹Pregnant lady presents with cough of white sputum then cough increased in
severity and she developed SOB she is allergic to dust, mites and pollens what is the most
imp investigation? Spirometry ✅. CXr CT

COPD
🌹Copd cor pulmonale whats best thing to improve survival ? A. Diuretic B. O2 Oxygen
therapy is great importance COPD
....

COPD patient with lower limb edema and hepatosplenomehay Right heart failure signs
What to do to confirm dx? -CXR Echo ✅✅✅ ïŽ Ó ‫ ﻣﻨﻄ‬ECG
....
What improves survival or reduce mortality in COPD? Smoking cessation ✅
…..
🌹Emphysema: low DLOC
‫‘ ﻋﻠﻴﻬﺎ داﺋﺮة ﻫﺬە اﺳﻤﻬﺎ‬o‫ ا‬bronchovascular markings ‫ﺎﺑﻬﺎ‬y‫ وﻣﻦ اﺣﺪ اﺳ‬emphysema
🌹 Most common symptom of emphysema ?A.productive cough B.dry cough C.dyspnea

🌹 Scenario about a man with epilepsy poor controlled, have productive cough and fever
Asked about what the source of infection?
A- Aspiration B- TB C- Infectious mononucleosis D - Pneumonia
...
🌹COPD on PFT ?
C. decrease TLC , decreased FEV1/FEVC <0.7, increase VC
D. increase TLC , decrease FEV1/FEVC <0.7, decrease VC
....
🌹How to assess COPD severity/prognosis: FEV1
....
🌹 Indication of home therapy for copd pt:
A. wo reading of PaO2 less than 7.3. B. Less than 7.3 Kpa. C. 2 measurements Ko2 < 7
...
🌹best way to deliver oxygen in COPD pt?
A. Mask B. Nasal cannula C. Venture bag D. Non rebreather mask
...
🌹COPD patient developed sudden symptoms of spontaneous pneumothorax, x ray was
done and showed *2 cm* pneumothorax, mx? A. Needle B. Chest tube C. Observation
🍒Scenario of SBP mx: A. lactulose enema+ceftriaxone✅ B. furosemide+IV metronidazole
# Copd elderly admitted exerberation at night became aggresive disoreinted what the
nurse should do first:
-Call the duty dr for lorazepsm iv. -Restrain her
-Call family sit beside her
-elevate head of bed put nasal oxygen try to oreinted her about place and time ✅
......... ....... ....... .....
# Patient KNOWN COPD when do you start home oxygen therapy?
PO2 less than( 7.3kpa ) on two occasions
….
#COPD, affected conscious level, low pH? A- Bipap B- Mechanical ventilation ✅
......
COPD & heavy smoker hypoxia co2 retention patient drowsy? Mechanical ventilation.

COPD exacerbation and he is in coma, what to do? mechanical ventilation ✅ ✔
….
#COPD elderly patient presented to the ER distress and full of secretion how
you will manage =
A. CPAP. B. mechanical ventilation. C. Nasal oxygen. D. Aminophylline infusion
…..
🌹COPD pt came with respiratory functions detotriation and LOC with sever
secretions in the throat mx? A. cpap B. mechanical ventilation C. o mask
....
🌹COPd with signs of respiratory acidosis ? A.Mechanical ventilation
….
Absolute indications of mecanical ventilantion in severe acute asthma:-
-Coma ✅ -Respiratory or cardiac arrest -Refractory hypoxemia
‫ اﻧﻪ‬¤‫ ﻋ‬ñ • ç ‫ﺔ اﻟﺼﺤ~ﺤﺔ ﻻن ﻣﻦ اﻟﻤﻮﻗﻊ „ﻠﻬﻢ ﻣﺘﻔﻘ‬u‫ ﻣﻌﺮوف ا´ﺶ اﻻﺟﺎ‬Òç‫ ﻏ‬coma and arrest ‫ ﻣﻦ ﺿﻤﻦ‬absulte
indican
,….
Patient k/c of COPD alert, has moderate respiratory distress.SO2 93%, ABG showed
hypoxia, hypercapnia and acidosis. What is next step:
A. noninvasive ventilation✅✅. B. Decrease oxygen
C. Increase oxygen D. Mechanical ventilation (when O2 below than 88 ) .
.......
When to consider invasive MV?
➔ PaO2 < 40 mmhg. ➔ Ph < 7.25. (Acidosis) ➔ CO2 > 60 mmhg. ➔ Respiratory arrest.
➔ Cardiovascular collapse. ➔ Severe exacerbation with a lot of secretions.
......
🌹Copd acute exacerbation with a mouth full of secretions = invasive Mecanical
ventillantion .🌹
.........

# exacerbation COPD Best management == non invasive mechanical ventilation✅


......
COPD patient with exacerpation, given SABA and not improved, next management is: (O2
sat is 89%) and he is conscious: A- Iv steroid B- noninvasive positive pressure vent. ✅
...... ....... .....
67 years old male, known COPD, come with severe respiratory distress, ABG: pH 7.2, O2
saturation 85, PO2 7.2, PCO2 40, bicarbonate 22. TTT? ‫ﻧﺣﺗﺎج ﻧﺗﺎﻛد‬
A- CPAP (answer) ✅. B- BIPAP. C- Mechanical ventilation D- Observation
.....
🌹COPD patient came with severe cough, sputum production and cyanosis what
do?A.Intubation and mechanical ventilation B. oxygen C. lasix D. bipap
.....
patient with COPD develop *reproductive cough* for 3 months, what is the diagnosis?
Chronic Bronchitis ✅ ✔
..... ..... ...
COPD patient with lower limb edema and hepatosplenomegaly Right heart failure signs
What to do to confirm dx? A-CXR. B-Echo ✅ ✔ C-ECG
.. . . .
#-Patient known case of copd and presented to er with sign and symptoms of
heart failure what is the initial investigation A-Spirometry B-Echo C-Ecg
.....

Patient KNOWN COPD when do you start home oxygen therapy?


A- PO2 less than 7.3k pa two occasions ✅ ✔
B- Less than 7.5 on two occasions
. .. . . .
🌹 Copd Oxygen at home.When you decide the pt need it:
A. 2 measurements Ko2 < 7
COpd home 2 measur
KO2 <7
‫ﻌﺔ ﻋﺸﺎن اﻟﺪوام‬x‫ﻞ اﻟﺴﺎﻋﺔ ﺳ‬x‫ ﻻزم ﻧﻄﻠﻊ ﻣﻦ اﻟﺒ}ﺖ ﻗ‬.
.......

Which one of the following gives BNP false positive?


1)COPD✅ 2)Obesity 3)Ace. 4)Furosemide. 🌹BNP false = COPD 🌹
…..
🌹🌹pulmonary function testing in COPD? decrease (FVC/FEV) increase (TLC)🌹🌹
8• ‫ﻂ =ﻣﺮ‬Ï‫ ر‬COPD ‫ اﻧﻪ •ﺎﺧﺬ ﺷﻬﻴﻖ و„ﻤ~ﺔ دﺧﻮل اﻟﻬﻮاء ﻗﻠ~ﻠﺔ‬ŒŽ • ‫~ﺪ ﻛﻔﺎﺋﺔ اﻟﺮﺋﺔ‬Ù‫»ء ﻣﺴﺪود ﻋﻨﺪﻫﻢ ف ا‬
Ž
¼ ‫„ﻞ‬
‫ﺴﺪاد ﻫﺬا‬D‫ﺐ اﻻ‬Þ‫ﺴ‬7 FVC ‫ﺴﺪاد‬D‫ﺐ اﻻ‬Þ‫ﺴ‬7) ‫ (ﺗﻘﻞ‬Ð‫ اﻟﺜﺎﻧ~ﺔ اﻻو‬ŒŽ • ‫ „ﻤ~ﺔ اﻟﻬﻮاء اﻟﺪاﺧﻞ‬،
‫و‬TLC ‫ اﻟﺮﺋﺔ‬-o‫ﺬە اﻟﻤﺤﺎوﻟﺔ ﺗﺨ‬¹ ¦ƒ § ‫ﻤ}ﺔ اوﻛﺴﺠ‬á ²ÿé‫ ﻟ}ﻪ ؟ ﻻن اﻟﺮﺋﺔ ﺗﺤﺎول وﺗﺤﺎول ﺗﺎﺧﺬ ا‬، ‫ﺪ‬wÙ‫ﻳ‬
Tlc = Total lung capacity: the volume in the lungs at maximal inflation .
.... ..... ....

🌹80 ys old smoker 40 ys labs all normal except high .ca ?


A. paget's B. hyperparathyroidism Lung Ca if not in choices = bone metastsis

Pt smoker and dyspnea and character of clubbing A.Copd B.Asthma C. Bronchial cancer
✅✅
‫ ﻣﺮض‬copd ‫ﺐ‬Þ‫ ب ﻧﻔﺴﻪ ﻻ ´ﺴ‬clupping ‫–ﺾ‬À‫ﻦ اذا ﺷﻮﻓﻨﺎﻫﺎ ﻣﻊ ﻣ‬Ä‫ ﻟ‬COPD ‫ﺎﻟﺮﺋﺔ‬u ‫ﻃﺎن‬è ‫ﻣﻌﻨﺎﻫﺎ ﺟﺎء ﻟﻪ‬
COPD = does not cause clubbing, but if clubbing is present in COPD, underlying lung cancer
and bronchiectasis must be ruled out 🍒🍒Pt smoker and dyspnea and character of
clubbing = Bronchial cancer🍒🍒
.......
Case of old smoker c/o cough, loss of nail bed angle? beonchogenic carcinoma

🍓Smoker pt, c/o with dyspnea and couph with hemoptysis, trachea wasn’t shifted, xray
showed massive right plueral effusion, whats the cause:
A. Heart failure B. T.B C. Brochogenic carcinoma with ipsilatral bronchus compression✅✅
. ‫ وﺛﺎﻧﻲ ھو ﻣدﺧن وﺛﺎﻟﺛﺎ ﻛﺣﺔ ﻣﻊ دم اﻟﺛﻼﺛﺔ ﻛﻠﮭﺎ ﺗﻣﺷﻲ ﻣﻊ ورم اﻟرﺋﺔ‬plural effustion ‫ﻗﻠﻧﺎ ﻛﺎﻧﺳﯾر ﻻن اﻟورم ﺑﯾﻌﻣل‬
🌹 smoker came with dyspnea and hemoptysis , examination and x ray showed positive
pleural effusion , but the trachea and the apex beat where not shifted ( ‫ ) ﻛﺬا ﻣﻜﺘﻮب‬. what
is the cause of non shifting trachea and apex beat ?
A. - transudate pleural effusion B. - TB pleural effusion
C. - cancer in the ipsilateral main bronchi ✅
🍒positive pleural effusion dyspnea and hemoptysis smoker= Cancer bronchio🍒
🍒Pleural effusion in patient has malignancy analytic Show Light criteria of pleural effusion,
High protein, and the pt have symptoms of malignancy?
A. Trans exudative due malignancy B. Exudative due malignancy
🍒A case of traumatic effusion what to find in CXR ? Effusions

🍒patient K/c COPD C/O SOB palpitation O/E irregular pulse Ttt?
A-Cardoversion B-Adenoseine C-Amiadrone ✅ ‫ ﻣﻨﺘﻈﻢ‬Òç‫ﺾ اﻟﻐ‬y‫ﻋﺸﺎ اﻟﻨ‬
🌷COPD C/O SOB palpitation O/E irregular pulse = Amiadrone🌹
......
🌹 smoker with recurrent respiratory infection and productive cough ? chronic bronchitis
Smoking is a strong hint for COPD + all other sx are seen in COPD
….
🌹 COPD function ==increases lung capacity
….
COPD Pulmonary function test:
A. Inc TLC, Dec FEV1/FVC, Normal VC. B. Inc TLC, Dec FEV1/FVC, Dec VC
…..
#COPD monitoring ? FEV1
….
🍒🍒COPD Best indication of disease progression ? FEV 1🍒🍒

🍓Know case of copd with cor pulmonale. which of the following will decrease mortality ?
A. oxygen mask therapy B. inhaled long acting beta blocker
C.Long term 02 therapy D.NPPV
🍎Common etiolog to cor pulmonary is COPD 🍎
….
🍓Smokers to non-smokers risk:-
Lung cancer: by 25 folds.
COPD:12 to 13 folds.🍓
Stroke and coronary
….
child with SOB and *Stony dullness* in percussion, what is the diagnosis?
Pleural effusion ✅ ✔
....... ............
🌹Burn with carbon monoxide inhalation how you will mange ?
Intubation and 100 oxygen
🍓- Patient was engaged in building fire has signs of inhalation injury ( nasal hair)
- admit to ICU for observation Correct answer : INTUBATION
..........
🍓COPD > ration less 70% and low DLCO what it the dx?
A. emphysema✅. B. bronchitis
.......
🍓obese snoring , lack sleep , sleep study done (OSA), how you will ventilate:=CPAP
....
🌹 Patient suspected to have left sided heart failure, had slightly positive BNP. What can
cause such condition? A. ACEI. B. Furosmide. C. COPD ✅
..
🍒smoker with non productive cough, fatigue and loss of 15% of weight what is the
diagnosis? A.COPD. B.Lung cancer ✅
......
🌹Smoker with productive cough=COPD

1Blood disorder
🍓 Patient with typical picture of Hsp asking about dx :
A. Vitamin c deficiency. B. Factor vll deficiency C. Hypersensitivity vasculitis
D. Drug Hypersensitivity reaction
🍓 patient 65 years old post massive surgery! Patient lost alot of blood +patient have given
15 bage of RBC after time nurse notes that the patient start bleeding from iv and
nasogastric tube? What is the cause of the bleeding
A. von Willebrand disease. B. thrombocytopenia. C. blood reaction

🍓Type of anemia in alcoholism? . Megaloblastic folate deficiency

A 73-year-old man comes to the office with fatigue that has become progressively worse
over the last several months. He is also short of breath when he walks up one flight of
stairs. He drinks 4 vodka martinis a day. He has numbness and tingling in his feet. On
physical examination he has decreased sensation of his feet. His hematocrit is 28% and
MCV is 114 fL (elevated). What is the most appropriate next step in management?
a-Vitamin B12 level. b-Folate level. c-Peripheral blood smear.✅✅✅
d-Schilling test. e-Methylmalonic acid level.
Correct answer : C. Although a macrocytic anemia could be from B12 or folate deficiency,
direct alcohol effect on the bone marrow, or liver disease, the first step is a peripheral
smear.
….
🌷He drinks 4 vodka martinis a day, numbness and tingling in his feet His hematocrit is
28% and MCV is 114 fL (elevated). First step = Peripheral blood smear🌷
..... ..... ....

🌹Asymptomatic 3 months old baby, with CBC result of mild microcytic hypochromic
anemia, what is the diagnosis? Thlasimia trial ✅ ✔

🌹pt on Total parenteral nutrition give value of PT and INR what is the cause ?
A. DIC B. vitamin K deficiency ✔

🌹60 y/o patient blood transusion after 10 m developed pain in the site of canula and
difficulty breathing and is febril:
A_ Febril non hemolytic anemia. B_Hemolytic anemia . C_Allergic reacti
🌹 CLL patient on day 17 develops neutropenic fever, what will you do:
A. Septic screen B. Septic screen and start oral broad spectrum abx
C. Septic screen and IV broad Abx

polythycemia 50 ys old what will be with him= Splenomegaly

²§‫ =ﻃﺤﺎل ﻛﺒ‬²§‫ر`ﻂ =دم ﻛﺜ‬.


by polycythemia vera makes your spleen work harder than normal, which causes it to
enlarge
🌹pt medically free go to tooth extraction and pt develop petechial rash andHb high+
erythropoietin low + Platelet high
A myelofibrosis B essential thrombocytopenia C_ poly sethemia vera✅
🌹Pt after blood transfusion become fibile:
A. Fibrile non hemolytic✅ B. Allergic reaction
🌹Bleeding from injection site after trensfusion of blood :
A. thrombocytopenia B. transfusion reactions

🍒How to prevent acute chest syndrome? Hydroxyurea


🍓21 boy go to dentist clinic for tooth extraction he have bleeding after extraction and
bruises and he know case of factor VIII dif , lab show slight increase in PT : Von wellbrand.
‫ =ر`ﻂ‬VON ‫ ﺣﺮوف‬٣ = V III= III
..........

Malaria
🍓 Malaria negative results what to do next
A. repeat thin. B. Repeat thick. C. Repeat after 48 h D- repeat every 8hrs for 43 hrs
🍓 Which anti malaria drug mostly malaria will be resistant for it :
A. Malarone. B. Atovaquone-proguanil. C. Mefloquine D. Chloroquine
🌹Malaria prevention sudan = give Mefloquine 🌹
🍓Case of pt came from africa w hepatosplenomegaly := malaria
🍓What is the peak time of west nile mosquito Infection ?
A. early morning ✅ B. Midday . C. Early night . D. Nigh
🌹35 indian male came for mecca then he developed convulsion intermittent, fever
breathing problem after 2 days What is this
A. Malaria B. Tb C. Yellow fever D. Influenza 🌻 Malaria present with CNS manifestatio
🌹pt came only with fever and hx of Southeast Asia ? A. malaria B. danger fever
🌹Most common cause of fever in subshara Africa? Malaria 🌹
🌻Sub saharan africa > malaria 🌻South east asia > dengue
🌹 Retro Orbital pain, joint pain , muscle pain , came from travel ?
A. dengue. B. chikunga. C. ebola. D. corona
🌹Young female presented with headache and fever,she recently came from
jeddah Labs shows low PLT , Low hemoglobin = Dengue
🌹Person wants to travel to malaria endemic area how to prevent malaria?
A- Chemoprophylaxis✅. B- not travel to endemic area. C- Vaccines
Chemoprophylaxis== use of drugs to prevent disease
🌹Malaria treatment: A. Quinine B.Mefloquine C.Artemisinin combination therapy
soldier going to southern of saudi arabia came for malaria prophylaxis :
Atovaquone / Proguanil 🌹‫ = رﺑﻂ‬atov = ‫اطﻮف ﺑﺎﻟﺤﺮم = ﻓﻲ اﻟﺴﻌﻮدﯾﺔ‬ .
🌹Malaria prophylaxis? Malarone ‫ھﻲ ﻧﻔس اﻟدواﺋﯾن اﻟﻲ ﻛﺗﺑﻧﺎھﺎ ﻓوق‬
🌹Malaria blood smear = M.falciparum 🌹.

‫ﻂ‬Ï‫ = ر‬fal = ‫–ﻠﺔ‬c‫ﻓﻠﻮﺗﺎت =ﺧﻠ~ﺔ ﺷ•ﻠﻬﺎ (ﻓﻼت = )ﻃ‬.


🌹Malaria case .. pt complain of fever Hx of Travel to work in sudan Blood smear -ve
What next A. - Repeat thin one B. - Repeat thick one C. - Repeat 2 for 48 hr ✅
‫ ﻟﻠﺴﻔﺮ‬Ò• ç ‫ ﺗﺠﻬ‬ñ
• ç ‫ ﻧﻘﻮم ﻣﻦ اﻟﻨﻮم وﻧﻄﻠﻊ •ﺎﺧﺬ ﻳﻮﻣ‬ýÓ‫ ﺳﺎﻋﺔ ﺣ‬٤٨ Ð‫ ا‬ñ
• ç ‫ﻂ =اﻟﺴﻔﺮ ﻟﻠﺴﻮدان •ﺠﻠﺲ ﺳﺎﻋﺎت ﻣﻦ اﺛﻨ‬Ï‫ر‬
🌹Patient known case of vitiligo, complaining of (signs of Vit B12 deficiency) what is the
treatment: A. Oral B12 B. Parenteral B12 ✅🌹
🌹B12 deficiency anemia pic (very common pic) with scenario mention tingling and loss of
sensation of limbs? Give B12
🌹hypersegmented neutrophils are associated with which type of anemia ?
B12 and folate.
‫ﺷوﻓﺗوا اﻟﻲ ﻟوﻧﮭﺎ ﺑﻧﻔﺳﺟﻲ ) اذا ﺷوﻓﺗوت داﺧل اﻟﺧﻠﯾﺔ ھذا اﻟﻣﻧظر ﻗطﻊ ﻛﺻﯾرة ﺻﻐﯾرة زي ﻛذا ﺑﻧﻔﺳﺟﯾﺔ اﻋروا اﻧﮫ ﻋﻧده ﺷﻲء اﺳﻣﮫ‬
. ‫ ﺑﺷﻛل ﺣﺎااد‬١٢ ‫ اﻟﻲ ﺗﯾﺟﻲ ﺑﺳﺑب ﻧﻘص ﻓﯾﺗﺎﻣﯾن ﺑﻲ‬megaloplastic anemia ‫وھذا ﺑﺳﺑب‬hypersegmental nuteophil
B12 deficiency is associated with an increased which of the following :
1- B12 deficiency 2-methylmalonic acid level✅ 3-Increased LDH
, ١٢ ‫ =ﻋﺷﺎن ﯾﻛون ﻣﻣﺛل ﻋﺎﻟﻲ اﻟﻛﻔﺎﺋﺔ = ﻻزم ﯾﻛون ﻋﻧده ﻧﻘص ﻓﻲ ﻧوﻣﮫ وﯾﺳﮭر وﻣﺎﯾﻧﺎم ﻗﺑل اﻟﺳﺎﻋﺔ‬increase ‫رﺑط = ﻣﯾﺛﯾل‬
..........
🌹hematouria + bleeding from other site , change on level of cons + sizer + low plattlet =
FFP and plasma exchang 🌹

🍒old pt with Chronic kidney disease present with labs show high WBC with arthritis= CLL

🌹 microcytic hyperchromic RBC = hereditary spherocytosis = Osmotic fragility test 🌹
‫ﻂ ﺣﺮف‬Ï‫ ر‬O ‫ ﻣﻦ‬osmatic ‫ = ﺷ•ﻠﻬﺎ داﺋﺮة‬spherocytosis. ‫–ﺔ‬À‫ﺷ•ﻞ اﻟﺨﻼ•ﺎ داﺋ‬
. . . .. .
🌹 Parvovirus b 19 in sickle cell anemia = Aplastic crisis 🌹

18 Y has ALL. 17 days after chemo he develops fever around 38.7 C. No focus of infection
found. Labs shows: low WBC 0.6 (normal was 4-10x..), Neutrophil level 60% (normal 40-
60%), i’m not sure if he has low platelets as well, what you’ll do:
B. Blood urine... culture and paracetamol C. Blood, urine,... culture and PO Abx
D. Blood, urine,... culture and IV Abx✅✅✅
4 O
‫ﺎ‬7‫„ ﺟﻤﻴﻊ اﻟﺨﻼ‬6 ‫ﻤﺎ ﺗﻌﻤﻞ ﻧﻘﺺ‬#‫ﺎ (و اﻟﻠﻮﻛ‬#‫ﻤ‬#‫ء ﻋﻨﺪە ﻟﻮﻛ‬N
6 ‫ اول‬wbc , nutrophil ...etc , ‫ وﺗﻌﺪاد‬، ‫ﻘﻮﻟﻚ ﻋﻨﺪە ﺣﺮارة‬7 ‫ ﻓﻬﻨﺎ‬nutrophil ‫• =ﻻن ﻫﻢ‬6#‫ﻃﺒ‬
‫ﻪ‬#‫• ﻫﺬا ﻣﻌﻨﺎﻫﺎ ﻓ‬6#‫ﻮﺻﻞ ﻟﻠﻄﺒ‬ž‫ ﻓﻠﻤﺎ ﻳﺮﺗﻔﻊ و‬، ‫ﻜﻮن ﻧﻘﺺ‬7 ‫ اﺻﻼ ﻋﻨﺪﻫﻢ‬infection ، ‫وﻻﻧﻬﺎ ﻋﻨﺪﻫﻢ ﻧﻘﺺ )ﺎﻟﻤﻨﺎﻋﺔ وﺧﻄﺮ ﺟﺪا ﺟﺪا اﺻﺎﺑﺘﻬﻢ ب اي اﻧﻔﻜﺸﻦ‬
‫ﺪ وﻟ ﺲ دواء ﻓﻤﻮي‬ž‫ﺎﺧﺬوا اﺑﺮة ﻋ• اﻟﻮر‬7 ‫ﻻزم‬
…..
Patient known case of polymyalgia rheumatica presents with malignancy
symptoms, what is the diagnosis ? CLL
🌹 known case of polymyalgia rheumatica presents with malignancy symptoms (weight
loss and other non-specific constitutional sx), what is the diagnosis ? Labs showed anemia,
low platelets and increased WBC (lymphocytes) = CLL

Neonate complaining of jaundice after 12 hours of delivery .. blood film and I think it show
microcytic hyperchromic RBC (hereditary spherocytosis ) What investigation you will order
? a) G6PD. b) Osmotic fragility test ✅Pyruvate kinase deficiency
🌷SCA patients take all vaccination after blood transfusion except MMR no if the
transfusion within the 11 months🌷
🍒5year old with wrist pain and ankle pain or bone cant remember and fever for one
week. == sickle cell == could be dactylitis or hand foot syndrome with SCA 🍒
🌹24y kc of SCA came with headache, lethargy, pallor, Lab: hg: very low, Reticulocytes:
high, what most likely diagnosis?
A.Splenic sq craisis B.Thrombotic crisis C.Hemolytic crisis ✅ D.Aplastic crisis
Sickler with pain and pallor , hbg low , reticulocyte high .. what type of crisis ?
A. hemolytic B. aplastic C. sequestration
‫ اي ﻣرة ﺗﺷوﻓوھﺎ اﻋرﻓوا ان اﻟﺧﻼﯾﺎ اﻟﺣﻣرتء ﺗﻛﺳرت وﺧرج ﻣﻧﮭﺎ ھذه اﻟﺧﻼﯾﺎ اﻟﺷﺑﻛﯾﺔ‬reticulocyte high ‫ﻻن ﻗﺎل‬
🍒limps pain with chest sx asking about dx = SCD🍒
🌹SCA came with chest ,back and lower limbs pain what’s mostly dx?
A. PE B. acute chest syndrome

🌹SCD patient presents with hands and foots pain, the most important ongoing care is:
A. Penicillin and vaccines ✅ B. Iron C. Vit B12
‫ﻂ =ﻗﺪم و–ﺪ‬Ï‫ = ر‬pencillin = ‫ وﺳﻄﻬﺎ‬nice = ‫ﻻزم ﻧ—–ﻦ اﻟﻘﺪم واﻟ~ﺪ و–ﻜﻮن ﺷ•ﻠﻬﺎ ﻧﺎ´ﺲ‬
........
🌹Child 6yrs c/o fatigue, pale ....Ix hb 5.6 retics high SCD+ve treatment?
A. Blood transfusions ✅. B. IVIM C. Steroid
.............
7 y/o presented to with SOB and fatigue Labs showed anemia + thrombocytopenia (no
WBC) Sickle screen was positive Which one of the following is the most important
investigation? A.Bone biopsy B.Ct abdomen C.Us abdomen D.Reticulocyte count ✅✅
..........
🌹Pt with respiratory distress and high fever also carsales , effected right lobe and low hb
and high Ritcs count and pulmonary systolic murmur what is the diagnosis:
A. Sickle cell anemia✅ B. Squsteration C. Pneumonia D. MI

Crisis of sickle cell patients , pale, low hemoglobin, no hepatosplenomegaly, type of crisis?
A. aplastic✅ B. sequestration
..........
🌹Sickle cell disease boy with pain in hands and foots. What is the initial management?
A. Iv fluid and analgesia B. antibiotic C. blood transfusion
....
🌹Sickle cell disease boy with severe abdominal pain, he looks pale and
fatigued, on P/E he has splenomegaly. Labs shows hemoglobin 3.2 gm/dl. What is the
initial management? A. Iv fluid and analgesia B. antibiotic. C. two pRBC transfusion
SLD=RBC
....
Sickle cell anemia boy Came with lethargy pallor and headache Dx?
A. Aplastic crisis. B. Sequestration crisis C. Thromboembolic crisis D. Hemolytic crisis
..
scenario of anemic pt. Lab showed *pancytopenia* dx:
A- aplastic anemia ✅ B- iron def. C- B12 def. pancytopenia = aplastic anemia
‫ ﻣﺎﯾﻧﮭﺿم‬aplastic ‫ ﻟﺑﺎن = ﻟﺑﺎن ﻣﺛل اﻟﺑﻼﺳﺗﯾك‬Pan = ‫رﺑط = ﺑﺎن‬
.. . . . . .
Sickle cell disease patient. Present with crisis. Respiratory symptoms. Splenomegaly.
Hemoglobin 3.4 g/dl (iʼm sure of v'lue). Wh'tʼs the best initi'l step in m'n'gement?
- Splenectomy - IV fluid and analgesia ✅✅ - RBC tr'nsfusion
..
question with a description of blood smear that showd (crescent RBC’s) = Dx: Sickle cell
anemia.

.......... ......
🌹Sickl cell anemia with Hb 5= blood transfustion 🌹

🌹2 Case of sickle cell =vasoaclusive (back pain🌹


,,
🌹Diabetic obese elderly female e sickle cell trait has Lithiasis what kind of stone she has:
A.mixed ✅. B. Fat C. pigmented
...........
🌹Women with sickle disease had children with sickle cell or lost children because of it not
sure she married a new husband and wants to know if she gets pregnant what are the
chances of her children to have it , what’s ur next step?
A. genetic testing of both B. electrophoresis of new husband ✅✅
C. electrophoresis of the woman
......
🍓 pt recurrent splenic sequestration ask about management = splenectomy
..
.......
🍓case with high HbA2 = A. beta thalassemia minor✅ B. beta thalassemia major
.......
🍒β-Thalassemia Minor= = defect in single gene (heterozygous) =
Hb electrophoresis show = specifc:
HbA2 increased to 3.5-5%.
no treatment required = a palpable spleen is very rare = microcytosis anemia.
......
🍒β-Thalassemia Major = defect in both gene homozygous = hemolysis of RBCs, and
increase in HbF(90_ 100%) = low HbA2 (2%) =severe anemia Hg(4_6) jaundice = iron
overload progressing to hemochromatosis (secondary to repeated transfusions )= gross
hepatosplenomegaly = skull x-ray has “hair-on-end” appearance =
pigmented gallstones= Treatment=
lifelong regular transfusions + iron chelation (e.g. deferoxamine) + splenectomy .
🌹 Male pt asymptomatic came for check up, drinks alcohol occasionally every weekend.
LFT showed mildly elevated enzymes, ferritin 490 high, TIBC high, dx:
A. alcoholic hepatitis B. hemochromatosis

......

Iron
🍒 Brittle Nails pict Dx? IDA🍒

Child only eat milk with little meat presented with microcytic hypochromic what is the
treatment? A.Multivitamins with iron B. Iron and re-evaluate after 4 weeks i think ✅✅

🌹 laboratory findings of Iron Defincy = All low except RDW,TIBS,transferrin are high 🌹
🌹 Child of iron ingestion came after several hour Iron 90, What you will do?
A. Iv desforoxmine ✅✅ B. N actylcystine C. Gastric lavage D. Activated charcoal
🌹pediatric with low mcv, mch = treatment
A. multivitamin with iron B.trial of iron supplement and follow up✅✅
🍒Case pt with bronze skin and long case with labs , what is important Investigation ?
A-liver boipsy to see iron ✅ B-serum copper C-urine something
🌹iron overdose before 4 to 5 hours and he was stable = Don’t need to give him at this
stage
🌹There was Same question but symptomatic ? IV deferoxamine🌹
🌹Iron overload is managed within 8 hours ? -Deferoxamine IV
🌹 In ER if pt is confused ( severe IDA ) best to give iron IV , IM respectively
🌹 Child *confused* and fatigued laps microcitic hypochromic anemia
low ferrtin high TIBC treatment A-oral iron B-Im iron C-iv iron ✅ ✔ D- packed RBC
🌹 pt pregnant have anemia and treat with iron supplements 't 20 week c'me with f'tigue
'nd tiredness hg low mcv low dx? A-IDA ✅✅ B-b th'l'ssemi'
🌹pregnant have anemia +hg low mcv low= iron defincy anemia 🌹
🌹Pt with anemia after two visit not improved with iron supplements what
test do? Hb electrophoresis
🌹Child appears pale hb 8 ,hypocromic microcitic=Trail iron and following-up 🌹

...................................
🌹severe case Glucose-6-phosphate dehydrogenase deficiency Treatment ?
Blood transfusions
‫ اﺣﻤﺮ‬²§‫ ر`ﻂ =ﻋﻨﺪە ﻧﻘﺺ ﺷﺪ“ﺪ {ﺎﻟﺴﻜﺮ =ﻋﻄﻮە ﻋﺼ‬.:)
Treatment
• folic acid
• stop ofending drugs and avoid triggers
• transfusion in severe cases
............

ITP
🌹 After URTI child complain of bruises ,, lab show low platelet?ITP 🌹✅✅
🌹wich will cured after (splenoctomy)?
1_idopathic thrombocytopenia purpura ITP 2_ Herdetary spherocytosis
🌹curable by splenectomy?‫¸ ﻟﻤﺎ ﻧ—–ﻞ اﻟﻄﺤﺎل‬Ž á ‫ ﻣﺮض ﻳﺘﻢ اﻟﺸﻔﺎء ﻧﻬﺎ‬.
A. -Beta thalassemia B. -Alpha thalassemia C. -sickle cell D. -ITP✅

ITP case. What is the best appropriate management?


1_ Splenectomy. 2_Prednisolone ✅. 3_ Azathioprine. 4o_Cyclophosphamide
ITP (immune thrombocytopenia) in child has petechi',ecchymois ,pl'telet 15 m'nge?
A-Intr'venous y globin B-platelet tr'nsfusion. C- observ'tion ✅
Treatment of ITB : ‫ﺎﺧﺘﺼﺎر •ﺨﺺ اﻟﺴﺆوال ﻓﻘﻂ‬u ‫ﻘﻮﻟﻬﺎ‬u
‫ اذا‬plattlet ‫ون‬Ò• ç ‫ ﻧﻌﻄ~ﻪ ﻛﻮرﺗ‬١٥ ‫اﻗﻞ ﻣﻦ‬
‫ اذا‬plattlet ‫ ﻓﻘﻂ •ﺤﺘﺎج‬١٥ ‫ ﻣﻦ‬ÒôÙ‫ ا‬observ'tion
🌹Picture of schistocyte after URTI , petechial rash and low platelet,Dx:
A. ITP B.Thrombotic thrombocytopenic purpura C. DIC D. HUS
‫ ﻣﻦ اﺳﻤﻬﺎ‬thrombocytopenia ‫ﻘﺎ‬u‫„ﺎن ﻋﻨﺪە ﺳﺎ‬+ ‫ اﻟﺼﻔﺎﺋﺢ ﻗﻠ~ﻠﺔ‬ýŽ • ‫ •ﻌ‬urti ‫ﺎﻟﻤﻨﺎﻋﺔ‬u ‫ ﺣﺼﻞ ﻟﻪ ﺧﻠﻞ‬ýŽ • ‫ﻋﺪوى •ﻌ‬
+ ‫ ﺣﺼﻞ ﻟﻪ‬petechia ýŽ • ‫ •ﻌ‬purpura .
🍓After URTI , develop thrombocytopenia gum bleeding and brusing in lower extremities =
Immune thrombocytopenic purpura
‫ﻘﺎ‬R‫ﺎﺟﻤﺖ اﻟﺼﻔﺎﺋﺢ [ﺴ]ﺐ وﺟﻮد اﻧﻔﻜﺸﻦ ﺳﺎ‬V ‫ﺎﻟﻤﻨﺎﻋﺔ‬R ‫ﻠﺔ‬P‫ ﻣﺸ‬MO N ‫ﻌ‬K ‫ﺲ‬I‫ﺎن ﻓ;ﻪ اﻧﻔﻜﺸﻦ واﻻن ﺣﺼﻞ ﺑﺮوز‬8 ‫ﻠﻬﺎ‬6‫ﻣﻦ اﺳﻤﻬﺎ ﻗ‬
……….
🌹 Patient has fever and headache petechiae. what’s the diagnosis ?
A. Hus B. TTP C. ITP

- thrombocytopenia and uremia in studies and has fever and headache


a. HUS b. TTP c. ITP
TTP : Fever, headache , hemolytic anemia, thrombocytopenia
HUS: hemolytic anemia, diarrhea, renal failure
ITP: Hx of URTI with low platelet only
HSP : Hx of URTi , abdominal pain , joints pain , hematuria, rash in buttocks

🌹Pt with fever and headeche . Pic shows shistocyte == TTP ✅Schistocytes are often
seen in patients with Thrombotic thrombocytopenic purpura🌹
🌹 picture of Schistocytes blood smear Young lady with 2 days history of fever and
headache. The patient presented only with petechial rash
Normal PT, aPTT, INR Thrombocytopenia
What is the most likely Dx ?
A. HUS ( must be E.coli infection )
B. TTP✅ (adult + Schistocytes+ fever and headache + petechial rash + Normal PT, aPTT,
INR = T T P)
C. ITP
D. DIC
Young lady with 2 days history of fever and headache. The patient presented only with
petechial rash. High [ PT, aPTT, INR ]. Low [ Platelet, Fibrinogen ]
A. HUS. B. TTP. C. ITPD. DIC ✅
‫ „ﻠﻤﺔ اﻟ† ﻟﻬﺬا اﻟﻤﺮض ﻫﻮ‬High [ PT, aPTT, INR ] +
Low [ Platelet, Fibrinogen ]
‫اﻟﺼﻔﺎﺋﺢ ﻗﻠ~ﻠﺔ ﻟﺬﻟﻚ •ﺤﺼﻞ ﻧ—–ﻒ‬
🌹2 qs about TTP with peripheral blood smear pic .. ask about Dx and then management =
Exchange transfusion
🌹 Patient with TTP scenario and labs. What’s the management ?
A. IVIG B. Blood transfusion C. Plasma transfusion
🌹 Patient done a major surgery which required 15 blood units, he afterwards started
bleeding from NGT, ETT, what is the dx? A. thrombocytopenia B.transfusion reaction
N.B: NO DIC in options
🌹Patient after surgery on blood transfusion developed bleeding from puncture site
what's diagnosis ? A.transfusion reaction B.DIC✅ C.thrombocytopenia
🌹23 y/o presented to ER with fever and hedache, on examination there was petechial rash
all over her body, vitally stable other than high temperature 38
🌹- First Q: Plt were low, PT&PTT&INR all normal, high bilirubin and reticulocytes and
low Hb (indicates hemolysis), creatinine was normal, your management:
A. Steroids and IVIG. B. Exchanged transfusion ✅. C. Platelet transfusion
‫ اﻋرﻓوا اﻧﮫ ﻓﯾﮫ ﺧﻼﯾﺎ ﺣﻣراء ﻣﺗﻛﺳره وﻟﻣﺎ ﺗﻛﺳرت طﻠﻊ ﻣﻧﮭﺎ ھذه اﻟﺧﻼﯾﺎ اﻟﻲ‬، ‫ ﻋﺎﻟﯾﺔ‬reticulocytes ‫اذا ﺷوﻓﺗوا‬
‫ واﻓﺿل ﺷﻲء ﻧﻌطﯾﮫ ﻧﻘل دم‬. Hemolysis = ‫ ﯾﻌﻧﻲ‬reticulocyts ‫اﺳﻣﮭﺎ‬
🌹 Second Q: Plt low, PT&PTT&INR all high, high bilirubin and reticulocytes and low Hb
(indicates hemolysis), low fibrinogen, creatinine was normal, you dx:
A. DIC✅✅. B. ITP. C. TTP. D. HIT
🌹 Third Q: Plt was very low (in 20s) and macrocytic anemia, all other labs were
normal, your dx: A. DIC B. TTP✅. C. ITP. D. HIT
🌷Plt were low, PT&PTT&INR all normal, high bilirubin and reticulocytes and
low Hb = Exchanged transfusion
🌷Plt low, PT&PTT&INR all high, high bilirubin and reticulocytes and low Hb (indicates
hemolysis), low fibrinogen = DIC🌷
🌹Plt was very low (in 20s) and macrocytic anemia =TTP🌷

.......................................

🍓patient with polymyalgia rheumatica, which of the following is associated? CLL


🍓Patient k/c of polymyalgia rheumatica , presents with fatigue CBC shows high
lymphocyte count Dx: -CML. -lymphoma _CLL ✅ (( Chronic lymphocytic leukemia))
........ ........ .....
AML= Auer bodies + adult + presence is myelperoxidase and esterase.
ALL= in children ( Down syndrome MC )
CML= Philadelphia chromosome + LAP
CLL= asymptomic , elderly , smudge cells , CD19 , CD20

Patient of ALL post chemo develops headache and dry mouth what is the electrolyte
derangement? Hypocalcemia ✅ Hyponatremia Hypernatremia
‫ﺎﻟﺴﻴﻮم‬6‫ل اﻟ‬Ò• • ‫ﻛ~ﻤﺎوي =•ﻜ† اﻟﻌﻈﻢ و–ﺨﻠ~ﻚ ﻛﺴﻮل ﻓﻴ‬

1Pulmonary
🍓 Proper ET position how to chick :
A. -co2 monitor B. -chest movement. C. -easy insertion.
🌷 Old patient, heavy smoker present with symptoms of Horner syndrome. what type of
cancer? squamous cell cancer
🌷-Patient with Pulmonary hypertension to be evaluated for chronic thromboembolism,
which investigation has the highest diagnostic yield?
A-V/Q scan✅✅ B-Echo. C-ECG D-Spirometry CT
🌹Old man + smoking + esophageal show squamous cell with highly dysplasia next step?
A. surgical resection. B. Stop smoking

🌹Patient with bronchiolitis and given supportive treat, what to do next?


A-Shift to ICU B-Admit under pediatrics. C-Observe in ER✅✅ D-Admit to high depency unit
🌹Pt smoker with lung cancer ie Hypocalcemia ? Ddx!@
A. Squamous cell B. Adenocarcinoma. C. Bronchogenic
🌹 Smoker diagnosis as lung cancer with hypercacemia Which type ?
A-Squamous cell✅ B-Adenocarcenioma
🍓Progressive dysphagia for both solid and liquid and weight loss: A. Achalasia B.
Squamous cell carcinoma ✅
🍒CXR on hilar mass , hypercalcemia? squamous cell carcinoma🍒
🌹Elderly smoker pul s&s of bronchogenic carcinoma which type:
A. Small cell ca B. Squamous cell ca C. Adenocarcinoma
‫ الﺳﻦ‬ŒŽ • Òç‫ﻛﻮاﻣﺲ =ﻛﺒ‬Ô
o = smoker
....

🌹pulmonary HT type 3 other name is ? A. Hypoxic B. Hypercapnic C. Neurological


🌹24 hrs post op patient has dyspnea and hypoxia cause?
A. lower lobe atelectasis ✅ B. PE

🍒Cerebral palsy pt develops respiratory failure ( give ABG results wad hypoxemla ) , what
is the typop of Respiratory Failler ?
A-Hypoxiac RF ✅ B-Neurogenic RF C-Hypercabnic RF D-Hypocabnic RF
🍒Case senario about burn and soat in pt mouth and throat what to do ? intupation

🌹 Pt with orotracheal intubation who can detect it is in normal position:


A. Movement of chest B. Carbon monoxide monitoring
c. continuous capnography✅ for of carbon dioxide (CO2) in exhaled air, which assesses
ventilation
🌹 CO2 poisoning hbco 40% tx? A. Hyperbaric * B. 100% oxygen C. Blood exchange

🍓 Case of lung cancer: non smoker male with sx found nodule in the lung whats your next
step? Previous X-RAY
🌹56 Y/O non smoker , asymptotic . Discovered lung nodule during the Old x-ray
A. Come back after 6 months routine checkups , B. Repeat X-ray
🍓Dyspnea at night, sterna pain Midline mass compressed trachea :Lymphoma.
🍓Plural effusion site of thoracentesis :
A. 6th mid clavicular. B. 6th mid axillary. C. 9th mid axillary✅
‫ﻂ‬Ï‫ = ر‬centasis = ‫ﺴﻌﺎ‬â= ‫ ﺳ|ﻨﺘﺎ‬.
🍓Pt came with lymph node enlargement his family concern about it to be cancer because
his son of uncle diagnosis before 3 weeks with lymphoma which think indicated biopsy:
A. It is size above 1 cm B. Came with hepatosplenomegaly
C. Tenderness. D. Fever✅
biopsy indicantion (especially if regional lymphadenopathy persistent >6 wk and/
or constitutional symptoms)Torrnot book.
Constitutional symptoms = fever, chills, night sweats, unexplained weight loss
..
.🍓After mechanical ventilation what will happen: A. hypercapnia B. hypocapnia ✅
🍓Bronchiectasis definitive dx test? HRCT ( High-resolution computed tomography(HRCT)
🍓 Smoker+hemoptysis or hoarseness =bronchogenic ca
🌹Pt came e mom ,, his cousin known case of leukemia Which lymph node enlargement
suspect ca?? Cervical of supraclavicular

🌹Breast mass, lost weight with pleural effusion, most likely?


A. Transudate B.Exudate 🌹✅
‫ﻂ‬Ï‫ = ر‬breast mass = ‫ ب‬EXudate .
🌹Case of a cancerous pt did multiple pleural tap due to pleural effusion with no
improvement, next? Chemical pleurodesis.
🌹 Patient with bronchogenic cancer presents with massive pleural effusion, done pleural
tap several times before, what should u do now?
A. Ultrafiltration, B. chemical pleurodesis C. Diuretics

🍓Patient know case of idiopathic bronchiectasis.. presented c/o progressive


SOB over the past 2 month and dry cough on physical examination decreased airway entry
entry bilaterally with coarse crepitation "S&S of pulmonary fibrosis= give =Predasone .

pneumonia
🌹 A smoker patient presented with ???. O/E he had skin lesions in the back with some of
them being crusted. A CXR was done and showed bilateral infiltrates. How will you
manage? A-antibiotics✅✅ B-steroids C-acyclovir D-antiseptics
🌹2yrs old presented efever and cough and resp distress on exam there is
Bilateral Crackles. X ray showed both lung infiltrate e something I forget what is the
problem? Bronchopneumonia ✅
‫ﻧﻤﻮﻧ~ﺎ ﻣﻌﺮوف •ﻌﻤﻞ ﺻﻮت ﻣﺜﻞ ﻛﺮا™ﻠﺲ‬
🌹treatment of Community-acquired pneumonia? Fluoroquinolones
‫ = ر`ﻂ =ﻧ}ﻤﻮﻧ}ﺎ‬flu= ‫ ﻓﻠﻮﻧﺰا‬.
🌹Old patient develop productive cough , Fever ,Dyspnea On examination crepetion CXR
showed bilateral right middle lobe consolidationThey mention on lab Reticulocyte Cold
agglutination test What is the Dx
A. BA B. Atypical pneumonia C. Streptococcus pneumonia D. Exception of HF
🌹Most common organism causing lobar pneumonia? Strept pneumoniae
🍒 pt develop productive cough and fever given antibiotics he become ok then worsen
(SOB) sx x ray show pleural effusion dx ? A.para pneumonic effusion✅✅ B. TB C. empyema
🌹Patient has a sudden cyanotic episode while a family picnic he was previously healthy, EX
: cough and chest x ray they found trachea shifted to left , what could be the reason of
tracheal shifting :
A. Right middle lobe pneumoniaB. Right lung consolidation C. right lung collapse
...

#Case of lobar pneumonia with moderate effusion what is you mx


A. Ceftriaxone and vancomycin B. Ceftriaxone and vancomycin with thoracocentesis

Long scenario about pt had dialysis 3 day ago central line and now came again for dialysis
peripheral line nurse mentioned he had fever and infection in site of previous dialysis site
what to do ?
A_ culture and iv abx and stop dialysis B_ culture iv abx and remove the center line ✅✅

🌹51 yeard with fever, headache & neck pain= organism =Streptococcus pneumoniae 🌷

Pt with pneumonia coming with hypotension(90/60) and lethargy, pulse was 160 ,T38.5
. Dx A- sepsis B- septic shock✅
...
Pleural friction rub= Differential diagnosis = pneumonia

🌹what time taken for patches on x ray of Lobar pneumonia to disappear?
A.a week B.3 wks C.6 wks D.8 wks
Òç‫ﻂ =ﺳﺘﺔ =د´ﺲ اﺑ‬Ï‫ر‬
🌹 pt admitted to medical word with pneumonia pt is agetited, confused..?
A.put her in dark room alone B.keep the family C.start haloperidol D.start rispridan

🌹 which one of those with lobar pneumonia treated as outpatient


A. 88 ys old Bp 180/ 100 temp 40 RR 20 pulse 116
B. 88 ys Bp 88/60 temp 37 pulse 120 RR 25
C.50 ys Bp 80/60 temp 34 pulse 120 RR 25
D.55 ys BP100/90 ... temp 39.5 pulse 100 RR 30 ✅
‫ﺒﻊ ﺳﻜﻮر اﺳﻤﻪ‬Û‫»ء ﻧ‬¼
Ž ‫ اول‬CURE 65 = ‫ﺪ اول ﺧ~ﺎر–ﻦ‬y‫ﺴﺘﻌ‬2•‫–ﻢ ﻓ‬c‫ وﻋﻨﺪە ﻧ~ﻤﻮﻧ~ﺎ ﻫﺬا ﻻزم ﺗﻨ‬65 ‫–ﺾ ﻓﻮق‬À‫ ف اي ﻣ‬،
‫ﺸﻮف ﺣﺮف‬D ‫»ء‬ ¼ ¸‫ ﺛﺎ‬c ý•‫ •ﻌ‬confustion ‫ و‬R ý•‫ •ﻌ‬RR 30 ‫ ﻣﻦ‬ÒôÙ‫ ﻟﻮ ا‬,
Ž Ž Ž Ž
‫ و‬Urea nitrogen ‫ و‬7 ‫ ﻣﻦ‬ÒôÙ‫ ا‬blood presser ‫ﺴﺎوي ﻧﻘﻄﺔ =اذا „ﺎن‬â ‫ „ﻞ وﺣﺪة ﻓﻴﻬﻢ‬90/60 ‫اذا اﻗﻞ ﻣﻦ‬
• ç ‫ع اﺛﻨ‬c‫ اﻟﻤﺠﻤ‬out patient.
‫ او اﻗﻞ ﻫﺬا •ﻌﺎﻟﺞ ك‬ñ
‫ ﻓﺎﻟﺠﻮاب دي ﻻن ﻓﻘﻂ ﻋﻨﺪە‬RR 30 ‫ف اﺧﺬ ﻧﻘﻄﺔ وﺣﺪة‬
....
🌹Influenza pneumonia case , what to consider when you want to isolate?
A. Airborne B. Droplets C. Contact D. standard

🌹 best antibiotic to treat community acquired pneumonia in healthy adults?


A. -fluroquinolones.✅ B. -vancomycin
‫ﺎ‬S‫ﻤﻮﻧ‬S‫ﺲ ﺟﺎﻟﻪ ﻧ‬x ‫ء‬uv
& ‫ﻪ‬S‫ﺔ ﻣﺎﻓ‬S‫ﺼﺤﺔ وﻋﺎﻓ‬q ‫ﻂ =واﺣﺪ‬n‫ = ر‬flower =fluro ‫ﺎرﺗﻪ ﻻن اول ﻣﺮة ﻳﺘﻌﺐ‬C‫ ﻋﻨﺪ ز‬FG‫ﺐ ﻟﻪ ورد ﻛﺜ‬S‫ﻧﺠ‬

🌹 Pneumonia vaccine in elderly?
B. PCV13 first, followed by a dose of PPSV23✅✅ Òç‫ﺒ‬Ä‫ اﻻول ﺛﻢ اﻟ‬Òç‫اﻟﺮﻗﻢ اﻟﺼﻐ‬
C. PPSV23 first, followed by a dose of PCV13
......
What is reduce mortality In CHF?
A. Diretick. B. ACE ✅
.......
Scenario of diphtheria the diagnosis not mentioned But there is
pseudomembrane over tonsillitis with mild fever What the complication you
suspect? A. pneumonia ✅✅ B.scarlet fever C.glumeroneohrits
‫ﺎ =داﻓﻮر =ﺳﻮدوا‬CFG‫ﻂ =دﻓﺘ‬n‫ ر‬psudo= FG‫ دواﻓ‬FG‫ﻛﺜ‬
‫ﻜﺬﺑﻮا ﻋﺸﺎن دراﺳﺘﻬﻢ اﺛﻨﺎء اﻟˆﻼم‬: tonsilits (‹&‫ﻂ ﻟﻠﺘﺬ‬n‫رﻣﺠﺮد ر‬
.........

🌹68 years old presented with fever, cough with yellowish sputum Oriented to TPP RR was
23, BP was normal Xray showed lobar pneumonia BUN was normal What is your
management?
A. Admit with ABx
B.ABx and discharge with 1 week follow up in the clinic ✅✅
C. Give ABx with 24h monitoring in the ER
‫ ﻻزم ﻣﻀﺎد ﺣﻴﻮي‬، ‫ اﻟﻌﻤﺮ‬ŒŽ • Òç‫ﻛﺒ‬

🌹"Interstitial pneumonitis" is a pathologic hallmark for: Viral pneumonia 🌹
Òç‫ﻂ =اﻧﺘ‬Ï‫ ر‬inter = ‫وس •ﺪﻣﺮﻫﺎ‬Òç‫ﻔ‬u ‫ﺔ‬u‫ﺎﻻﺟﻬﺰة 🔛 اﻻﺟﻬﺰة ﻇﻌﺮزﺿﺔ ﻟﻠﺘﺼﺎ‬u ‫ زر ﻣﻮﺟﻮد‬.

🌹Dyspnea at night, sterna pain Midline mass compressed trachea :


A. Thymoma û‫ اﻟﻤﺮيء واﻻور‬¤‫•ﻀﻐﻂ ﻋ‬
B. Goiter ‫ﻧﺎل‬Òç‫ ﺳﺘ‬ŒŽ • ‫ﺐ اﻟﻢ‬Þ‫ ﻻ ´ﺴ‬C. Lymphoma ‫✅ ﺻﺢ‬
D. Lung cancer ‫ﺎﻟﻮﺳﻂ‬u ‫اﻏﻠﺐ ﻣ•ﺎﻧﻪ ﻣﺎ•ﻜﻮن‬

🌹 Pt complains of SOB and difficult swallowing U/S Midline mass compress


trachea? A. Goiter. B. Thymoma. C. Lymphoma
🌹 Man came with dysphagia and can’t take breath There is swelling in
medstaunm dx? A. Lymphoma. B. Goitre. C. Thymoma
🌹pt intubated after that developed ulcer what type: stress ulcer 🌹
🌹sarcoma how to assess metastasis: A.bone scan B. Ct chest ✅✅ D. Xray
🍒The most common site of distant spread of sarcomas is the lung 🍒
‫ﻂ‬Ï‫ = ر‬sarco = ‫ع‬À‫ق ﺟﻬﺎزك =ﻗﻠﺒﻨﺎ •ﺨﺎف و–ﻨﺨ‬è ‫ق =واﺣﺪ‬è = chest .
.............

🌹indication for thoraccentasesis:


(glucose should be less than 0.6 or ph less than 7.2 or loculated) 🌹
‫ﻂ‬Ï‫ == ر‬centasis = ‫–ﺰ =ﺳﺘﺔ‬Ò• ç ‫ ﺳ|ﻨﺘ‬.
= thorac = 7 ‫ اﻗﻞ ﻣﻦ‬ýŽ • ‫ ﺣﺮوف •ﻌ‬6 ‫ﻋﺪدﻫﻢ‬

🌹features of lung cancer with SVO and ask about what type of lung cancer: Answer is:
non- small cell carcinoma 🌹

💐Scenario about croup, pt having barking cough, diagnosis? laryngotracheobronchitis 💐
…..
TB
🌹Tuberous sclerosis= multiple genetic screening
(( its autosomal dominant pattern of inheritance and can be caused by mutations in the
TSC1 or TSC2 gene .))

🌹Case of T.B asks how to confirm diagnosis: A. Sputum culture ✅ B. Cxr


‫ •ﻜﻮن‬sputum ‫ﺎل‬u ‫ﺸﺨﺺ‬E‫–ﺎ و‬Òç‫ﻜﺘ‬y‫ﺎﻟﻤﺌﺔ ﻳﻮﺿﺢ ﺷ•ﻞ اﻟ‬u ‫ ﻣﺌﺔ‬T.B ‫ ﻻن‬Sputum ‫–ﺎ ﻫﺬە‬Òç‫ﻜﺘ‬y‫ﺎﻟ‬u ‫ﻣﻠ~ﺎااان‬
.....
🌹Patient with cough and hemoptysis. On auscultation: consolidation on the right upper
lung zone. What will help in confirmation of diagnosis?
A. PPD test B_ Sputum culture✅ C. Chest X-ray

🌹child of parents have TB his PDD test is 10mm ? A-positive ✅✅ B-neg'tive
....
🌹TB percoaution = Airborne 🌹
🌹Patient was found to have cavity on x ray, what is the type of precaution? Airborne

🌹Pt with TB. CXR revealed pleural effusion. How to knowledge the effusion is due TB?
A. Pleural protein over serum is < 0.3 B.Pleural LDH > 100 ✅
C. Pleural WBC > 1000 D. Pleural LDH over serum > 1/3
🌹 Indication for thoracentesis or chest tube in pt with pleural effusion:
A. protein to serum < 0.5. B. LDH to serum > 0.6. C. WBC > 1000
‫ “ﻜﻮن‬LDH ‫و‬% 60 ‫ ﻣﻦ‬², é‫ ا‬WBC 1000 ‫ ﻣﻦ‬², é‫ا‬
🍒indication of thoracentesis?
A. Loculated pleural fluid. B. Ph more than 7.20C. Glucose more than 80
🍒Indication of thoracentesis? A. Glucose<60 also high PMN or PH less than 7.35
🍒Pleural effusion in RA ? Answer : glucose less than 1
Patient with ascites, he has a fever and low SAAG?
A. TB ✅✅ B. Cirrhosis ( high SAAG) C. CHD (high SAAG)
🌹Patient with ascites + fever +low SAAG = TB 🌹

Back pain + fever and xray showed calcified right sacroiliac joint, what's the most
appropriate test to order? a) PPD skin test b) Tube agglutination test
….
💐 patient with TB on drugs development paresthesia, treatment?
A. pyridoxine ✅✅ B. Thiamine C. B12

Which mother can give breastfeeding to her son?
A-direct breastfeeding HIV mother ‫ ا‬B-expressed milk HIV mother
C-direct breastfeeding TB mother D-expressed milk TB mother ✅✅

Patient k/c of congestive heart disease and HTN on ACEI and Warfarin was diagnosed with
TB and started on 4 regimen what is true:
A/Increase Warfarin✅✅
B/Rifampicin should be stopped
Rifampicin may reduce the effects of warfarin Need to increase the dose.
............
TB patient he start to have some priks in the finger tips which been incresing what to do?
A-stop the medication B-give him pyridoxine ✅ ✔ C-give him cobalamin
….
Known case of TB after 3 months of starting antiTB medication there is elevation in LFT
what to do?
A-stop INH B-stop ethambutol C-stop rifampicin D-stop all drugs ✅✅

Patient on TB medications, comes for follow up after 3 weeks and shows
abnormal LFT, negative AFB, and resolution of TB symptoms, what to do?
A-Stop all drugs and repeat LFT in a week ✅✅
…..
💐TB medications has an increase in uric acid = Pyrazinamide 💐
‫ﻂ‬Ï‫ = ر‬pyrazain = ‫ران •ﺤﺘﺎج ﻟﻬﻢ (أﺳﺪ )•ﺨﻮﻓﻬﻢ ﻋﺸﺎن ´ﺴﻜﺘﻮا‬Ò• £ ‫ ( ﺑﺰران =اﻟ‬Acid ) ‫–ﺮة ﺻﺢ‬è¼ ، ‫😂اﺳﺪ‬
... ..... ..
a house worker came from South Africa with bloody cough and x-ray showed cavitation
what will you do next ?
A) Give the Anti TB Med B) Do acid fact bacilli ✅

Patient post TB bronchiectasis presents with cough, hemoptysis. CXR shows right upper
lobe cavitary lesions. What is the best *initial* next step?
A- Place patient in right lateral decubitus position B- Chest physiotherapy ✅ ✔
….
🍒They asked also about pulmonary HTN grade 3 class = Fibrosis 🍒

🌹TB pt on meds, developed paresthesia.. what to give him?Pyridoxine
….
🍒 the type of lung cancer causing superior vena cava obstruction? Non small
cell carcinoma
🌹 Facial plethora and JVP distended, which of the following is associated
with this condition? SLCC
🍓 Case that seemed like obstructive sleep apnea. Obese patient with uncomfortable sleep
+ snoring. Has cyanosis and dyspnea best investigation= polysomnography
🍓 Sleep apnea: C-pap
🍒Patient with RA has increasing SOB and dry cough, cyanosis, clubbing, bibasal fine
crepitations = idiopathic pulmonary fibrosis🍒
🌹Patient has restrictive pattern, diagnosis? Lung fibrosis

🌹40 years old come to ER complain of SOB . He cigarette 20/day but otherwise
well. Spirometry show restrictive picture.DX A. pulmonary fibrosis B. asthms

🍒🍒 Neonatal get pneumonia type of immunity = Active natural immunity🍒🍒


💐smoker since 20 years presented with dry cough and sob ?
A. Interstitial fibrosis ✅✅ 💐 B. broncoetsis (prouductive cagh not dry)
.......
🌹Superior vena cava syndrome (SVCS) most common causesd wich lung cancer ?
Non-small cell lung carinoma 🌹

🌹 Small lung Ca (forget if he has pain or not ) he will die before one year, ca is 3.5 normal is
1.1 what u will do for Him A. Iv normal saline. B. Iv dextrose. C. Sc Morphine(if he has pain)

🌹 Most common lung CA causes SVC syndrome :


A. – NON- small lung CA ✅✅ B. - squamous CA (SVC = small)
🌹Small lung Ca complaining of back pain for 12 h neurological exam is
NORMAL what most appropriate next step
A. MRI spine. B. Steroid. C. MRI + iv steroid D. Not need urgent management

🌹scenario of elderly patient who had chest inf 4 weeks ago, initially improved on antBx,
now worsened again, CXR: right pleural effusion.Dx?
A. Lung cancer. B. TB. C. parapneumonic effusion

#Primary prevention for TB?
A.Chemoprophylaxis B.vaccination C.Health education D. Treatment of exposure by ant tb
drugs.

🌹A case about a pt with pleural effusion and asks about the most
accurate test to detect TB: A. -ADA level >35. B. -AFB culture
🌹Pnuemonia symptoms and with meduim Plueral effusion :!@
A. Abx with thoracosentesis. B. Abx alone. C. Abx with acetmavir
🍒patient with s/s of pneumonia, rt. Side dullness on percussions and decreased air entry,
CXR showed middle and lower rt lobe consolidation with pleural effusion, dx?
pneomonia with parapneomonic effusion👍
🍒similar question but asking about management: IV abx + thoracocentesis✅
🍒Patient came with Fever, dyspnea, purulent cough yellow sputum , hemoptysis,
examination, marked dullness half of the right chest and decreased air entry also half of
the chest X ray: consolidation in middle and lower lobe, and moderate pleural effusion.
Best Management ?
A.IV ceftriaxone and metronidazole
B.IV, ceftriaxone and metronidazole and thoracocentesis ✅
C.Steroid
….
which of the following pneumoconiosis associated with TB or predispose to TB something
like this and the choices were : A. Silicosis B. Farmer lung. C. Asbestosis
….
Anti TB cause hyperuricemia? Pyrazinamide

Plural effusion case and has high interferon gamma dx?
A. Plural TB B. Pneumonia. C. Lung cancer. D. ILD
…..
Pt with TB. CXR revealed pleural effusion. How to knowledge the effusion is due TB?
A. Pleural protein over serum is < 0.3 B. Pleural LDH > 100
C. Pleural WBC > 1000 D. Pleural LDH over serum > 1/3
🌹Middle age male complains of SOB, CXR evidence of pleural effusion. Aspiration of fluid
to analysis was done. What statement of the following confirm that the pleural effusion is
exudative?
A. Plural protein/serum protein is LESS than 0.5
B. Plural pLDH/ serum LDH is MORE than 0.6
C. LDH is more the 1/3 of upper limit ..
.....

pt suspected Tuberculous pleural effusion , what is the sensitive test that support
your diagnosis :
A. AFB of pleural effusion B. High lymphocytes in pleural effusion
C. Low glucose in pleural effusion D. Elevated pleural fluid ADA levels
..
🌹Patient was diagnosed with pulmonary TB , and started 4 drugs for the treatment, he has
family history of ureteral stone. Patient presented with red urine. Lab ( slightly decreased
HB and PLT), What is the possible cause?
A. side effect of medication ✅✅ B. ureteral stone C. effect of TB on blood
🌹Elderly in ER with s&s of TB .. What is the initial action ? Isolation ‫»ء ﻧﻌﺰﻟﻪ‬¼
Ž ‫🌹 اول‬
‫ﻂ‬Ï‫ = ر‬TB ‫ﻌﺪە‬y‫ﺎ🤬 =ﺟﺎﻟﺲ ﻳﺘﻬﺎوش ﻣﻊ اﻟﻨﺎس ﻻزم ﻧﻌﺰﻟﻪ وﻧ‬y‫¸ =ﺗ‬Ž £ ¸Ž Ó .
.............
🌹pt on anti tb drug , has increased in liver enzyme after treatment , what should you do ?
stop all if more than 3 fold
.......
🌹pt has in x ray findings a cave in upper lung how this can transfer?
airborne because cavities in upper lungs = TB
........
🍓Long scenario regarding paitent with suspecting pulmonary TB effusion
confirming the diagnosis?
A. positive AFB culture in pleural fluid ✅ (acid-fast bacteria AFB) . B. lymphocytes in
pleural fluid
.........
🍓TB treatment and the drug cause High LFT = pyrazinamide is the most hepatotoxic.

🍓Anti TB drug cause neuropathy
Isoniazid = ‫ = ر`ﻂ‬isolantion= iso ‫ﺘﺌﺎب‬é‫ﻋﺰل =اﻟﻌﺰل واﻟﻮﺣﺪة “ﺠ}ﺐ ﻣﺮض ﻟﻼﻋﺼﺎب واﻻ‬
........
🍓x ray of apical lesion and diagnosis? TB

● pleural TB most sensitive test:
A. Adenosine deaminase(ADA)= Sensitivity ‫ﻂ =ادﻳﻦ =دﻳﻦ =اﻟﺪﻳﻦ ﺟﺪا ﺣﺴﺎس ﻣﺎﺣﺪ ﻳﺘ•ﻠﻢ ﻓ~ﻪ‬Ï‫ر‬
B. AFB =Specificty
….
🍓Pt with suspected Tb pleural effusion which is the most sensitive:
A. + positive AF culture B. Low glucose C. High adenosine more than 45 IU ‫دﻳﻦ =ﺣﺴﺎس‬

🌹Pt with previous TB and bronchiectasis came now with severe hemoptysis cxr show
🌹cavity of upper lobe = A. Chest physiotherapy✅ B. Pneumonectomy
🌹bronchiectasis case .. best diagnostic test: A- CT scan ✅✅✅. B- Bronchoscope.
🌹45 years old male patient smoker c/o dyspnea, hemoptysis, clubbing and other
features suggest bronchiectasis. What is the height diagnostic test of value?
A. CXR B. Bronchoscope C. high resolution CT

...........
🍒 TB with Plura effustion wich sensitive finding in tapping ?
A. LDH✅. B. ADH
🍒 Plura effustion analysis show interferon gamma Dx?
A. TB✅. B. mesothelioma. C. para pneumotic. D. empyema
🍒pt RA with Plura effustion which of following typical for RA ?
A. transudate. B. high PH. C. glucose less than 1✅. D. haemorrhage
will cause exudative effusion -> low glucose and pH
🍒Which of flowing risk for TB? A-Silicosis ✅ B-Asbestosis

🍒Before starting ant tb meds what to order =LFT

🌹 Pt on TB medication present with hyperuricemia and ask about the drug that cause it ?
Pyrazinamide = ‫ﺎﻳﺮاز‬u= ‫ﻂ‬Ï‫ ر‬pyraz = ‫ = ﺑﺮاز‬uric= urin .
….
🌹another tb q ask what to do firs= Isolation 🌹
......
🌹Another tb q about mode of transfusion:= . Airborne 🌹
.....
🌹And another tb q and what's the management: = RIPE( the four drugs) .
= rifamycin isoniazid, pyrazinamide, and ethambutol.
... ....
🌹And another tb with parents have tb and child ppd is 10mm= Positive 🌹
..
🌹Indian with history of Tb presented with elevated JVP and hepatosplenomegaly:
. Constrictive pericarditis
.....
🌹Patient diagnosed with TB and stared on anti tb medications, which of the following
might increase his creatinine? A. -Isoniazid B. -rifampin✅ C. -ethambutol D. -pyrazinamide
......
🌹Indian patient known case of TB, he has tb bronchiectasis, came with fever, dyspnea
and massive hemoptysis, x-ray: right side infiltrate, What’s the best next appropriate
management? A.Pneumonectomy B.Chest tube
C.lateral decubitus position on the site of healthier lung.
....
🌹patient diagnosed with TB, What is the most important thing to do before starting
treatment: A. start B6 B. Do LFT
....
🌹TB How to Diagnose: Sputum culture

🌹 TB pt what is the medication increase liver enzymes ?! INH
..
🌹Patient with Signs of Symptoms of TB but they didn’t give the diagnosis, and he had
pleural effusion, what is the cause of his pleural effusion:Tubercles Pleural Effusion

🌹 Interstitial pneumonitis is characteristic for what?
A. Viral pneumonia B. Bacterial pneumonia C. TB D. Bronchopneumonia
...
Patient on ACEI,BB,CCB “I forgot the name of medication”and warfarin , dx with TB, on
quadra medication, what you will do:
A. Stop ACEI B. Increase dose of warfarin C. decrease dose of rifampicin
<
‫ ﻻن دواء‬Rifampicin ‫; ﻋﻼج‬: ‫ﺴﺘﻌﻤﻞ‬š 9:‫ ا‬T.B ‫&ﻦ ﻓﻨﺤﺘﺎج ﻧﺰود اﻟﺠﺮﻋﺔ‬%‫ﻘﻠﻞ ﻣﻦ ﻛﻔﺎﺋﺔ دواء اﻟﻮارﻓ‬2 .
....
🌹pleural fluid analysis shows exudative fluid witn high gamma interferon level
dx? A. empyema B. mesothelioma C. TB D. paranumatic effusion
...
🌹diagnostic invx. for Trachomalsia? brochioscopy.🌹

Case of smoker with 3 months hx of dyspnea and productive cough
hemoptysis OE course crackles : Which have highest diagnostic value
A. PFT. B. High resolution CTÖ

1Renal disorder
🌷 Pt with bilateral hydronephrosis with dilated bladder and proximal
urethra? A. Posterior urethral valve B. Uretropelvic occlosion

🌹 50’s Patient with intermittent weakness of urination for months. Normal


psa and urine analysis. Examination of prostate: median lobe enlargement. Best
management: A. Periodical psa (best relative). B. Renal ultrasound annually!@

🌹the best way to take sample of urine in UTI = A-Midstream sample ✅✅ B-24 hour ..
🌹Pt with aspirin toxicity, what is the most important next step in management:
A. activated charcoal B. urine alkalinization
🌹 Case of pyelonephritis + Nitrate in UA, what the best next step?
A. Oral Abx B. Iv Abx C. US of urinary

🍓Pt with bloody urine no symptoms what to do? A. US B. Urine analysis ✅


🍒Young pt has recurrent UTI, stones and electrolytes disturbance on imaging he had
bilateral kidney enlargement with multiple variable size thin walls cyst throughout the
parenchyma is the diagnosis? A-Medullary sponge disease B-Polycystic kidney disease ✅
🌹To say it’s UTI , which of these more accurate in Urinanalysis = A. Nitrate B. Leukocyte
……
🌹Pt k/c of chronic kidney disease, presented with numbness and dec
vibration sensation, absent LL reflexes. On labs: high Cr, high K , MCV was low, whats most
appropriate management? A. Hemodialysis B. Vit B12

🍓Indication for dialysis : Hyperkalemia ✅


Uremic perfcarditis, uremic encephalopathy, refractory hyperkalemia, refractory
metabolic acidosis.
Which of the following indicate heart Diseas in AKI patients
A- refractory 7.5 hyperkalemia not responding to 3 medical
B- another GFR <5
🌹CKD, no hx of dialysis mentioned, complain of tingling and numbness.Examination:
absent reflexes,Labs, slight anemic, potassium 5.5 (NUL 5.1) urea and Creatinine very high
What is the most appropriate next step in management?
A,Erythropoietin. B.Oral bicarb. C. Vit B complex.D. Initiation of dialysis.✅✅

🌹Foley catheter for 4 days + pt took paracetamol and codeine, came later with fever and
the nurse noted a cloudy urine what is the cause? A. Codeine B.Foley catheter ✅✅
🌹Patient on foley catheter developed cloudy urine, how to prevent such events?
Daily check up for need of new foley catheter
🌹when we say this pt have ( resisitent to steroid in nephrotic syndrom ??
When 4 weeks with daily steroid and no response ) 🌹
)
‫ اي‬W% ‫ﺪ وﻣﺎ‬6‫و‬AB‫ ـﻊ اﺳﺎﺑﻴﻊ ﻣﺘﻮاﺻﻠﺔ ﺳﺘ‬Q‫ﺾ اﺧﺬ ار‬65‫ ﻟﻤﺎ &ﻜﻮن اﻟﻤ‬،‫ﺐ ﻓﺎ&ﺪة؟؟‬F‫ﺪ وﻣﻮ ﺟﺎﻟﺲ &ﺠ‬6‫و‬AB‫ﺾ ﻋﻨﺪە ﻣﻘﺎوﻣﺔ ﻟﻠﺪواء اﻟﺴﺘ‬65‫( ﻧﻘﻮل اﻧﻪ اﻟﻤ‬,‫( ﻣ‬% ) ‫&ﻌ‬
‫ﻢ‬f‫ﻄ‬Q5‫ﺎرات ﻣﻤﻜﻦ ﺗﺨ‬F‫ﻻن اﺧﺘ‬. ‫ ـﻊ اﺳﺎﺑﻴﻊ‬Q‫ﺎ ار‬b‫ اﺣﻔﻈﻮوو‬،. ‫ﺔ وﻻ ﺗﺤﺴﻦ‬Y‫ اﺳﺘﺠﺎ‬.
🍓nephrotic and asking about which common type Answer is =
Minimal change > Pediatric FSGN > Adult
🍓Pt with nephrotic syndrome taking medication how can you know that the disease
remission ? A. Disappear Albumin in urine✅ B. Disappear of true edema
🌹Nephrotic syndrome first line Tx?Steroid 🌹
🌹Nephrotic pt, which of the following will decrease proteinuria = A. ACEI. B. prednisone✅
🌹Long case about a patient with edema Urine analysis: Oval fat body, some hyaline cast.
Occasional RBC case. B. Nephritic syndrome. C. NephrOtic syndrome. ✅✅
NephrOtic= Oval == ‫ﺣﺮف‬O ¦ ƒ § ‫ = ﺳﻤ‬Fat
🌹When to consider a nephrotic syndrome is a cortisone resistant:
4 weeks of daily cortisone
%
‫ون‬F% G ‫ﻪ اﻧﻪ ﺣﺼﻞ ﻣﻘﺎوﻣﺔ ﻟﻠ—ﻮرﺗ‬S‫ﺪا ﻫﻨﺎ •ﺴﻤ‬q‫ﺔ ا‬q‫ اﺳﺘﺠﺎ‬$& ‫ون وﻣﺎ‬F% G ‫ ـﻊ اﺳﺎﺑﻴﻊ ﻛﻮرﺗ‬n‫ اذا اﻋﺪﻳﺘﻪ ار‬، ‫ ـﻊ اﺳﺎﺑﻴﻊ وﻟ‘ﺲ ﺷﻬﻮر‬n‫ﺘ•ﻪ اﻧﻪ ار‬Œ‫ﻧ‬
🌹 Typical case on nephrotic syndrome ask about highly diagnostic test =
A. . Serum albumin B. . Us. C. . Urine total protein✅ D. . Electrolytes

🌹Pt with renal impairment with hyperkalemia 6.5 . = calcium gloconate


Òç‫ „ﻼﻣﻬﺎ ﻛﺜ‬hyperkalemia = ‫ ﺧﻼص‬calciu ýŽ Ó ‫اﺳﻜ‬
🍓how to prevent calcium stone? A. Give him thiazides✅ B. Decrease Ca intake

🍒 Chronic kidney disease pt asking what food to avoid becouse of High K


Answer is : tomato , banana …. ✅✅

nephrotic syndrome, with edema , doctor advice low NA diet .. what you will give also ?
a) Steroid ✅ b) Furosemide
………
🌹pyelonephritis case what ttt ?Admission and IV Abx 🌹
……
🌹pregnant with blood pressure 160/110 with protein in urine what first thing to give ?
Magnesium sulfate ( pre.eclamsia)🌹
.....
Male has incontinence and has a history of difficulty in starting urinary stream What type
of incontinence? Over flow ✅. Stress Urge
🌹difficulty in starting urinary stream = Over flow incontinence🌹
.......
🍓stress incontinence =kegel exercise 🍓


....
🌹Pt with urine leaking and can't control stream, bladder distend after voiding?
A. stress incontinence. B. overflow incontinence
C. Reflex incontinence D. urge incontinence

🍒Initial management of nephrotic syndrome? steroid and for symptomatic >>


furosemide🍒
.................
🍒Old male on 3 antihypertensive , uncontrolled ,asymmetrical kidneys.= Renal artery
stenosis🍒
🌹Pt on 4 antihypertensive medication, Us shows asymmetrical kidney, dx:
A. Pheochromocytoma. B. renal stenosis.✅ C. Adult polycystic kidney disease

pediatric patient with hematuria and hemoptysis? Goodpasture's syndrome


….
🌹Young Pateint with fever and dark urine has history of constipation , urinalysis:
high leukocyte, high erythrocytes, high protein :
A. pyelonephritis✅ ( (fever, chills, pyuria, vomiting + flank pain + hx of UTI ) + look for
pyuria as it the most common finding in patientsacute pyelonephritis
Glomuronephritia
🌹Progressive Glomuronephritia pathology🌹 = crescent moon🌜🌜🌜
‫ﻂ‬Ï‫ = ر‬glomo ‫ ﺟﻠﻮﻣﻮا =ﻧﺠﻮم =ﻣﻊ ﻗﻤﺮ‬moon .
🌹Adult with URTI 3 weeks ago followed by renal signs and labs :
A. IgA glume. B. Poststreptococcal glomerulonephritis
🌹Case with hematuria and RBC cast on urine analysis , recent hx of URTI . which inv
support dx of PSGN ? low c3
🍓pt with lymphadenopathy, and membranous over the tonsil What is the complication :
A. pneumonia B. Scarlet fever✅ C. glomerulonephritis
🍓child with multiple joint pain and subcutaneous nodules ask about another ringing
support your dx? A- anti ASO cus it is Rh ✅✅‫ ( رﺑط‬ASO = ‫ ﺣرف‬O ‫ ﻣﻊ‬N*o*dule ).....
Anti o is: Antistreptolysin O (ASO) is an antibody targeted against streptolysin O, a toxic
enzyme produced by group A Streptococcus bacteria. )

🌹 Q about felty’s syndrome symptoms arthritis and splenomegaly ask about


diagnosis. = felty’s syndrome
🌹Female pt asymptomatic, previous Hx of URTI 2 weeks ago. O/E: mid diastolic murmur,
how to confirm dx? A. Echo B. ASO titre

🍒 URTI resolved within weeks and symptoms of nephrotic syndrome ?PSGN


🌹PSGN, more invg= A. urine culture B. serum creatinin C. abd ultrasound D. abd xray

🌹Male pt with hx of right knee pain and swelling, left ankle pain and swelling, for
6 month. Previous hx of UTI 2weeks prior to symptoms, what to give?
A. methotrexate B. cyclosporin C. Sulfasalazine for(RA) D. Azithromycin

1Headache
🍓Clear case with headache that is thropining and unilateral aggravating with light and
movement : migraines .
🌹frontal sinus headache how to reach the diagnosis.
A. -full history and examination✅ B. -CT sinuses ‫ﺎﻟﺘﻔﺼ~ﻞ اﻟﻬﺴﺘﻮري‬u ‫»ء‬¼
Ž ‫ﻻزم اﻋﺮف اول‬

🌹Female, with 6 months headache, unilateral, mild relieved to NSAID, now came with
nausea and vomiting, neuro and fundus examination normal, how to reach diagnosis?
A.CT B.MRI. C.Electroencephalogram D.Hx and examination l✅
........
🍓Patient with acute migraine What is the treatment? Triptan
‫ﺘﻮن‬é‫ب ﺷﺎي ﻟﻴ‬è¼‫ﻂ ﺻﺪاع ﺷﺪﻳ|ﻴ~ﺪ =ا‬Ï‫ ر‬. Triptan .
🌹 patient experience the worst headache this morning after trying to pick up his keys =
Subarachnoid hemorrhage 🌹 ‫ ﺻﺪاع ﻗﻮي ﺟﺪا‬، ‫ﻠﻬﺎ‬¢ ¨6 © ‫ﺎ‬#‫„ ﺣ‬6 4 ‫ﻘﻮﻟﻚ ﺣﺴ ﺖ ب اﺳﻮء ﺻﺪاع ﻣﺮ‬7 ، ‫ﻒ اﻟﻤﺦ‬ž‹4 4 ‫ ﻟ‬£‫ﻠﻤﺔ اﻟ‬¢ .
🍒 There was a tricky Q about pt develop sudden headache when he was bending to take
his keys from the ground, describe it as severe headache in his life which is the most likely
Dx? A. subarachnoid Hemorrhage✅ B. cluster headache .
🌹 A young female presents with unilateral throbbing headache. Her headache is associated
with nausea and vomiting. She tells you that she is sensitive to light. Which of the following
is used for acute treatment?A. Aspirin B. Triptan✅ C. B-Blocker D. O2

🌹headache in child band like , throbbing , with stress at school dx = Tension headch 🌹
.........
headache at 3 am with eye symptoms what is best prophylaxis ? Verapamil ✅
( Cluster headache = Verapamil = this drug is its calcium channek block , it can treat this
type of headch)
‫ ﻣﻨﻄﻘﺔ وﺣﺪة‬ŒŽ • ¢ • • • •
Ž £ • ‫ اﻟﺴﺪاع‬ýŽ ‫ وﺣﺪة •ﻌ‬ñç‫ﺎ ﻋ‬y‫ ﻣﺪﻣﻌﺔ وﺣﻤﺮاء وﻏﺎﻟ‬ñç‫ ﺗﻜﻮن اﻟﻌ‬ñç‫ اﻟﻌ‬¤‫ •ﺎﺛﺮ ﻋ‬، ‫ﻓ~ﻪ ﺻﺪاع ﻣﻦ ﻗﻮﺗﻪ‬
‫؟‬ñ• ç ‫ •ﺠ~ﻚ ﻣﻌﻪ اﻋﺮاض ﺗﺼ¨ﺐ اﻟﻌ‬Ð‫ ا´ﺶ اﺳﻢ اﻟﺼﺪاع ﻫﺬا ا‬، ñ • ç ‫ ﺟﻨﺐ اﻟﺮأس وﻧﺎزل ﻟﻠﻌ‬¤‫ﻋ‬
Ž
Cluster headache .
🌹headache at 3 am with eye symptoms what is best prophylaxis=Cluster headache
=Verapamil = its calcium channek block 🌹 ÒÓ‫ﺎﻣ~ﻞ =ﻛﺮاﻣ~ﻼ و„ﺎﺳ‬u‫ا‬Òç‫ =ﻓ‬Òç‫ =„ﺎﺳﺘ‬Òç‫„ﻼﺳﺘ‬.
🌹Pt came with headache for 2 days 3 times in months between eyes (cluster headache)
A. ct B. Carfual history ✅

.......

🍒Acute management of migraine headache ? A. Triptan B. Aspirin


mild to moderate > Aspirin. Severe > Triptan🍒

Adolescent with headaches in frontal sinuses region Next step?
A. Paranasal sinuses CT B. Detail Hx and Examination ✅
‫»ء ﻳﻨﻔﻊ ﻣﻌﻪ‬¼ •
Ž ŒŽ ‫ و„ﻞ ﻣﺮة اﻟﺼﺪاع ﻣﺮة ﻳ—–ﺪ وﻣﺎ‬Òç‫ ﻟﻮ ﻗﺎﻟﻚ اﻧﻪ اﺳﺘﺨﺪم ادو–ﺔ ﻛﺜ‬، ¸Ž £ ‫ﺲ ﻛﺬا ﻧﺨﺘﺎر‬7 ‫ﻟﻮ اﻟﺴﺆوال‬
‫ اﺧﺘﺎروا‬A ‫ﺪ اورام او‬y‫ﺴﺘﻌ‬D ýÓ‫ ﺣ‬Infection
......

Inflammatory bowel disease ( crhons + ulceritve colitis )


🌹16 years old female. Fever and Chronic diarrhea for 10 months Post meal Paraumbilical
pain? - Crohnʼs disease ✅ ✔. B- Celiac
***Crohnʼs disease = Fever and Chronic diarrhea + abdominal pain .**
(Common Crohn's disease symptom: Unexplained weight loss. Fever. Abdominal pain and
cramping. Fatigue and a feeling of low energy.)
🌹Pt had IBD flare , had been managed with IV fluid, what to do:
A. Oral budesonide ‫ون •ﻘﻠﻞ اﻻﻟﺘﻬﺎب‬Ò• ç ‫ﻮرﺗ‬Ä‫ اﻟ‬B.Iv methylprednisolone
🌹Young male known with Inflammatory bowel disease, resection of the lesion
done with ** something not sure** ( fat-soluble supplements) ‫ﺎﺧﺪ‬S‫ﺾ ﻓﯿﺘﺎﻣﯿﻨﺎت ﺑ‬Qc‫ﺗﻌ‬
‫ ﻟﻠﻤ•ﺎن‬ÐŽ‫ﺌﺼﺎﻟﻪ ﺗﻢ ا‬Û‫ اﻣﺘﺼﺎﺻﻪ ﯾﺘﻢ ﻣﺎرح ﻻﻧﻪ اﺳ‬.. he q is which site of the gut was resected?
Termenal ileum

🌹Answer is :Crohn's disease case with ilium 1 stricture in and ultrasound: how
to manage ? Management of crohn’s strictures
Single <5 cm without any complications > endoscopic dilatation.
Long ≥5 cm, multiple, diffuse or recurrent > Strictureplasty.
Complicated or multiple strictures within a short segment of the bowel, in ileocaecal
junction> Small bowel resection

🌹Crohn’s disease with stricture management? Strictureplasty ✅🌹


🌹Crohn's disease and have stricture it was single 1 cm , 1cm from the ileocecal
valve = strictureplasty

🌹One come for consultation he is a smoker he said his brother have crohn's disease he is
afraid of he’s chance of having it ; what’s the risk factor:
smoking increases the chance of having it and smoker protective in UC
🌹 Pt has chron illeocecal since 3 months he started on steroid and pentasa from 3 weeks,
he is complaining of anal discharge and fistula what is the tx u wanna give him
A. Methotrexate B. Azathioprine C. Infliximab

🌹crohn's diseases patient with perianal abscess on infliximab and azithromycin. Mx?
A.Increase infliximab dose and decrease frequency B.IV antibiotics ✅✅ like =Cipro,
metro. C. perianal swab
🌹extraintestinal symptom of crohn Disease :
A- Erythema nodosum B- Erythema gangrenosum C- Erythema marginatum

🍒Pt that is taking steroid for her IBD crohn came with bilious vomiting abdominal
distention ,abdominal pain and tenderness in the RLQ area on imaging she has single
stricture I cm proximal ileocecal valve what you will do? ==crons strictuur (Resection) .

🍒 crohn came with bilateral anterior shin raised tender nodules = Erythema nodosum
… ‫ = ﻛﺮون =ﻣﻜﺮوﻧﺔ‬nodosm = ‫🍝🍝🍝 ﻧﻮدل‬
🍒🍒weight loss over 2 months. Ilieal biopsy showed noncaseating granuloma = Crohn
..
🍒tender lesions on shin .....with what? A. coelia B. crohn✅erythema nodosum”
🌹 Women who has progressive Dyspnea and fatigue, on auscultation she has fine
crackles, while examining her legs she has tender red induration on her shin . What is the
best next step? A. Oral steroid B. Chest x ray ✅
Erythema nodosum= characterized by inflammation of the fat cells under the skin, resulting
in tender red nodules or lumps that are usually seen on both shins.
🍓 Case of bloody diarrhea biopsy shows transmural inflammation where is the most
common site ? ileum
🍓 common site for Crohn’s disease? ileum colon
🍓 patient with abd. pain, fever, endoscopy showed ulceration transmural but in patches
(not continuous), the lesion most likely affecting which part of GI :
A- jeujenum. B- ileum ✅✅== crohns C- colon D- Rectum
cronhs ......‫ اول ﻣﺎ ﺗﺷوﻓوﻧﮭﺎ اﻋرﻓوا ﻋﻠﻰ طول ﺗﺷﺧﯾص اﻟﻣرض ھو‬patches* lestion* ‫ﻧرﻛز ﺑﺎﻟﺳؤوال ﯾﻘوﻟك‬

🍓 Male with non bloody dirrhea, pale, oral ulcer? This is the exact question with no extra
info! A. Crohns = Oral ulcer > Crhons B. ilcerative C. IBS D. Celiac
‫ح {ﺎﻟﻔﻢ‬v‫ﻪ =ﻗ‬wv‫ ر`ﻂ =ﻛﺮون =ﻛ‬.
🍒Pt known case of crohn's did terminal ileum resection complain of diarrhea what to give?
A.B12✅✅ B.Prednisone. C.Azathioprine
Etiology Vitamin B12 Deficiency = crohn's.torrnot book.
🍓 Risk factor of crohn's disease? A. smoking B. being male
🌹Crohn's disease, the most common site ? A. Was written ileocolic not ileocecal “it is the same”
🌹Patient came with diarrhea in biopsy it is transmoral ask dx = Crohns
🌹 Poorly controlled diabetic patient care with right sided eye ptosis and difficulty
in adduction and elevation of Single stricture in terminal ileum 1 cm away from
ileocecal valve mange? A. stricturoplasty B. resection with ileostomy
🌹 -pt have chron and Anemia feature was? Vit B12

🌹lead pip point on X.ray = ulceritive colitis 🌹

🌹What increases risk of colorectal cancer in UC?


Disease duration and extent + age at onset of UC + primary sclerosing cholangitis + age
below 15 years at onset as an independent risk factor for cancer 🌹
🌹Fistula in crohn not taking any medications : Infliximab 🌹
🌹Patient has fatigue and pallor, Hx of chronic diarrhoea laps showed microcytic anemia +
low B12 ? A- Celiac disease B- UC C- crohn disease
🌹Patient with crohs, developed fistula between ileum and jejunum I how to treat?
A-Open drainage and fistulectomy. B-Percutaneous drainage C_simple fistulectomy in
general ✅✅

🌹Ulcerative colitis ercp done shows intra and extra hepatic bile duct strictures. What is the
dx? A. Primary sclerosing cholangitis B. primary biliary cirrhosis C. choledocal cyst
🌹Ulceritive colitis , investigations : A- TFT B- CT abdomen C- Celiac disease
🌹One had diarrhea and he lost wt becoz of that it was for 3 months Otherwise he is free U
would give him ? A. Anti biotic B. Anti diarrhea. C. Ante parasitic
🌹Young male patient care with chronic bloody diarrhea, joint pain and skin rash
what’s the most likely diagnosis? A. Crohn's B. UC C. IBS D. Enteric fever
🌹Pt With hx of UC , developed single stricture in 1cm proximal to iliocecal valve how to
Mx? - Right hemicolectomy - Observe - Resection of stricture✅
🌹- patient with history and physical examination of Ulcerative colitis asking about
common site of UC? A-Ilium B-Jejunum C-Rectum

🍓IBS ‫🍓 اﻟﻘوﻟون اﻟﻌﺻﺑﻲ‬


🌹A case of typical of a male with IBS presentation ( abdominal pain somewhat relieved
with defecation, frequent loose stool, feeling of incomplete emptying, no loss of weights)
on abdominal examination: normal How to confirm diagnosis:
A. thyroid function test B. celiac disease test C. abdominal CT
D- diagnosed clinically ‫ ﻣﺎ“ﺤﺘﺎج ﻓﺤﻮﺻﺎت “ﻜﻮن واﺿﺢ ﻣﻦ اﻻﻋﺮاض‬îo ÿ ‫اﻟﻘﻮﻟﻮن اﻟﻌﺼ‬
🌹26 yrs male patient with crampy abdominal pain not related to specific food with
diarrhea alternating with constipation ??
A. irritable bowel✅. B. chrons. C. lactose intolerance
🌹 history of irritable bowel
A. avoid lantini ✅. B. eat the fruit. C. eat coconut oil
🍓IBS case taking Loperamide and another medication her symptoms not
improved what is most appropriate management :
A. cognitive behavioral therapy B. Psychotherapy C. Tricyclic antidepressants✅
🌹IBS case ask about best advice ? increase fruit ingestion
🌹IBS case, how to manage? A-high Fiber diet ✅ ✔ B-tricyclic antidepressant
N.B: first diet control; High fiber diet, increase fruits and vegetables, if not improving go to
medical ttt and TCA.
🌹 Patient talking to aliens ttt? A. Antidepressants. B. Behavioral therapy. C. Antipsychotics
🌹 Teenager girl, she had multiple panic attack about death she scared of
dying : A. Agoraphobia. B. panic attack

……

1Autoimmune disease
🌷The following is a contraindication to IVIG A.Hypernatremia B.Anemia C.Increased CRP
🌹female patient with *positive Anticardiolipin antibody* and other symptoms, what is
the diagnosis? Antiphospholipid syndrome ✅ ✔
‫ ﻟﻣﺎ اﺟﻲ اذاﻛر ﺷﺎﺑﺗر اﻟﻘﻠب ﻻزم اﺟﯾب ﻣﻌﮫ ﻓﺳﻔس ﻋﺷﺎن اﺗﺳﻠﻰ واﻧﺎ اذاﻛر‬anti phospho ) ‫ ( و‬anti cardio ‫رﺑط‬
...... ..... ....‫ ف ﻧﺟﯾب ﻓﺳﻔس‬، ‫ﻻن ﺷﺎﺑﺗر اﻟﻛﺎردﯾوا ﺻﻌب وﯾﺑﻐﺎﻟﮫ ﺷﻲء ﯾﺳﻠﻲ‬
🌹Whats indicate poor IVIG response?
A. Anemia B. Neutropenia C. High CRP✅ D. Hypernatremia
🌹 –Guillain Barré syndrome trat by = intravenus immunoglobulin (not A- plasma
exchange or plasmapherases🌷
🌹 Pt with ascending limb weakness with history of gastroenteritis 3 weeks back
“ GBS “ what’s the prognosis of these diseases:
A. deterioration B. resolve with weakness C. full recovery✅
🍒peripheral muscular weakness history of infectious befor 3w? Guillain barre syndrome
…….
🍒 (Myasen gravies case ) pt well morning and deteriorating through out the day
Ask about pathophysiology of MG? autoantibodies to ach receptors ✅ 🌹
🍒male with typical m*y*asthenia gravis, what to prescribe:
A- rivastigmine B- ph*y*sostigmine ✅
🌹Bilateral ptosis, upper and lower limb weakness, Simpson test +ve, edrophonium test
performed and ptosis improved (I think), what medication to gave?
A. pyridostigmine ( myasins gravis).

…….
🌹 Vitiligo pathogenesis = Autoantibodies against melanocytes 🌹
🌹55 yo man diagnosed with acromegaly what investigation will be required in the future?
A. TTE B. colonoscopy (Acromegaly increase risk of colonic polyps and colorectal cancer
🌹Case about Acromegaly. what should be done for the patient ?
A _ Echo ✅✅ Bez risk of Heart failler. B.colonoscopy
🌹 Case about Acromegaly, what you will do for management?
Transphenoidal resection ✅ ✔ Transsphenoidal surgery is used to remove tumors of the pituitary gland

goodpasture , Wegener
🌹Tt of goodpasture syndrome dyspnea, cough, fatigue, hemoptysis, and hematuria?
A. Steroid B. Blood transfusions C. Cyclophosphamide
🌹Hx of recurrent sinusitis presented with hematuria = Wegener
🌹 Hematuria hemoptysis= Goodpasture syndrome
🌹AKI + hemoptysis + * recurrent sinusitis + saddle nose * = WEGNER
🌹AKI + hemoptysis = Goodpasture
🌹Nephritic syndrome clinical pic then develops hemoptysis what is the likely cause:
Goodpasture syndrom
4 Œ ‫„ اﻟﺮﺋﺘ‬4 ‫ﻒ ﻣﺘﻜﺮر‬ž‹4 4 ‫‹ ﺑ‬4 Œ ‫ﻳﺘﻤ‬: ‫ﻣﺘﻼزﻣﺔ ﻏﻮدا‬
•¯‫ـ ـﻊ ﻟﻠ‬ž£‫ اﻻﻟﺘﻬﺎب اﻟ‬5‫¬ )ﺎﻻﺿﺎﻓﺔ إ‬
) ) 6
** ‫ ﻓﻬﻨﺎ اﻟﺮئ‬، ‫ﺔ‬F‫ﻠ‬l‫ﺎﻟ‬Y ‫ﻔﺮﺗﻚ‬F‫ﻨﺎ ﻗﺎﻟﻚ ﻛﺤﺔ ﻓﻴﻬﺎ دم و ﻋﻨﺪە ﻣﺘﻼزﻣﺔ ﻧ‬b** ‫ ﻣﺘﻼزﻣﺔ ﻏﻮدا‬W% ‫ اﻟﺮﺋﺔ ﻓﻜﺮ ﻓﻮرا‬W% ‫ﻠﺔ‬f‫ﺔ ﻣﻊ ﻣﺸ‬F‫ﻠ‬l‫ﺎﻟ‬Y ‫ﻠﺔ‬f‫اذا ﺷﻮﻓﻨﺎ ﻣﺸ‬
)
‫ ﻣﺘﻼزﻣﺔ ﻏﻮدا‬W% ‫ﺔ ﻓﻴﻬﻢ اﻧﺮاض ﻓﻮرا ﻓﻜﺮ‬F‫ﻠ‬l‫واﻟ‬

🌹IgA Nephropathy (Berger Disease)= Look for an Asian patient with recurrent episodes
ofgross hematuria 1 to 2 days after an upper respiratory tract infection (synpharyngitic).
This actually helps . Treat by 🍯🍯== Thirty percent will completely
resolve. ((Support )).🌹 but if there is ( sever protinuria gove ACEI. )
²§‫ ر`ﻂ =اﺳﻤﻬﺎ ﺻﻐ‬Ig ¦ ƒ § ‫ﺲ ﻳﻮﻣ‬ñ= îƒ‫ “ﻌ‬Ig + ‫ ﺣﺮف‬A ‫ = اول ﺣﺮوف اﻟﻬﺠﺎء‬Uper resp = ‫ء ﻓﻮق‬ÔÕ ‫“ﻘﻊ اول‬.
o o
Iga = Ig= 2 days = A = upper resp
...
🌹Goodpasture also presents with lung and kidney involvement just = Treat with
plasmapheresis and steroids. Cyclophosphamide can be helpfu🌹
/o ‫ﺸ‬ò ‫ﺸﻮف ﻣﻜﺮوﻧﺔ {ﺎﺳﺘﺎ {ﺎﻟﺠﺒﻨﺔ واﺷ}ﺎء‬ï ‫ ﻟﻤﺎ‬good pasture = 🍝🍝 ‫ ﻧ÷ﻨﻔﺲ {ﻘﻮة ﻣﻦ اﻟﺤﻤﺎس‬lung ‫ﺪء‬x‫وﻧ‬
-e‫ﻖ اﻟ‬wv‫ﻞ ﻋﻦ ﻃ‬k‫ﺬا اﻻ‬¹ ‫ واﻟﺠﺴﻢ ﻳﺘﺨﻠﺺ ﻣﻦ‬، ‫ﻞ‬k‫ ﻧﺎ‬kidney
.......
🌹Wegener granulomatosis (WG), there is no upper respiratory tract involvement. But
there is skin, joint, GI, eye, or neurological involvement.🌹
🌹diagnosis and symptoms of granulomatosis with polyangiitis = is: Wegner’s:
Hematuria; hemoptysis; sinusitis & URTI. ✅✅✅
🌹Scenario of patient with hematuria, pulmonary hemorrhage ,recurrent sinusitis and
numbness in Rt Upper limb and Lt lower limb ( they didn't mention any Rash )
A. Polyarteritis nodosa. B. HSP C. Giant cell arteritis D. Granulomatosis with polyangiitis✅

......
Goodpasture also presents with lung and kidney involvement just
ƒ § ‫ ﻣ”ﺎﻧ‬mƒ ‫ﻮا ﻓﻘﻂ ﺣﺼﻞ ﻟﻪ ﻣﺸ”ﻠﺔ‬w‫ﻨﺎر‬Û‫ﺸﻮﻓﻮە {ﺎﻟﺴ‬ò ‫ﺬا اﻟﻤﺮض‬¹
‫ ﻣﺜﺎل‬-e‫ﻤﺎ {ﺎﻟﺮﺋﺔ واﻟ‬¹‫¦ ﻻ ﺛﺎﻟﺚ ﻟﻬﻤﺎ و‬ o
(hemoptosis and hematouria ) = The best initial test is antiglomerular basement
membrane antibodies = Treat with plasmapheresis and steroids
……
🌹 IgA Nephropathy came after 3 days of URTI = Supportive
🌹 Child had urti one day next develop hematuria ?
A-psgn B-IgA nephropathy✅✅
🌹child with upper resp tract infection then devlop hematuria = IgA nephropathy ( Berger's
disease) 🌹
IgA . ‫ ﻋﻠﻰ طول ﻓﻛروا ب‬hematuria ‫ وﺑﻌدھﺎ اﺻﺎب ب‬upper resp infection ‫اذا ﺷوﻓﺗوا‬
upper = ‫ = اول اﻟﺣروف‬A = ‫رﺑط‬

SLE
🍓Pt with malar rash, chest infection , anemia , renal impairment, what’s the dx:
lupus nephritis

🌹 Malar rash with photosensitivity , with hematuria what is the dx:


s lupus nephritis
lupus.\‫ ﯾﻌﻧﻲ‬photosensitivity ‫ ﯾﻌﻧﻲ اﻟﻛﻠﻰ ﺗﺎﺛرت و‬hematuria ‫ﻣن اﺳﻣﮭﺎ ﻧﯾﻔراﯾﺗس = واﻟﺳﯾﻧﺎرﯾوا ﻓﯾﮭﺎ‬

🌹 SLE on medication came with active arthritis =A. Steroid. B. Hydroxy. C. Cyclo
Mild to moderate > hydroxy. Severe > steroids

🌹 SLE pt follow up in clinic, no symptoms, controlled on medication,


High Urea , high Creatinin, high protein (nephritis) What is your management:s A. order
Anti Ds-DNA B. renal biopsy

🍒🍒They asked about the mechanism of action of one DMARD : Slow the progression
(Disease-modifying anti-rheumatic drugs (DMARDs)

🍒🍒 Heart block? .SLE🍒

🌹 Which of following is coming with criteria of SLE = hemolytic anemia

🌹critairia of SLE ?? autoimmune Hemolytic anemia 🌹 ( Not wight loss)

🌹SLE Obese + on hydrochloqin + whats will less effective of treatment or poor pronosis ?
Smoking 🌹

🌹Pt have SLE with neurological manifestation, pt already on corticosteroids what to add
now = Cyclophosphamide ( Not Phenytoin) 🌹

🌹The most common medicines known to cause drug-induced SLE = Isoniazid.
Hydralazine.✅ ✔
...... ...... ....
🌹Coomb test diagnosis what? Autoimmune hemolytic anemia 🌹

🌹Relation between smoking and SLE ? exacerbate the symptoms 🌹
🌹Pt has SLE s&s and he cames with I think on urine there is RBCs or red cast= .Lupus
nephritis 🌹
..
🍓SLE symptoms with protein in urine the : Lupus nephritis
..
🌹Known case of SLE, asymptomatic for years i think, presented with lupus nephritis
manifestation, urine analysis normal, next step? A. US B. Renal biopsy

SLE on medication what vaccine you will give him? Influnza.


..
🍓SLE with UTI , drug Contraindicated? Sulfa🍓

🍓 Women known case of sle has been free of all symptoms for many years and off
treatment labs show protienuria and high creatinine and urea what to do next:
A. Renal doppler B. Anti dsdna C. Ana D. Renal biopsy✅For lupus nephritis
.......
🍓 What medication to avoid SLE flare with

🌹 Clear case of UTI and SLE, what’s the drug associated with SLE flare up = Sulpha

SLE patient developed UTI. medications should be avoided?. Sulfamethoxazole ✅

🌹 SLE hydroxychloroquine effects:
A. decrease with smoking ✅✅ B. should not for lactating women
C. should not for pregnant woman D. increase by mycophenolate mofetil
............
SLE smoker with flared disease, what to ask the patient to decrease flaring? Stop smoking

🍒SLE nephritis and imparid renal function + proteinuria what lifestyle modifications to do
= . smoke cessation

🍒clear Hx of. SLE ask about diagnostic test: anti dsDNA
......
🍒SLE patients with active arthritis and malar rash treatment: Hydroxychlorochoine
......
🍒SLE case with lab) All lab normal except low c3 , c4 or c2 Ask about treatment:
A-Methotrexate andhydroxychlorine B-Methotrexate and cyclophosphamide ✅
C-Methotrexate and MMF D-Azithromycin No steriod in choice
….
🌹 Pt presented with malar rash, mouth ulcer, arthritis. There was proteinuria,
high Cr, low complements, drug to give?
A. Steroid and hydroxychloroquine✅ B. Steroid and mycophenolate mofetil
C. Steroid and methotrexate
..
🌹Case of SLE on hydroxychloroquine with typical presentation and investigation of
obstructive jaundice and high liver enzymes, ask about what is your management
A. Steroid✅. B. MTX D. urodeoxycyclic
Jaundice and high liver enzymes indicated acute hepatitis SLE with hepatitis treated by steroids initially.

Patient with symptoms of SLE arthritis malar rash what medication should be started:
A/hydroxychloroquine and mycophenolate mofetil
B/hydroxychloroquine and methotrexate
C/ hydroxychloroquine and steroid✅✅
Hyrocycholoquine alone or with steroids if severe arthritits

….
🌹Pt with RA, on methotrexate was in remission, her Liver function tests previously was
normal. But upon this follow up, her LFTS were elevated, next step?
A. Start glucocorticoids B. Stop methotrexate✅

🌹Management methotrexate hepatotoxicity:-


-Newly or persistent increase in transaminases > Reduce methotrexate, investigate
-3 folds increase in transaminases > stop methotrexate, investigate.
.....
🌹old female with progressive joint pain during the day and Stiffness > 2 hours in morning,
female, wrist pain, nodule. Dx? A- reactive arthritis. B- Osteioarthritis. C- RA ✅ ✔

🌹Obese women RUQ pain jaundice puritus know SLE on Hydroxychloroquine Gtt and
indirect high , AST: 700 ALT: 700 AlP: high Bright echogenicity Liver, They did a Biopsy and
they find (interface hepatitis with plasma cell) what what is management ?
A_Prednisolone B_Methotrexate
….
🍒Pt SLE nephritis What to give now = steroid + cyclophosphamide
......
🌹SLE with arthritis on NSAID , hydroxychloroquine what to add?
A. Methotrexate B. azathioprine. C. cyclosporine
..
🌹 Patient known SLE, came with pale skin and fatigue, Labs Hb 10(low)
and ferritin 600(high), what is the cause?
A. Hemolytic. B. iron deficiency anemia ÐŽ‫اﻟﺤﺪ•ﺪ ﻋﺎ‬. C. anemia of chronic diseases\
🌹pt came with fever ,pleuritic chest pain and arthritis in small joints Labs.showed high
both CRP ER dx? A. SLE B.infective endocarditis C. RA with vasculitis D. PE

🌹Case of post viral petechiae and other symptoms, What to expect in bone marrow?
Increased megakaryocytes 🌹

Child 10 years old , abdominal pain , jaundice since 3 days Lab show high AST and ALT and
indirect bilirubin, Dx? a) Gilbert syndrome ‫ﻦ‬Ï‫ﺔ اﻟﺒ~ﻠﻮرو‬y‫ﺴ‬D ‫ﻦ ﻳﺮﻓﻊ‬Ä‫ﺪ وﻟ‬yÄ‫ ﻻ ﻳﺮﻓﻊ اﻧ—–ﻤﺎت اﻟ‬b)
Infective hepatitis ✅ ‫ﺪ واﻟﺒ~ﻠﻮروب‬yÄ‫ﻳﺮﻓﻊ اﻧ—–ﻤﺎت اﻟ‬
🌹Child 10 years old , abdominal pain , jaundice since 3 days Lab show high AST and ALT
and indirect bilirubin= Infective hepatitis 🌹‫ﻦ‬Ï‫ﺪ واﻟﺒ~ﻠﻮرو‬yÄ‫ﻳﺮﻓﻊ اﻧ—–ﻤﺎت اﻟ‬
🌹 indicate prognosis in hepatitis: decreas Albumin + high INR +high bilirupin .
…..
🌹Recurrent axillary infection with multiple sinus most common cause?Hair follicle🌹
. . . . ..
Pt with DM and RA, has bone pain and hypercalcemia, high renal function test . What’s the
diagnosis? A. DM nephropathy B. Amyloidosis✅✅ C. Membranous GN
• •
‫ ﻻن ﻣﺎدة‬amyloid ¤6‫ اﻟ‬ŒŽ ، ‫ „ﻞ ﻣ•ﺎن‬ŒŽ ‫ﺳﺐ‬ÒÓ‫ اﻟﻤﻬﻢ اﻣ~ﻼ•ﺪ ﻫﺬا ﻳ‬، ‫ﺖ ﻟﻪ اﻟﻢ ﻓﻴﻬﺎ‬yé‫ ﻓﺴ‬، ‫ﺗﻄﻠﻊ ﻣﻦ اﻟﻌﻈﺎم‬
‫ﺐ ا´ﺶ اﻣﺮاض ﻣﻨﺎﻋ~ﺔ اﻫﻤﻬﺎ‬Þ‫ﺴ‬7 ¢ Ž £ • ‫ﺎ‬y‫ﺪ ﻏﺎﻟ‬yÄ‫ واﻟﻘﻠﺐ واﻟ‬، ¤6‫ﺎﻟ‬u ‫ﺐ ﻓﺸﻞ‬Þ‫ﺴ‬Û‫ﺎﻟﺬات ﻓ‬u RA
….
🍒Left eye unable to abduct, when looking forward there’s squint towards the
nose which nerve affected: left 6th🍒
....
Pediatric patient with eczema, bruising and repeated infections, his uncle and other male
relative has same problem, diagnosis? wiskott aldrich syndrome
🍒🍒eczema, bruising and repeated infections = wiskott aldrich syndrome 🍒🍒
‫ﺴﻜﻮت‬E‫ ﻣﻊ و‬¢ ô
Ž £ • ÐŽ‫¸ ا‬Ž ‫ = اﻟﺜﻼ‬thrombocytopenia + Eczemia + repeated infection
............
Child with recurrent URTIs , eczema and thrombocytopenia both brother and uncle have
the same condition: Wiskott aldrich
..
child presented With recurrent chest infection and eczema and had family history of same
condition in uncle= Wiskott aldrich

🍒🍒child With unkonwn liquid paracetamol, patient was observed for 4 hours and was
stable, asymptomatic, what to do? observe for other 4 hours🍒🍒
..

🌹alcoholic hepatitis with high Alt and Ast = Alcoholic hepatiitis 🌹



🌹Schistosomiasis patient presents with dyspnea, what complication most likely happened:
A. Pulmonary HTN ✅🌹. B. Pericarditis
‫ﺴﺖ‬ã‫ﺴﻮﻣﺎ =ﺷ‬ã‫ﺴﺘ‬ã‫ ر`ﻂ =ﻣﻦ اﺳﻤﻬﺎ ﺷ‬chest = lung .
.......

1Rumatic RH
Question about rheumatoid arthritis and itʼs effect on pregnancy:
A) abortion✅ B) somthing antibodies C) retardation
# rheumatoid arthritis induce abortion.

🍓RA management = steroid and methotrexate


🌹 RA patient with severe symptoms not responding to methotrexate, hydroxychloroquine,
what to add? A. NSAID B. Adalimumab Answer is: B, for rapid relief > Nsaids
🌹Patient with rhumatic arthritis on methroxat and Hydrochloroquine but still not
managed. What to add?
A-Cyclophosphamide B-Azathioprine C-Adalimumab ✅ D-Sulfasalazine
.‫ ادﻋﻲ اﻟﺗﮭﺎب اﻟﻣﻔﺎﺻل ﯾروح ﻣﻧﻲ ﺟرﺑت دواﺋﯾن وﻣﺎ ﻧﻔﻊ وﺑﺟرب اﻟﺛﺎﻟث أدﻋﻲ اﻧﮫ ﯾروح ﻣﻧﻲ‬Adal = ‫رﺑط = ادال‬
🌹 rhumatic arthritis on MTX and Hydrochloroquine = Adalimumab 🌹
🌹 Patient know RA still symptomatic patient is on methotrexate, mycophenolate and
NSAIDS? What to add? Cyclosporin Azathioprine Adalimumab ✅

🌹Patient with RA and has lung symptoms (dyspnea and other symptoms):
A. bronchogenic cancer B. Pulmonary fibrosis C. caplan syndrome ✅
🍒Caplan disease or rheumatoid pneumoconiosis) is a combination of rheumatoid arthritis
(RA) and pneumoconiosis that manifests as intrapulmonary nodules,🍒
🌹A lady with Rumatic arthritis that has progressed what marked is needed to confirm?
Anti- CCP (Anti-citrullinated protein antibodies)
.....
🌹What of these is the minor criteria of jones criteria of rheumatic fever :
Fever , arthralgia , Prolonged PR interval , High CRP and ESR , leukocytosis.
🍒What is of these criteria is minor jones criteria ?
A. syndenham chorea B. erythema marginatum C.fever✅

🌹 Rheumatoid arthritis on DMARD ?


DMARD=modifying antirheumatic drugs
A.Exercise prevent post inflammatory contracture✅ B.Complete bed rest
......
🌹32 years old male complaining of left pain O/E: the left knee is swollen and tender knee
aspiration result: WBC 40000, with negative culture Patient also had history of right
reactive arthritis few months ago ، appointment medication?
A.Metho ✅ bez its rhumtaic arthritiis B. Ibuprofen C. Amoxicillin
🌹man with hx of reactive arthritis has another attack with aspiration of 40000
neutrophils. Intial management? A. Ibuprofen B. MTX C. infliximab
🌹 reactive arthritis ttt if no response to nsaid ? A. sulfa drug✅ B. cyclosporine
🌹 Reactive arthritis Mild> NSAId. severe > steroids
Nsaids If failed > Steroids If failed >DMARDS (sulfa, MTX)

🌹multiple joint pain and subcutaneous nodules = Rheumatic fever = anti ASO 🌹
🍓Case of rheumatic fever with painful nodule.and fever and joint pain ///what
investigation to order = Culture Throat culture or ASO

one of these is a minor criteria for rheumatic fever :


a-subcutaneous nodules. b-pancarditis. c-polyarthralgia (answer)✅. d-chorea
….

most common complication after strep throat is: (rheumatic fever not in choices)
A- scarlet fever B- post strep. GN ✅✅ C- pharyngitis
.... .....

… Celiac…

🍓Celiac extra-intestinal manifestation? erythema nodosum , perianal disease


🍓picture of dermatitis herpetiformis = clear scenario of celiac disease
)‫ = ھﺮب ﻣﻦ ﺳﻠﻚ اﻟﻜﮭﺮب ﻟﺤﺴﻦ ﯾﺨﺮب ﺑﺸﺮﺗﮫ‬herp = ‫ ( = ﺳﻠﻚ‬celic = ‫رﺑﻂ‬
🍒abdominal bloating and diarrhea after eating wheat = manifestation associated =
Dermatitis herpitiform (Celiac disesde)
🍓Female with a I year history of abdominal discomfort that improved with
defecation. Diarrhea with mucus. Tenesmus. No blood in stool. No weight
change. What is the diagnosis? A. Celiac ✅ B. Hyperthyroid
🍓which of the following can't be prescribed for celiac patients?
A. rice B. potatoes C- Barley
🍒🍒Celiac disease = Ass by = tissue transglutaminase IgA🍒🍒
🌹31 yrs old diagnosed of celiac disease still having symptoms, suspected no compliance to
medication, how to know for sure? A. endomysal B. biopsy
C. anti Tissue transglutimase D. dite diary
🌹Pt 28 yrs e symptoms of celiac disease symptoms. Which part of intestin is almost
affected? A.ilium B.colon C.jujenum ✅ ( Proximal intestine )
🌹What is true about celiac?
A. Biopsy has no role in diagnosis. B. Can’t affect adult C. Simple blood serology can confirm
dx D. Successful Treatment of gluten free diet consider diagnostic✅
Very long scenario and at the end it says biopsy from intestine was obtained and
shows total villous atrophy diagnosis? A-chron B-UC C-celiac✅ D-short iluem syndrome
🍓 Very long scenario and at the end it says tissue transglutaminase IgA was positive what
is the treatment? A-wide spectrum antibiotic. B-steroids. C-gluten free diet ✅ ‫اﺗﻮ‬
🍓 Celiac px , lost follow up what expect to see?
A. low Vitamin D3. B. Low vitamin C. C. Low vitamin E
🍓 Celiac px lost follow up , what expect to see?Wrist x- ray shows Cupping and fraying of
metaphysis

🍒Indicator of cystic fibrosis= poor wait gain🍒


🌹Case of cystic fibrosis but without mentioning the dx, what u will find else in
examination? Nasal polyp (Nasal polyps are common in people with cystic fibrosis)

🌹 Long scenario child with LL edema, periorbital, has fever and lethargy with dark urine
and protein in urine what else u would ask for?
a. history of sickle cell anemia b. impetigo 1 month back✅ c. recurrent uti

1Radiology
🌹 Case of 30 years old asymptomatic found accidentally lung mass
measured 7*8 mm everything is normal examination and Inx what most
appropriate management :A. Take biopsy. B. Repeat CT 3 to 6 month. C. Reassurance

Best diagnostic imaging for a scenario of intestinal obstruction


A.Plain x ray B.Enema C.CT contrast✅✅
🌷Best diagnostic imaging for a scenario of intestinal obstruction = CT contrast🌹

🌹 Patient with headache that comes and goes a lot during the month and it is usually on
the left or right sinus she used over the counter analgesic decongestants but didn't help
she complains now of severe headache she's otherwise healthy with no symptoms what
will you order to confirm the diagnosis: A/ESR B/CT of paranasal sinuses✅ C/MRI
infection ‫ او‬، tumor ‫طﯾب ﻟﯾﮫ طﻠﺑﻧﺎ اﺷﻌﺔ ؟ ﻋﺷﺎن ﻧﻌرف اﯾش ﺳﺑب اﻟﺻداع اﻟﻣﺳﺗﻣر واﻟﺷدﯾد ﯾﻣﻛن ﯾﻛون ﻓﯾﮫ‬
، c.t paranasal ‫ ﺛﺎﻧﯾﺎ ﻧرﻛز ﺑﺎﻻﺧﺗﯾﺎرات ھو ﺣدد‬، ‫ﻓﻼزم اﻋﻣل اﺷﻌﺔ ﺣﺗﻰ اﺳﺗﺑﻌد وﺟود ﻣﺷﻛﻠﺔ وﺣﺗﻰ اﻛﺗﺷف اﻟﺳﺑب‬
‫ وﻟﻠﻌﻠم‬C.T paranasal . ‫ ﺑﺷﻛل ﻋﺎم ﺑدون ﻣﺎﯾﺣدد ﻟﻲ اﻟﻣﻛﺎن ﻓﻧﺧﺗﺎر‬MRI ‫ ﻛﻣﺎن ﯾﺳﺎﻋدﻧﻲ ﺑس ﻻن ﻗﺎل‬MRI ‫ﻻن‬
sinus headch .....‫ﺗﺷﺧﯾص ھذه اﻟﺣﺎﻟﺔ ﯾﺳﻣﻰ ب‬

🌹 Elderly with liquid dysphagia and ask about valuable diagnostic tool ? Endoscopy
🌹 pt with loss of wt and vague epigastric pain for 3 months and loss of appetite :
Endosopy✅ (‫) اﺣﺘﻤﺎل ورم‬. CT Mri
🌹 Dr decided to manage the pt by US guided percuteness drainage, how should take
concent? GS resident. – Radiologist - Medical intern - Any one of medical team
.....

🌹) 70 y/o with multiple hepatic masses seen by CT, what is the next step =
Repeat CT. Biopsy. Colonoscopy ✅ MRI
‫ ﻟﯾﮫ اﺧﺗرﻧﺎ‬Colonoscopy ‫ ﻻن ﻏﺎﻟﺑﺎ اﻻورام اﻟﻲ ﺗﯾﺟﻲ ﻓﻲ اﻟﻛﺑد ﺗﻛون ﺑﺳﺑب‬metastasis ‫ﯾﻌﻧﻲ ورم طﻠﻊ ﻣن ﻣﻛﺎن وﺣﺻل ﻟﮫ اﻧﺗﺷﺎر وﺧﺎﺻﺔ ﻟﻣﺎ ﻗﺎل‬
multible mass ‫ ف اﻧﺳب‬. ‫ﻓﻣﻣﻛن اﺻل اﻟورم اﻟﻲ اﻧﺗﺷر طﻠﻊ ﻣن اﻟﻘوﻟون وﺣﺻل اﻧﺗﺷﺎر ﻟﻠورم وراح ﻟﻠﻛﺑد ﻓﻼزم ﻧﻌرف اﺻل اﻟورم ﻣن ﻓﯾن طﺎﻟﻊ‬
‫ ﺑﺎﻗﻲ اﻻﺟوﺑﺔ ﻣﺎﻟﮭﺎ داﻋﻲ ﺧﻼص ﻋﻣﻠﻧﺎ‬. ‫ ﺟواب ھو ﻓﺣص اﻟﻘوﻟون‬c.t ‫ ﻟﯾﮫ ﻧﻌﯾده وﻟﯾﮫ ﻧﺧﺗﺎر‬MRI ‫وﺧﻼص ﻧﺣﻧﺎ ﻋرﻓﻧﺎ اﯾش ﻣوﺟود ﺑﺎﻟﻛﺑد‬..... . . .. . . .
🌹Case of diaphragmatic hernia first step in order to diagnose? Cx ray✅✅
🌹patient with symptoms and signs of alcoholic and deep breathing initial test?
A.Alcohol concentration in blood✅✅ B. Patient rebreath in bag
🌹case of cardiac tamponade how to dx? -echo✅
🌹 Long scenario: Radiologist asked by the treating team to do Ct guided Fluid aspiration,
Who should get the informed consent? A- Radiologist✅ B- The resident from the treating
team C- The nurse
🍒 Pt 55 years old Dm and Htn complain of thigh swelling and tenderness have previous
history of trauma in same area before 3 month what is the initial investigation ?
A. us B. xray✅

🌹Diagnosis of meigs syndrome = CT🌹


A. CT ✅ B. MRI NO US in choices ‫ﺠﺲ‬w‫ﻂ =ﻣ‬n‫ ر‬megis = ‫ﺎرة ﺟﻤﺲ‬w‫ ﺳ‬¤: } ‫ =ﻫﻞ ﺳﻮﻗ‬¤: } ‫ =ﺳﻮﻗ‬£: } ¢
: = ‫ﺎرة ﺟﻤﺲ‬w‫ﺳ‬
🍓pt with thigh mass 5x8 cm , biopsy show sarcoma high grad , what investigation you will
order for staging: A. MRI abdomen B. CT chest✅ C. bone scan D. X ray thigh
Sarcoma staging:- Local invasion: MRI distant: CT
🌹case with situs inversus and infertility: = kartenger
Kartagener's syndrome is a rare genetic disorder. It involves abnormalities of your cilia,
which can cause a variety of health problems. It also involves situs inversus= condition in
which your internal organs develop on the opposite side of your body than normal.
‫ = ر`ﻂ‬kart = ‫ﺬا اﻟﻤﺮض “ﻜﻮن ﻣﻌﻪ اﻻﻋﻀﺎء ﻋﻜﺲ {ﻌﺾ‬¹= ‫ﺎرت ﻟﻠﻌﻜﺲ‬e‫ ﻧﻘﻠﺐ اﻟ‬، ‫ﺔ اﻻوﻧﻮا‬x‫ ﻟﻌ‬.
...........
🌹Pt with CKD what give Pt before CT with contrast? iv fluid
🌹Pt will do CT with contrast after give contrast develop whizzing what you want to do ?
A. Steroid B.Epinephrine ‫ﻐﺔ‬y‫ = ﺣﺼﻞ ﻟﻪ ﻣﻦ اﻟﺼ‬allergic

🌹Old patient with anemia , next step ? colonoscopy🌹

- P'tient post resection and colostomy presents with spiking fever for 1 week. Chest is
clear. Abdomen is clear. Wound is clear. Fever 38.2 oC. Next step?
- CT abdomen✅✅. - Reassure

🌹Screening for polycystic kidney==US🌹


🌹pt with ling cancer now he has ( basel crakels and rise JVP ) Confirm diagnosis by = C.T
scan 🌹
.....
🌹Whats type of kidney injury heppen when use "contrast " = Acute tubel necrosis 🌹
🍓 Patient in his 40s CT with contrast was done for him, they found out later that he has
renal failure etc what kind of damage will this do to his kidneys: Acute tubular necrosis
🌹Gold investigation poly cyst in kidney: US 🌹
...............

🌹High prolactin next step in the investigation? Brain MRI ✅


‫ ﻟ~ﻪ ﻗﻠﻨﺎ‬MRI ‫ ﻏﺪة‬ŒŽ • ‫ﺐ ورم‬Þ‫ﺴ‬7 ‫ •ﻜﻮن‬ñ
• ç ‫ﺘ‬Ù‫وﻻ‬Ò£‫ ﻻن ارﺗﻔﺎع اﻟ‬Pitotary gland = ‫–ﻖ‬À‫ ﻫﺬا اﻟﻮرم ´ﺸﺨﺺ ﻋﻦ ﻃ‬MRI
🍒Macroadenoma of pituitary cause sign and hypofunction, management:
A. steroid. B. surgery ✅
🌹 Women with sudden loss of vision . Radiological finding found mass in prussered over
optic chiasm inside it hemorrhage when do surgery ?
A. Urgent Resection now B. Repeat Imaging after 1 Wk
……..

🌹 Female came complains of headache + sudden painful loss of vision, Examination:


loss of vision in right eye. CT brain : pituitary hemorrhage MRI brain: pituitary mass,hge +
compressing on optic chiasma & cavernous sinus. Best next step?
A. Close observation B. Medical therapy C. Inferior petrosal sinus sampling
D. Urgent neurosurgery referral

🌹screening polycystic kidney by = US🌹


🌹gastric pain worse after Eating what (beast ) invest = Endoscopic 🌹
.
🍒CXR finding horizontal fissure : transient tachypnea of newborn🌷
🌷Patient present with loss of height, next investigation‫ ؟‬A. BMD. B. x-ray to spine
🌷Cushing syndrome, ACTH<1=adrenal CT🌷
🍒Anemia sarcoidosis = Anemia of chronic disease 🍒

🌹 +40yo female presented with dyspnoea, dry cough, reddish nodules on both of her
shins, fine crackles on auscultation. What is the appropriate management?
A. Montelukast B. salmeterol C. Steroid
‫ ﯾﻌﻣل اﻟﺗﮭﺎب ﺑﻛل ﻣﻛﺎن ﺑداﯾﺔ ﻣن اﻟﻌﯾن ﺗﻛون ﻣﺣﻣرة ﺛم اﻟرﺋﺔ ﺗﻌﻣل ﺿﯾق ﺑﺎﻟﻧﻔس وﻛﺣﺔ ﺟﺎﻓﺔ ﺛم‬sarcoidosis ‫ﻣرض‬
‫ ف اﻓﺿل‬، ‫ ﺣﻣراء ﻣؤؤﻟﻣﺔ ﺛم اﻟﻣﻔﺎﺻل ﺗﻌﻣل ﻓﯾﮭﺎ اﻟﺗﮭﺎب‬nodule ‫ ﺛم اﻟﺟﻠد ﺗﻌﻣل‬، ‫اﻟﻛﺑد واﻟطﺣﺎل ﺗﻌﻣل ﺗﺿﺧم ﻓﯾﮭم‬
steroid ..=‫ﻋﻼج ﯾﻘﻠل ﻛل ھذه اﻻﻟﺗﻌﺎب‬

🌹 pulmonary sarcoidosis symptoms + skin manifestation. How to confirm?


A- chest x-ray B- skin biopsy ✅
🌹+Young female presented with dyspnoea, low grade fever and arthralgia.
On examination there was tender erythematous nodules on her shin. And on auscultation
basal crackles. next? A. Chest X Ray B. Steroid C. Blood culture D. Skin biopsy
🌻sarcoidosis case

🍒 if bilateral hilar mass lymphadenopathy > Sarcoidosis🍒

🍒HCV how to screening = antibody


🍒How confirm HCV= PCR🍒
🍒female with lower limb edema. Distal pulses are intact. Only dilation of greater
saphenous vein tributaries. What is your investigation?
A. CT venography B. Venous duplex ultrasound* ✅✅
🍒Female what to do ct scan which doctor has to ordered before ct scan =
A. Pregnancy test ✅🍒 . B. Full bladder
.... .....

🌹What is sign in radiology for duodenal ulcer ==clover leaf


‫ = ر`ﻂ‬doudenum = ‫ = ودود‬love = clove

….
….
💐💐A man his chest x-ray shows opacity in the apical lobe, what to do? review previous x-
rays 💐💐

💐Where to put midaxillary Thoracentesis? 8th to 10th intercostal space 💐💐

🌹coarction of aorta ( radial pulse more than femoral whats the nexst = = X.ray
< _ ‫ﻪ اﺧﺘﻼف <; اﻟﻨ•ﺾ ﺑ‬w‫ اول ﻣﺎ ¦ﺸﻮﻓﻮن اﻧﻪ ﻓ‬radial and femoral ‫ﺎ~ﻢ ﺗ«ﺴﻮﻫﺎ‬2‫ ا‬، ª‫ﻪ ﻛﻮرا©ﺸﻦ اور‬w‫ ﻓﻜﺮوووووا ﻓ‬. 🌹
¨ :
. . . ..
🌹Elderly with central line swab E coli what do: Remove central line. 🌹
🌹Gram positive grow from central and peripheral line what to do?
A. Remove line✅ B. give antibiotics

Embolism
🌹5 days post orthopedic surgery had sudden dyspnea and confusion on examination
shows rash on neck and on cxr bilateral lower lobe infiltrates .cause?
A. Fat embolism B. PE C. pneumonia
🌹Chronic kidney disease what to use=UFH
🌹Now Admitted for pneumonia, have hx of ESRD and do hemodialysis regularly , best
prophylaxis to her condition at the hospital ? A. UFH B. stoking C. LMWH
UFH‫ ﻧﻌطﯾﮫ‬anticoagulntion ‫ﻣرﯾض ﻓﺷل ﻛﻠوري وﯾﺣﺗﺎج‬
16) Female has HTN / DM / end stage kideny disease on dialysis admeted due to
pneumonia / what is the best way for DVT prophylaxis ?
- enaxprin -UFH ✅. - fondanprix - compression stocking
........ ......

🌹Elderly with abdominal pain and heart disease = Mesenteric ischemia 🌹


🌹Thumbprint sign found in x ray= mesenteric ischemia
🌹 Hx of cardiomyopathy 65Y old male , Sudden Abd pain for 8h , tenderness, abnormal
bowel sound , high Amylase ?
A. mesenteric ischemia. B. Acute pancreatitis. C. Choledocholithiasis
🌹Patient with CHD developed epigastric pain and increase bowel movement ?
A. Pancreatitis B. Acute mesenteric emboli
‫ة راح ﻟﻼﻣﻌﺎء وﻋﻤﻞ ﻟﻬﺎ اﺳﻜ}ﻤ}ﺎ‬², ‫ﺎن ﻋﻨﺪە ﺟﻠﻄﺔ ﺳﺎ{ﻘﺔ اﺣﺘﻤﺎل “ﻜﻮن ﺧ‬á ‫ﻻن‬
🌹Old patient with claudication symptoms, = Aortoiliac insufficiency ✅

🌹DM, htn, claudication after walk 200 m , no distal pulse :


A. heparin and follow up B. surgical catheter C. subcutaneous thrombolysis
🌹case with Hemiparesis due to previase infarction , now present with C.t infarction with
no clinical symptom what will give ? LWMH ✅✅
( another choice wrong = was warfarin , embeloctomy , thrombolytic )
🍒 Fat embolism= After long bone fractures, sudden SOB + Petechial rash in the chest 🍒
Patient developed Heparin Induced Thrombocytopenia, what to do? (They
don’t give you the diagnosis, only the lab results)
A-Continue and observe B-Switch to SC Enoxaparin
C-Switch to another kind of medication (I don’t recall its name)
D stop Heparin and add new oral anticoagulant like (Argatroban)✅

🌹patient with signs of acute limb ischemia. Peripheral pulses absent in right leg (dorsalis,
popliteal) and dorsalis pulse absent in left leg.
A-heparin and observe B-femoral thrombectomy C-catheter thrombolysis✅
Catheter-directed thrombolysis (CDT) is an endovascular approach to the treatment of
acute limb ischemia;
🌹old age male patient post RTA and stroke (did not mentioned if it was hemorrhagic
or not) that complicated with hemiparalysis since two years ago, presented to ER with SOB
and cough, diagnosis of PE was suspected, investigation was done showed massive PE (high
pressure in pulmonary veins, patient vitally was unstable) management=
A. IVC filter ✅ B. Heparin C. Warfarin D. Thrombolytic
🌹case of 55 y/o make pt known case of afib (nothing else mentioned in q)what would
prevent further complication? A. warfarin. B. heparin C. no drugs recommended. D. aspirin
🌹77 female with palpitation. Clinical assessment showed asymptomatic atrial fibrillation.
asking about the first drug that should be prescribed
A. Aspirin B. Clopidogrel C.Anti- coagulant ✅✅🌹
🌹 young Pt had Afib ( No DM , HTN, CVA ) ask about prophylaxis A. No need. B. Warfarin.
C. Heparin
🍒Old man 65 years, diabetic and hypertensive, came with atrial fib, most appropriate
management? A. aspirin and clopidogrel B. Aspirin C. Warfarin ✅ D. clopidogrel
🌹 Atrial fibrillation prophylactic: A) aspirin b) clopidogril C) warfrin✅
🌹 Old Pt K/C of DM and HTN complaining of Rt LL pain and swelling femoral pulse intact
and absent of distal pulse.=: A. embolectomy. B. Heparinization and catheterization
🍓 Case of peripheral artery disease Pt came with pain , reduce pulse, cold limb , no
sensation, decreas motor Most apporpeat next step?
A. Heparin B. Ct angiography C. Conventioal .... D. Duplex us

🍓 diabetic Pt concern about his foot wound that did not heal (wound was managed in
hospital 2 months ago) A1C: within normal, The cause of his problem?
A. Decrease blood supply..✅. B. poor compliance to medication
🌹Patient k/c diabetes and smokers with claudication for 100m when he goes to the
mosque, his femoral pulse is intact but distal pulse is diminished, what is most appropriate:
A- medical risk stratification B- surgical bypass of stenosis
C- endovascular repair of stenosis D- Smoking cessation with aggressive blood sugar control

🍓 After MI and PCI pt on warfarin, ?


A. Discontinue warfarin +add 2 antiplatelets B. Continue warfarin + add 2 antiplatelets
C. Continue warfarin + add 3 antiplatelets
🌹Post pci came with Afib on warfarin statin , What will you do ?
A. Continue warfarin monotherapy B.Warfarin and two anticoagulant✅
🌹Patient with MI on warfarin and now present with afib ?
A. Continue on warfarin B_warfarin and dual antiplatelet
🍒Minum corse of warfarine post PE /3 months = True if provoked
🌹Man in 50s hx of brain infarction 6 months ago on warfarin :
A. Dc warfarin B. Same Management✅ C. Add Aspirin
🌹the case on warfarin and INR is 7 will u do= stop warfarin then repeat INR next day
‫ ﯾﻛون ﻣن اﺛﻧﯾن اﻟﻰ‬INR ‫ اﻟﻣرﯾض اﻟﻲ ﯾﺳﺗﺧدم وارﻓرﯾن ﻣﻔروض‬، ‫ او اﻗل‬1 ‫ ﻣﻔروض ﯾﺳﺎوي‬INR ‫ﻻﻧﺳﺎن اﻟطﺑﯾﻌﻲ ال‬
. ‫ ﺟدا ﻋﺎﻟﻲ ﻓﻼزم ﻧوﻗﻔﮫ‬7 ‫ اﻣﺎ ھﻧﺎ‬3.5-2.5 ‫ وﻟو ﻣرﯾض ھﺎي رﯾزك ﻣﻣﻛن ﻧزود اﻗﺻﻰ ﺷﻲء‬INR = 2.0-3.0 ‫ﺛﻼﺛﺔ‬
🌹Old pt, hypertensive, with left hemiparesis for 10 days! Imaging showed area of
infarction, what to give: A. Warfarin B. aspirin C. one of the NOAC’s (apixaban i think)
🌹 72 y/o pt known of asthma and migraine, came with tachycardia with irregular sinus
and dx as A.Fib what should give for this pt:-
A. asprin B. dipyridole C. asprine and clopidogrel D. warfarin ✅
🍒 55 y with afib +\_ diabetes?
A. Warfarrin ( we can use if INR 2-3). B. Heparine (just for pt has PE)
C. Enoxaparine ✅( Bez he is high risk for thrombosis )
🌹priphrall artery dis case asked about best next step A.Haparin B_CTA
🌹PAD asked about most accurate test A. Duplex B. CTA
🌹Patient had RTA and became paraplegic for I forget the duration but after2 year came
with clear sign and symptoms of DVT with Right sided heart failure what is the most
appropriate management= A/Enoxaparin B/Warfarin C/Infusion of alteplase
🌹Pt w/t arrhythmia on warfarin and high risk of stork:
A- Complete warfarin and add clopidogrel B- Only warfarin ✅✅
🌹77 years old DM HTN with prior Hx of TIA came for OPD for routine evaluation he doesn't
complain about anything, OE irregular pulse and ECG shows Afib with rate 70 was not there
6 month ago (exactly written like this) what is the most appropriate management :
A. Start pharmacological cardioversion B. Give warfarin to prevent stroke
C. Aspirin D. Follow up
🌹Case of aspirin overdose with lab results what’s the best next step =
Alkalinization of urine 🌹
🌹patient on TPN for 2weeks. with low INR. what complication do you expect
A.Vit k deficiency ✅ B.Chronic liver disease
🌹52 yo Dx as chronic A-fib, at wich pt increase risk of:
A. Acute Mi B. Sudden death.
C. Ventricular fib D. Cerebrovascular accident ( not sure from this answer)

1DVT and PE
🌹 old with DVT which of the following indicate thrombophilia test?
A. age. B. Hx of ocp use. C. Connective tissue disease “my answer” D. Negative family hx
🌹Px with first time PE , you will give anticoagulant for how long ?
A- 6 months B. 3 months
If Provoked > 3 months (immobilization, surgery, obese, pregnant, OCP, cancer)
If Unprovoked > 6 months
🌹Young lady with unilateral progressive leg swelling with no hx of surgery or prolonged
bed rest what is the most important question to ask in history:
A. History of smoking🌹 B. Family history of the same presentationC. History of fatigue
D. History of sleepiness
Smoking risk for DVT ‫وﺧﺎﺻﺔ اﻟﻘﻠﺐ‬. ‫ﺎﻟﺠﺴﻢ‬R ‫ﺎن‬P‫ﺠ;ﺐ ﺟﻠﻄﺎت ب اي ﻣ‬K ‫ﺎﻻوﻋ;ﺔ اﻟﺪﻣ~•ﺔ‬R ‫ﺎﺛﺮ‬K ‫ء‬z{ y N
O x‫ﺜﺜ‬v‫ ا‬tu‫اﻟﺘﺪﺧ‬
🌹 Pt with pyelonephritis,DVT ..? A. enoxaparin B. we choic UFH renal injury
🌹Sign of DVT no family hx, Most important to ask in the history:
A. using OCP B. History of trauma
🌹Contraindication for thrombolytic therapy (absolute):
A. Intracranial hemorrhage 2 years ago ✅✅ B. Cardiopulmonary resuscitation for 10 mi
C. Pregnancy.
🌹 I don’t remember the scenario exactly but pts was on heparin with no obvious complain
but in the laps there’s thrombocytopenia:
A. Stop heparin and start argatroban✅✅ B. Continue and reassess after 6 hours
C. Stop and start warfarin ‫ ( = ﺧﻄﺎ‬Warfarin should only be restarted when platelet count
>100 x 109/L)
Mangmant of Heparin-Induced Thrombocytopenia=
should prompt discontinuation of heparin and LWMH
Initiate anticoagulation with a non-heparin anticoagulant: e.g. argatroban, danaparoid,
fondaparinaux

🌹 patient with DVT, how to manage? enoxeprin


🌹 A woman who is on DVT prophylaxis (enoxaparin) presents with sudden onset
SOB and dyspnea. Upon physical examination she has diminished breath
sounds bilaterally. Imaging shows a pulmonary embolus lodged in the small
bronchial tree. The best isA. continue same dose of enoxaparin B. Switch to warfarin
🍒Female got admitted for DVT 2 days ago & started on LMWH heparin, now presents with
pulmonary embolism, vitals were provided and she’s stable, what’s the appropriate step?
A. continue same management B. Switch LMWH to UFH C. tPA D. thrombectomy
🌹 A case of PE, patient with hx of one year MVA and paraplegia. Now he is
hypotensive, And angio showed DVT extended to the thigh Mx?
A. IVC filter B. Thrombolysis C. Warfarin D. Aspirin
‫ ﻋﻨﺪە‬hypotensive ‫ﺾ‬65‫( اﻟﻤ‬% ) ‫ وﺟﻠﻄﺔ &ﻌ‬unstable ‫ﻪ ﻣﺬ&ﺐ ﻟﻠﺠﻠﻄﺔ‬F‫ع دواء ﻧﻌﻄ‬‰‫ف ا‬
.....
🍒Female pt presented to er with pyelonephritis then suddenly developed
PE features and unstable vitals what to give ? A. Heparin. B. Warfarin C. Alteplase >>
🍒Massive DVT after surgery what to give? A. Enoxaparin✅. B. Heparin. D. Thrombolytic
🍒 A PE developed in a DVT case on heparin Rx, no CI to thrombolytics in the
question?A. Interventional thrombectomy B. Increase the heparin C. Thrombolytics
🌹Pt w/t HTN, DM, ESRD.. will go for hemodialysis .. how t prevent DVT:
A- UF heparin✅ B- Enoxaparin C- One of DOACs
🌹two questions about pt. With A.fib and signs and symptoms of pvd (claudication) for 6
h,one about dx: A- CT angio✅ B- classical angio C- dopple
🌹and the other is about treatment:
A- heparin ✅ B- endovascular thrombolysis C- thrombolytic drug i think
🌹DVT thrombus from popliteal extend to femoral, vitally stable, ttt?
A. Enoxaparin B. Heparin & warfarin C. IVC
🍒Pt with pyelonephritis and dvt ?
A-We treat by enoxaprine ✅ B-UFH with warfarin C-atplase
🍒40 yrs primegravida healthy with previous DVT hx What to do now?
A. Asprirn B. Enoxaparin✅ C. Heparin D. No anticoagulant
🍒 known with afib with leg pain what to do?
A.Heprin ✅ ( first give heparin , Bez suspect DVT )
B.warfrin ( If say "treat of atrial fib without any embolism , just give as prophalctix from
embolism , but here the embolism happen in the lef so satrt with hwparin)
🌹Patient with renal failure, DVT prophylaxis? Unfractionated herpatin ✅🌹
🌹patient with PE scenario, on radio saddle shaped PE was seen and on vital signs patient
wad hypotensive, what's the treatment?
A. Alteplase ✅✅ B. Warfare C. Lmwh D. Unfractionated heparin
🍒 Symptoms of PE women and was unstable ?thrombolytic ‫ ( ﻣﺜﻞ دواء‬Alteplase)🍒
🍒“”Saddle “” pulmonary embolism management
A. alteplase✅ espichally if hemodynamically unstable or it affected the heart.
🍒old male ( 72) medically free except from episodic weakness and resolved after 10 mins
and has Afib .. asking about the Tx:
A. - no tx B. – ASA C. - warfarin INR 2-3✅✅✅ D. - warfarin int 3-4
🌹 Massive DVT after surgery what to give?
A. Enoxaparin✅ B. Heparin C.Warfarin D. Thrombolytic
🌹DM, htn, claudication after walk 200 m , cold leg and pain there is pulse in femoral but no
distal pulse, ECG show A.fib What is most appropriate management :
A. heparin and observe B. surgical catheter C. Embolectomy✅ D. thrombolysis
Pulmonary Embolism
= investigantion :
1⃣_ Pulmonary Angiogram= (Gold Standard ) = negative angiogram excludes clinically
relevant PE .
2⃣_ D-Dimer = Highly sensitive D-dimer result can exclude DVT/PE.
3⃣_ Venous Duplex U/S or Doppler .
4⃣_ ECG = S1-Q3-T3 with massive embolizati.
5⃣_ Order scan if CXR normal, no COPD
Or there is Contraindication to CT (contrast allergy, renal dysfunction, pregnancy) .
Normal= excludes the diagnosis of PE
Treatment:
• patients with DVT only are often sent home on LMWH) .
Acute anticoagulation:
...Start with therapeutic-dose... SC LMWH or IV heparin...
🍇Anticoagulation will stops clot propagation‫ﺸﺎر اﻟﺠﻠﻄﺔ ﻳﻮﻗﻔﻬﺎ‬Û‫ اﻧ‬, prevents new clots and
allows endogenous
fibrinolytic system to dissolve existing thromboemboli over months.
🍇for SC LMWH like= Enoxaparin 1 mg/kg
🍇for IV heparin: bolus of 75 U/kg (usually 5,000 U) .
long-term anticoagulation
🍇warfarin: start the same day as LMWH/heparin – overlap warfarin with LMWH/heparin
for at least 5 d and until INR in target range of 2-3 for at least 2 d
IV thrombolytic therapy
‫ﻦ ؟؟؟‬w‫ﺎر‬x‫‘ “ﺬوب اﻟﺠﻠﻄﺔ {ﺪﻻ ﻣﻦ اﻟﻬﻴ‬o‫ ﻧﻠﺘﺠﺎة ﻟﺪواء ا‬în‫ﻣ‬
🍒 if patient has massive PE (hypotension or clinical right heart failure) and no
contraindications)
IVC filter:
‫ ﻧﻠﺘﺠﺎ ﻟﻠﻘﺴﻄﺮة ل ازاﻟﺔ اﻟﺠﻠﻄﺔ ؟؟‬în‫ﻣ‬
‫ﻒ‬wÙ‫‘ ذﻛﺮﻧﺎە ﻓﻮق ﻣﺜﻞ ﻟﻮ ﻓ}ﻪ ﺣﺴ÷ﺴ}ﺔ ﻣﻦ اﻟﺪواء او وﺣﺪة ﺣﺎﻣﻞ او ﻧ‬o‫ﺔ ا‬w‫ﺸﻦ ﻟﻼدو‬ã‫ا اﻧﺪ“ﻜ‬²n‫ﺎن ﻓ}ﻪ اي ﻛﻮﻧ‬á ‫اذا‬
‫ﺔ‬w‫ اﺳﺘﺨﺪم اﻻدو‬4o ƒ ‫ او اي ﺳ!ﺐ اﺧﺮ ﻣﻨﻊ ا‬،
‫ﺔ‬w‫‘ اﻟﺘﺠﺎة ﻓ}ﻪ ﻟﻠﻘﺴﻄﺮة اﻟﺮؤو‬o‫ﺬا اﻟﺴ!ﺐ اﻟﻮﺣ}ﺪ ا‬¹.
only if absolute contraindication to anticoagulation + recent proximal DVT .
🍇 duration of long-term anticoagulation?
‫ﺔ ؟؟‬w‫ﺔ اﻟﺠﻠﻄﺔ اﻟﺮؤو‬w‫ ادو‬-‫ﺴﺘﻤﺮ ﻋ‬ž ‫ﺾ‬wv‫ واﻟﻤ‬în‫ا‘ ﻣ‬
🍒 if reversible cause for PE (surgery, injury, pregnancy, etc.): 3-6 mo
🍒if PE unprovoked: 6 mo to
Thromboprophylaxis
🍒If Low Thrombosis Risk=
if patients is fully mobile and he done Surgery less than 30 min, fully mobile = No specific
prophylaxis
🍒Moderate Thrombosis Risk=
Most general, gynecologic, urologic surgery
Sick medical patients = LMWH.
🍒High Thrombosis Risk =
Arthoplasty, hip fracture surgery, Major trauma, spinal cord injury = LMWH Or
Fondaparinux Or Warfarin (INR 2-3) Or Apixaban Or Rivaroxaban .
🍒Warfarin, Dabigatran, Rivaroxaban, Apixaban For = Atrial Fib .
................
‫ﺬا اﻟﺴﺆوال “ﻤﻜﻦ ﺣﻠﻴﻨﺎە ﺧﻄﺎ ﺳﺎ{ﻘﺎ‬¹ ‫ف ﻣﺜﻞ‬
🌹DM, htn, claudication after walk 200 m , cold leg and pain there is pulse in femoral but no
distal pulse, ECG show A.fib What is most appropriate ( next step ) management :
A. heparin✅ B. CT angio C. duplex US

🍒🍒Elderly male k/c of DM & IHD presented with severe abdominal pain on examination
no findings in the abdomen but peripheral pulse was irregularly irregular == Mesenteric
ischemia 🍒🍒
‫ ﻗﺎﻟﻚ‬irregular irregular ‫ ﻋﻨﺪە‬ýŽ • ‫ •ﻌ‬atrial fibrlantion ‫ﺎﻟﺠﺴﻢ م وﻻن‬u ‫ ﺟﻠﻄﺎت ب اي ﻣ•ﺎن‬ýŽ £ ‫وﻣﻌﺮوف اﻧﻪ ´ﺴ‬
‫ اﻟﺴ|ﻨﺎر–ﻮا ﻓ~ﻪ اﻋﺮاض ﻟﻞ‬abdomen ‫ ﻣﻌﻨﺎﻫﺎ ﺣﺼﻞ‬embolism ŒŽ • mesentric ñ • ç ‫ﺎراﺷ‬y‫–ﺎل ﻓﻴ‬À‫ﺐ اﺗ‬yé‫ﺴ‬7 .
......
1Endocrine
🌹pt with turner + amenorrhea = FSH and LH ✅

🌹25 yrs old male referred from orthopedic clinic with fragility fracture, he has sparse
axillary and face hair. What inv. You will do?
A. Calcitonin B. Testosterone and gonadotropin C. Bone densitometry D. TSH

🍒Case of tumor lysis syndrome? Hypocalcemia


🍒Kidney failure, food avoid?tomato‫ﻮﺗﺎﺳﻴﻮم‬Ï‫ﻻن ﻣﻠ~ﺎن و‬
🍒 34 YO male came complaining of infertility, he has a hx of recurrent sinusitis, when he
did semen analysis they found normal counts, but decreased motility. Chest x-ray show
situs inversus. dx? A. cystic fibrosis. B. wegener granulomatosis C. kartagener's syndrome
✅ 🌹abnormal sperm motility = kartagener's syndrom🌹

🍓pt k/c of depression complaining of excessive urination and thirsty for 2 months. She did
mastectomy but now she has mets to the lungs. In the labs there is hypernatremia
,hyperglycemia and the urine osmolality is *low* What is the cause:
A. Diabetes insipidus✅. B. SIADH. C. Psychogenic Polydipsia
🍒 Patient RTA with head trauma, increased urine output , decrease in Urine osmolarity
increased blood osmolarity? Central diabetes insipidus✅
🌹high Na and low osmolarity in urine = Diabetic insipedis . 🌹.....
‫ وﺑﻌدھﺎ ﺳﺎر ﯾﺷﺗﻛﻲ ﻣن‬، ‫ ﺑﺳﺑب ﺣﺎدﺛﺔ او ﻏﯾره اي اﺻﺎﺑﮫ ﺑﺎﻟدﻣﺎغ‬brain ‫ ﻟﻣﺎ ﺗﺷوﻓون ﺣﺎﻟﺔ ﺷﺧص اﺻﺎب ﻓﻲ‬Diabetes insipidus , ‫اﻟﺗﺷﺧﯾص ھو‬
‫ ﻛﺛﯾﯾﯾر‬polyuria ‫ ﻟﻛن اﻟﻔﻛرة اﻧﮫ ﺳﺎر ﯾﺷﺗﻛﻲ ﻣن‬، ‫ ﻛﺎذب ﻻن ﻣﺎھو ﻣرض ﺳﻛر‬،( ‫(ﺿروري ﺗﺣطون ﻓﻲ ﺑﺎﻟﻛم ﻣرض )اﻟﺳﻛري اﻟﻛﺎذب‬polyuria)
‫ ﯾرﺳل اﺷﺎرات ﻟﻠﻣﺛﺎﻧﺔ وﯾﺧﻠﯾﮭﺎ ﺗوﻗف اﺧراج ﯾورﯾن ھذا اﻟﮭرﻣون ﻛل‬urine ‫وﺧﺎﺻﺔ ﻓﻲ اﻟﻠﯾل واﻟﺳﺑب اﻧﮫ ﻓﯾﮫ ھرﻣون ﺑﺎﻟﺟﺳم ﺳﺑﺣﺎن ﷲ ﯾﻣﻧﻊ وﯾوﻗف‬
‫ ﻓﻠﻣﺎ ﯾﺣﺻل ﻟﻠدﻣﺎغ ﺿرر ﺑﺳﺑب ﺣﺎدث او ﺟﻠطﺔ ھذا اﻟﮭرﻣون ﺧﻼص ﻣﺎﻋﺎد ﯾطﻠﻊ وﻣﺎﯾﺳوي ﺑﻠوك ﻟﻠﻣﺛﺎﻧﺔ‬brain ‫ اﺳﺎﺳﺎ ﯾطﻠﻊ ﻣن ﻏدة ﻓﻲ‬، ‫اﻧﺳﺎن ﻋﻧده‬
‫ ﻓﯾﮫ ﺷﻲء ﻣﮭم ﺟدا ﯾﺟﻲ ﻣﻊ اﻟﻣرﺿﻰ ھذوﻻ وھو‬، : antidiuretic hormone ‫ ھذا اﻟﮭرﻣون اﺳﻣﮫ ﻣن اﺳﻣﮫ‬polyuria ‫ ﻓﯾﺳﯾر‬، urine ‫اﻧﮫ ﺗوﻗف‬
‫ ﻻن اﻟﺟﺳم ﻓﻘد ﺳواﺋل ﻛﺛﯾرة ﻓﺎﺻﺑﺢ ﻛﻣﯾﺔ اﻟﺻودﯾوم ﺑﺎﻟﻧﺳﺑﺔ ﻟﻠﺳﺎﺋل‬، ‫اﻟﺟﻔﺎف ﺑﯾﺑب ﻛﺛرة اﻟﺗﺑول و ﺷﻲء ﺛﺎﻧﻲ ﻣﮭم وھو اﻧﮫ اﻟﺻودﯾوم ﯾﻛون ﻋﺎااﻟﻲ ﻟﯾﮫ ؟‬
Desmpressin........... ......‫ او‬vasopressin . ‫اﻟﻣﻔﻘود ﻋﺎاﻟﯾﺔ طﯾب اﯾش اﻟﻌﻼج ؟ ﻧﻌطﯾﮫ اﻟﮭرﻣون اﻟﻲ ﻣﺎھو ﻣوﺟود ﻓﻲ ﺟﺳﻣﮫ واﺳﻣﮫ‬

🌹seizure and low Na and high osmolarity in urine= SIDAH.


‫ اﻋرﻓوا‬low osmo ‫ واذا ﺷوﻓﺗوا‬، ‫ اﻋرﻓوا اﻧﮫ ﻛﻣﯾﺔ اﻟﯾورﯾن ﻗﻠﯾل ﯾﻌﻧﻲ اﻟﻌﻼﻗﺔ ﻋﻛﺳﯾﺔ‬high osmo in urin ‫ﻟﻣﺎ ﺗﺷوﻓوا‬
‫ ﻣن اﺳﻣﮫ ﺳﻛر ﯾﻌﻧﻲ ﻛﻣﯾﺔ اﻟﺑول ﻛﺛﯾﯾﯾرة‬Diabetis insipids ‫ ف ﻣرض اﻟﺳﻛري اﻟﻛﺎذب اﻟﻲ ھو‬, ‫ ﻋﺎاﻟﯾﺔ‬urin ‫ﻛﻣﯾﺔ‬
osmolorty low in urin ‫ ﻓﺎﻟﺑﺗﺎﻟﻲ ﯾﻛون ﻣﻌﺎھم‬، ‫وﯾﻌطﺷوا ﻛﺛﯾﯾﯾر وﺧﺎﺻﺔ ﻓﻲ اﻟﻠﯾل‬
🌹 Scenario about DI but did not mention the diagnosis,and ask about management?
Desmopressin ✅ Treatment for diabetes insipidus (DI) the e primary problem is a
deficiency of antidiuretic hormone (ADH)—also known as arginine vasopressin (AVP)
‫ ﻓﺎﻟﻌﻼج ﻧﻌطﯾﮭم ھذا اﻟﮭرﻣون اﻟﻲ ﻋﻧدھم ﻧﻘص‬، ‫ﻟﻲ ھو ﻛﺛرة اﻟﺗﺑول وﺧﺎﺻﺔ ﻓﻲ اﻟﻠﯾل ﺳﺑب اﻟﻣرض ھو ﻧﻘﺻﺎن ﻓﻲ اﻟﮭرﻣون اﻟﻲ ﯾﻣﻧﻊ او ﯾوﻗف اﻟﺗﺑول‬
Desmopressin‫ او‬vasopressin ‫واﺳﻣﮫ‬

🌹Patient diagnosed with major depression complaining of excessive thirst and urination
+ she has cancer with lung mets. investigations shows hyponatremia
and low urine osmolality what's the dx:
A. DI = (hypernatrima + low osmo urin)
B. psychogenic polydipsia✅ (come with chronic mental illness hyponatrimai + low osm
urine)
C. SIADH = (hyponatrima+ high osmos urin)

🌹Elavated 5HIAA = carciniod 🌹


🌹scenario was signs and symptoms of carcinoid (flushing, diarrhea....) Ask about the high
yield lab should be ordered? 5HIAA 🌹
🌹 case with “High 5 HIAA” diagnosis: A. Carcinoid B. Gastric stromal cancer
‫ﻂ‬Ï‫ =ر‬Hiaa =‫🚗ﻫ~ﺎ ﻣﻌﻬﺎ =ﺳ~ﺎرة ﻛﺮاﺳ~ﺪة‬carci
🍒Carcinoid tumor best initial diagnostic test? 5-HIAA🍒
5-HIAA is the major urinary metabolite of serotonin, are produced in excess by most carcinoid tumors.
……

‫اﻣﯿﻦ‬.‫اﻟﻠﮭﻢ اﺷﻔﻲ ﻛﻞ ﻣﺮﯾﺾ ﺑﺎﻟﺴﻜﺮ وﺧﺺ ﻣﻨﮭﻢ واﻟﺪي‬


1DM
🍓 pt with dm, smoker, with peripheral artery disease, what would you do to
improve his condition? A. dual aspirin. B. smoking cessation program C. exercise program
D. strict glucose control program
🌹 How to manage diabetic neuropathy Current A1C hb 7 , Previous 2 reading were 9-11:
A. Amitriptyline. B. strict glycemic control
🌹 Diabetic 70 year htn what is the most leading cause of fracture?
a-Motor accident b-Fall✅
‫ﺴ_^ ﻟﻪ ﻛ¸ ﻻن‬š ‫ﺎﻻرض ﻣﺎراح‬B ‫ ﺑ?ﻨﻤﺎ ﻟﻮ ﺷﺨﺺ ﺷﺎب وﻃﺎح‬، ¸‫ ﻟﻮ اي ﺷﺨﺺ ﻋﻤﻞ ﺣﺎدﺛﺔ ﺳﻮاء ﻛﺒ_^ ﺻﻐ_^ راح ﻳﺘﻜ‬، ‫ ﺳﻨﺔ‬٧٠ ^_‫ ﻋﻤﺮە ﻛﺒ‬±: } ‫ﻣﻨﻄ‬
¸‫ﺤﺼﻞ ﻟﻬﻢ ﻛ‬2 ‫ﻔﺔ ﻣﻤﻜﻦ‬w‫ﺤﺔ وﻟﻮ ﺧﻔ‬w‫ ﻋﻈﺎﻣﻪ ﻗ»&ﺔ ﻣﺎﺷﺎء ¼ ﺑ?ﻨﻤﺎ ﻛ•ﺎر اﻟﺴﻦ ﻳ•ﺪء ﻋﻨﺪﻫﻢ ﻫﺸﺎﺷﺔ اﻟﻌﻈﺎم و واي ﻃ‬.
.............
🌹 Dm on metformin not good response what add= oral hypoglycemia
🍒DM 2, has sudden monocular cuisine loss that lasts 20min and then subsides. No other
findings??
a-TIA✅ transient ischaemic attack. b-Retinal detachment
* transient vision loss (TVL) is a frequently encountered complaint thatEpisodes are often
ischemic in origin like (ocular infarction) also transient ischaemic attack can causes sudden
vision loss.
* ofetin unilatral vistion loss somthing on the eye either infarction or iscemic
Bilatral vistion loss somthing on brain like iscemia or hemohhrage . , but TIA can causes
vision loss on one eye .
🌷sudden monocular cuisine loss that lasts 20min and then subsides=ocular infarction or
transient ischaemic attack🌷

🍒pt. Did retinal operation 7 days ago, now need DVT prophylaxis, what to give:
A- heparin B- enoxaparin C- mechanical compression ✅✅ D- A+ C
‫ة‬FG‫ﺎﻟﺴﺎﻋﺎت اﻟ—ﺜ‬q ‫ة وﺟﻠﺲ‬FG‫ﺔ ﻛﺒ‬S‫ﻦ ﻟﻮ ﻋﻤﻠ‬C‫ﻨﻤﺎ اﻟﻬﻴ•ﺎر‬£‫ ﺑ‬، ‫ﺘﺤﺮك‬C‫¡ و‬ v % %
& ‫ﻤ‬: ‫ﻘﺪر‬: ، ‫ﻄﺔ‬S‫ﺴ‬x ŸG‫ﺔاﻟﻌ‬S‫ اﻟﺸ•ﻜ‬$& ‫ﺔ‬S‫ﻦ ﻻن ﻋﻤﻠ‬C‫ﺤﺘﺎج ﻫﻴ•ﺎر‬: ‫ﻣﺎ‬
‫ﻘﺪر ﻳﺘﺤﺮك‬:‫ﺎت وﻣﺎ‬S‫ﺎﻟﻌﻤﻠ‬q
🍒test of nephropathy in DM patient is by detecting which of the following:
A- urine albumin. B- GFR. C- protein creatinine ratio ✅
🍒DM patient on Metformin 1g BID, random blood glucose and a1c still high, what to
modify in treatment: A- increase metformin dose. B- give insulin. C- add _gliptin D- add
_glenide✅
‫ ﻣﺎﻧﻔﻊ‬Metformin ‫ ﻣﺎﻧﻔﻊ ﻧﺑدء ﻣﻌﮭم ب‬، ‫اول ﺷﻲء ﻟﻌﻼج اﻟﺳﻛر ﺣﺳب اﻟﺟﺎﯾدﻻﯾن ﻧﻣﺷﻲ ﺑﺎﻟﺗرﺗﯾب ﻧﺑدء ﻣﻌﮭم ب رﯾﺎﺿﺔ وﻧﻘﺻﺎن وزن‬
_glenide.‫ ﻣﺛل‬orall hypoglycima ‫واﻟﺳﻛر ﻣﺎزال ﻋﺎﻟﻲ ﻧﺑدء ب ﺟرﻋﺔ‬

🍒Pt DM on oral hypoglycemic drugs and now his DM is controlled with BMI 30 and tried a
lot of dite and for long time on low carbs dite with no reduction of wt how to decrease wt :
_Herbal substance. _Briatric surgery _Dite regimen _Medical reduction of weight ✅ ✔
Beriatric Sx:
■BMI ≥40 kg/m2 without comorbid illness.
■BMI 35.0–39.9 kg/m2 with at least one serious comorbidity; (type 2 diabetes, fatty liver
disease, hypertension).

🌹when to diagnose Diabetic nephropathy? A)_Hyaline deposition B)_ Proteinuria ✅ ✔


🌹DDP4 hypoglycemic medications have good effect in lower the blood glucose.
Which of the following medication does not have the side effect of the DDP4
hypoglycemics? A. linagliptin B. Saxagliptin
DM female, grey white vaginal discharge on speculum with spurs or something like that.
She is at risk of? A- Candida✅ B- Trichomonas. C- BV

🌹 Antibiotic contraindicat in diabetic pt = ciprofloxacin ‫ﺎﻟﺪم‬u ‫ﻳﺰود ﻣﺴﺘﻮى اﻟﺴﻜﺮ‬


🌹 What is the earliest effect of Diabetes Mellitus on the kidney:
Sclerosis with ↑ protein excretion.
ƒ § ‫وﺗ‬²ÿ‫ﻤﺎن اﻟ‬á ‘‫ اﻟﺒﻮل ﻋﺎ‬mƒ ‫ﺎ ﻣﺜﻞ ﻣﺎ اﻧﻪ اﻟﺴﻜﺮ‬¹‫ ار`ﻄﻮ‬، ‘‫ﻦ “ﻜﻮن ﻋﺎااا‬w‫ اﻟﻴﻮر‬mƒ ¦
‫وز‬²§‫ﺤﺼﻞ ﺳﻜ}ﻠ‬w‫و‬. ‘o‫¦ ﻋﺎ‬ ƒ
o o o o § ‫وﺗ‬²ÿ‫اﻟ‬
sclerosis .

🌹Diabetic patient on metformin 500mg twice a day with Dec sensation on both
lower limbs ha1c: 8 , FBG: 216 give? A. sulfonylurea✅B. mixed insulin C.long acting insuln

🌹Diabetic retinopathy with macular edema = Panretinal photocoagulation

🌹 Patient with family hx of type 2 DM came with, polyurea and FBS of 7.0, HbA1c of 7.1.
Dx? Type 2 DM

🌹Signs of nephropathy in DM2 ?A. Hematuria B. Proteinuria C. Hylin cast in urine....

🌹Hypoglycemia after insulin therapy:A- Honeymoon B- Somogyi C- Down


🌹Risk factors for DM 2= A.High bmi B. Abdomina l obesity ✅
🌹 Biggest risk factor for DM? A. Central obesity✅✅ B. High body mass index
🌹Most benefit for control dm? decrease carb intake

🍓DM Renal screen:=microalbuminuria🍓


🌹 DM microalbuminuria, how to confirm?
A. Us B. Biopsy C. repeat urine analysis D. 24-h monitor microalbuminuria
🍓55 years old , male , have a family history of diabetes, HgA1C is 6.3 , fasting 7.3 what will
you do next? A. Random glucose B. Fasting after 3months ✅ (pre_diapetic)
C. HgA1C after 6 weeks D. 2 hours glucose
🌹Patient just diagnosed with diabetes type 2 , HbA1c 7.8 how do you start therapy ?
A. Diet and exerciseB. Diet and exercise plus metformin✅C. Sulfonylurea
🌹25y old male with c/o of excessive wt gain since 5 ys wt 90 bmi 34 examination
normal = diet and exercise.
🌹A patient presented with the classical triad of polydypsia, polyphagia and polyuria. A
diagnosis of Diabetes Mellitus was made. Which of these changes is observed in Diabetes
Mellitus? Impaired phagocytosis🌹
🌹A 50-year-old woman presented with sudden onset shortness of breath, PND and
feelings of drowning. On examination, she has pulmonary edema and s3 gallop. She is also
a known case of Diabetes Mellitus and was currently on metformin for glycemic control.
Which of these medications is considered safest for glycemic control of this patient?
SGLT2 inhibitor ✅🌹= sgLT2 = gallop s3 .=‫رﺑط‬
‫ﻂ‬Ï‫ر‬
🍓Elderly K/c case of long time poorly controlled DM2 and hypertension she is on aspirin
atenolol metformin ,insulin, multivitamin She complains of dizziness when she stands what
is the cause In examination There is decreasing of BP when standing but the HR *didn’t
change* A. Side effects of the medication. B. Autonomic neuropathies✅✅
🍒Case of dm in undiagnosed child with clear symptoms what's the most important
investigation: A. Urine dipstick B. H1ac ✅
🌹Patient with type 2 DM on Metformin, his hga1c is 7.8, what’s your target of HgA1C in his
case ? A.<5 C.<6 D.<7✅
🌹What’s Is the relation between DM and obesity ? A. Odds ratio B. Relative
🌹 Diabetic patient Elderly came with ingrowing nail, First thing to do:
= s Check peripheral pulse
🌹1ry prevention of DM? A. Diet control B. Screening

DKA
🌹Patient with severe diarrhea and vomiting. Labs provided showing metabolic acidosis.
whats sign is observed clinically? A. Kussmaul breathing✅ B. Low urine output
🌹An obese 15-year-old girl is brought to the emergency department by her parents due to
excessive thirst and urination for 1 week. Past medical history is unremarkable. Family
history is positive for type 2 diabetes in her father and sister. Physical examination reveals
moderate dehydration. Her random plasma glucose is 395 mg/dL. Plasma or urine ketones
are negative. Which of the following is the next investigation of this patient condition?
A. Autoantibodies to glutamic acid decarboxylase = its happen in
type 1 diabetes✅
🌹glycosuria + ketonuria + Sx of hyperglycemia = DKA start IV NS
🌹DM type 1 already in insulin ( glargine and another one) now came with DKA start with IV
which insulin give : A- IV insulin + long acting B_Regular
Regular‫ ﻧﻌﺎﻟﺟﮭﺎ ﺑﺎﻟﺗرﺗﯾب‬DKA ‫رﺑط= ﻟﻣﺎ ﻧﺷوف ﺣﺎﻟﺔ‬
🌻Start IV regular insulin 0.1/ kg
🌹 DKA patient, on iv insulin, be careful of? A. Hypoglycemia C. Cerebral edema ✅
🍒 case of DKA and Dr. give him ringer lactate and low insulin dose, What are you afraid of?
A. Cerebral edema ✅ B. hypoglycemia
🍓Child admitted by DKA and after receiving Iv fluid he is confused what is the diagnosis ?
Cerebral edema
🌹DKA case what the complication? Cerebral edema
🍓 DKA aftet IV fluid, what to do next? Give insulin🍓
21)_ Pt with symptoms of DKA and labs suggest DKA also was given in fluids. next: IV insulin
🌹DKA patient came to ER the initial step?
A-insulin and dextrose B-calcium gluconate C-sodium bicarbonate D-IV fluid ✅
🌹pt with DKA he starts to breath rapidly to buffer his acidosis through= carbon dioxide
(Dont choice monoxide) .
🌹Child DKA symptoms how to prevent: Listen to the child what he is worry about🌹
🌹Dm 1 diagnosed after DKA now blood sugar is stable and she is ready to be
discharged which regimen
A. - Only short acting without NPB B. - Short acting before meals and glargine
🍓Patient had strong hx of T2DM and had high fasting glucose and Hbalc of
6.5 asking about next step A. OGTT ✅ B. Hbalc after weeks C. Repeat fasting
🍓34 y /o Healthy man ask about risk of ischemic heart disease?
A. BM131 B. BP 135/80 C. two fasting glucose reading 9.5 then 9.4mmol ✅✅
1Thyroid , parathyroid
🌹Case of Parathyroid adenoma with hypercalcemia and low Phosphate, how to manage?
1-Rehydrate and stains 2-Rehydrate and diuretics ✅
3-Biphosphonate and diuretics and stains
N.b if mild to moderate rehydration and decrease oral calcium is enough
Diuretics = for sever hypercalcemia
If with bone changes or osteoporosis bisphosphonate
🌹Parathyroid adenoma with hypercalcemia and low Phosphate = Rehydrate and
diuretics🌹
🌹 Case of primary hyperparathyroidism. What is an indication to undergo elective
parathyroidectomy? - Age > 50 - High PTH. - Signs of osteoporosis✅
🌹 subclinical thyroiditis is hypo or hyper ? #subclinical thyroiditis is= hyperthyroidism
🌹 Pt with graves and eye symptoms, wt will increase eye dz in graves?
A. High free T 3,4. B. Male. C. Smoking!@

🌹2/25 years female with thyroid nodule TSH and T4 normal , FNA cytology done and
according to Besthesda classification ( Stage III ) what is the most accurate management :
a. hemithyroidectomy b.total thyroidectomy c. repeat FNA and observation ✅✅
d. radio active iodine
(( N.B: Betheseda score: 1,2,3 >> repeat FNA and observ. 4 >> lopectomy
5,6 >> total thyroidectomy )))
. (( ‫ ﻧﺷﯾل ﻛل اﻟﻐدة ﻓﺎﻟﺑﺗرﺗﯾب ﻧﻣﺷﻲ ﺣﺗﻰ ﯾﺳﮭل اﻟﺣﻔظ‬٥ ‫ ﺑﻌدھﺎ رﻗم‬، ‫ واﺣد‬lope ‫ ﻧﺷﯾل ﺟزء او‬٤ ‫ ﺑﻌدھﺎ رﻗم‬، ‫ﻧﻣﺷﻲ ﺑﺎﻟﺗرﺗﯾب اول ﺷﻲء ﺑدون ﺟراﺣﺔ‬
🌹pt with hypothyroidism on thyroxine 175 microgm for 10 months dose increased to 200
after that lab showed TSH high , T4 normal. what is the diagnosis
a.sub acute thyroiditis. b.hashimoto thyroiditis✅ c. sick euothyroid syndrome
، ‫ ﻋﺎﻟﻲ ﻣﻌﻧﺎھﺎ ﻓﻲ ﻧﻘص اﻓراز ﻏدة اﻟﺛﺎﯾروﯾد‬tsh ‫ ﻻن ﻟﻣﺎ ﯾﻛون‬Hypothiroid ‫ﻛون ﻟﮫ ﻋﻼﻗﺔ ب‬d ‫ ﻋﺎﻟﻲ ﯾﻌﻧﻲ ﻧدور ﻋﻠﻰ ﺟواب‬TSH ‫🌹طﯾب ھﻧﺎ‬
(( ..‫ ھﺎﺷﯾﻣوﺗوا ھذا ﯾﻌﻧﻲ ھﺟوم ﻣﻧﺎﻋﻲ ﻋﻠﻰ اﻟﻐدة ادى اﻟﻰ ﺗدﻣﯾر اﻟﻐدة وﺑﺎﻟﺗﺎﻟﻲ )) ﻧﻘﺻﺎن اﻓراز اﻟﻐدة‬، haahimoto ‫ﻓﺎﻟﺟواب‬
🌹Pt with. Hypothyroidism on low dose hormonal therapy, still complaining of symptoms?
A. Increase the dose + recheck 3 wks B. Increase the dose + recheck 6 wks
C. Decrease the dose + recheck 3 wks D. Dec the dose + recheck 6 wks
🌹Pt Newly diagnosed with Hypothyroidism, after 2 wks, TSH 7.5, what to do?
A. -increase dose. B. -continue the same dose F/U 1 month

🌹Women have tremor and palpitation and irregular irregular pulse dx as a fib by
ecg what to do first? Tyroid Function Test
🌹unialateral neck swelling in the RT side by investigations :hot thyroid nodule the rest
cold thyroid No LN enlargement Treatment? Start atinthyroid meds, if failed radioactive ☢
, the best definitive ttt is surgical excision✅
🌹Pt with constipation, behavioral irritability wrist x ray showed bone erosions. On lab:
PTH elevated, Ca elevated. Appropriate management:
A. Hydration, diuretics, bid phosphate✅. B. Hydration, diuretics, statin
🌹Female with calcium high, PO4 low vitamin D low. = A. 1ry hyperPTH B. 2ry hyperPTH
‫ ﻣن اﺳﻣﮭﺎ ﺳﻛﻧدري‬،‫ ﯾﻛون اﻟﻛﺎﻟﺳﯾوم واﻟﻔوﺳﻔﺎت ﻋﺎﻟﻲ‬scondry ‫اﻟﺑراﯾﻣﺎري ﯾﻛون ﻛﺎﻟﺳﯾوم ﻋﺎﻟﻲ واﻟﻔوﺳﻔﺎت ﻣﻧﺧﻔض ال‬
. ‫ﯾﻌﻧﻲ اﻻﺛﻧﯾن ﻋﺎﻟﯾﯾن‬
🍓patient with fracture , very high PTH w calcium was at the upper limit of the normal and
phosphate was normal What is important to do :
A. 24 h urine ca , B. sestimabi scan ✅ C. ALP
Sestamibi scanning is the preferred way to localize diseased parathyroid glands prior to an
operation. ... This very mild and safe radioactive agent is injected into the veins of a patient
with parathyroid disease (hyperparathyroidism) and is absorbed by the
overactive parathyroid gland.
🌹Recurrent fracture + high PTH + multiple renal stones. Investigation?
C. Ca urine D. IVP E. ALP F. Sestamibi scan
Recurrent fractures + renal stones are absolute indication for surgery

🌹Parathyroid hormone is high + high calcium And low vit D = primary parathyroidism
cause PTH and Ca high.
🌹 Patient with a history of kidney stone 2 weeks ago His labs showed hypercalcemia ,
hyperparathyroid , hypophosphatemia Urea and creatinine elevated What is the cause of
hypercalcemia ?
A-Primary hyperparathyroidism B-Secondary hyperparathyroidism
C-High 1,25 dihydroxycholecalciferol
‫ ﻟو اﻻﺛﻧﯾن ﻋﺎﻟﯾﯾن اﻟﻲ ھم اﻟﻛﺎﻟﺳﯾوم‬primary , ‫ واﻟﻔوﺳﻔﺎت ﻣﻧﺧﻔض اﻻﺛﻧﯾن ﺿد ﺑﻌض ﯾﻌﻧﻲ‬hyppercacium ‫ﻻن ﻗﺎﻟك‬
scondry....‫واﻟﻔوﺳﻔﺎت ﻣن اﺳﻣﮭﺎ اﻻﺛﻧﯾن ﯾﻌﻧﻲ‬
🌷bone resorption asking about the cause, they gave increased PTH and Ca + high
phosphat ? Tertiary hyper parathyroidism🌷
🌷secondary hyper parathyroidism = (high PTH+low ca + high phosphat) .
🌹Patient with a BMI of 40, she doesn’t c/o visual disturbance, no hypothyroid symptoms,
she came with lab works of: TSH high, prolactin normal, FSH and LH are abnormal. What is
the possible cause? A.non-functioning prolactinoma B. Hypothyroidism ✅✅🌹 C. Morbid
obesity
🍒clinic signs of hypothyroidism and TSH 5 , which is in the normal range. best next step to
diagnose her thyroid disease? A. repeat the test after few months. B. T4 ✅
🍓A 50 year old patient presents with complaints of insomnia and palpitations. He had a
history of Afib to which he takes Amiodarone. He is on SSRI, ACE and 2 other medications.
Upon examination everything is normal, BP normal, HR normal, What is your next step?
A. Add B-blocker. B. Check TSH and T4✅. C. Change his psychiatric medications
D. Refer to psychiatry clinic
🍓management of Graves’ disease with exophthalmos not responding to antithyroid for
10 months:
A. Subtotal thyroidectomy✅ B. Near total thyroidectomy
C. Radio iodine therapy D. Continue antithyroid medication
🍓Hashimoto’s disease associated with :
A. lymphoma✅ B. Papillary cancer C. Follicular cancer D. Medullary cancer
🍓Elderly with signs and symptoms of hypothyroidism he had a history of radiation before
3 years .thyroid examination was normal in labs TSH 5 (0.4-5) What will you wil do to
establish the diagnosis?
A. REPEAT the TSH after 4weeks B. Free T4✅✅ C. Thyroid us D. Thyroid scan
🍒Pt with dyspnea and dysphagia symptoms increased during sleep with neck enlargement
dx: A. goiter✅. B. lymphoma. C. thymoma

🌹Palpantion + neck discount with TSH high + T4 low + ESR high : subacute thyroditis
🌹Patient known with hypothyroidism presents to your office accusing no symptoms but
showing you lab results where TSH levels are elevated. She has been on thyroid hormone
replacement therapy initiated two weeks ago. Which of the following is the correct
approach? A. Continue same dose and follow up in 4 weeks ✅✅🌹🌹
‫ =ار` ـﻊ اﺳﺎﺑﻴﻊ‬4 ‫ﻪ‬x‫ﺸ‬ž ‫ﺪ ﺷ”ﻠﻬﺎ‬w‫ر`ﻂ =ﻏﺪة اﻟﺜﺎﻳﺮو‬
🍓Female in mid 40 came with neck swelling for 2 months, with decrease in weight
complains of stoped menstrual cycle for 6months and milky discharge , what investigation
you will order ? A. Thyroid US✅ B. Ovary US C. Mammogram D. Ultrasound something..

🍓euthyroid syndrome, reading of T3 T4 and reverse T3 ?Low t3 t4 but high rt3


🍒 euthyroid infection. what will her TSH T3 &T4 levels!? all low except high rT3
🌹 What is an Euthyroid Sick Syndrome? A. Normal T3 reserve and low T3,T4

......
🍓DKA complicantion? cerebral edema🍓
🌹Diabetic tight control for past year Hba1c 5.8 before that Hba1c was 9-10, complaining of
leg pain and loss of sensation, asking about tx? Vitamin B12 ✅
🌹One additional question about vitiligo patient, what further investigation you would ask
for? Intrinsic factor level✅
🌹Patient with macrocitic anemia and preipheral neuropathy low B12 normal folic acid
what should you check? A-TFT. B-LFT. C-intrinsic factor ✅✅. D-KFT

🌹Alcoholic with + babinski sign + blood smear was given ( couldn’t find a similar picture)
What is the cause? A. Alcohol B. Hypothyroidism C. B12 deficiency ✅✅ Babi = B12
🌹Pt complain of tingling sensation which of the following is decrease ? B12
🌹pt generalized fatigue and palpitations with vitiligo and absent reflex and low
B12 Low HB normal folic acid ? A.oral b12 B.oral ferrus C.folic acid D.parenteral B12✅

🌹Medullary thyroid carcinoma management = Total thyrdoctomy 🌹


🌹case asking of Diagnosis of DM1 ? HbA1c 🌹
🌹thyroid nodule US done and TSH done whats the next = isotope scan and bone scan .🌹
🌹after chemotherapy electroly imbalance = hypocalcemia 🌹
Patient known case of atrial fibrillation and depression which take fluxetine‫ﺬا دواء‬¹ and
amiodarone‫ﺸﻦ‬ã‫اﻟ‬²ÿ‫ﺎل ﻓﻴ‬wv‫ دواء ﻟﻼﺗ‬for it came with palpitations lethargic, insomnia.. how
you will manage? A. Give BB B.Measure TSH and T4 ✅
🍒year old patient with absent red reflex (picture) what is your action= urgent referral to
ophthalmologist 🍒
🌹 type visual defect come with pituitary adenoma = Bitemporal hemianopia🌹
🍓Pt with signs of hyperprolactinemia , MRI shows 0.7 mm pituitary adenoma ,
what’s the TTT: A. Surgery. B. cabergoline✅
🍓pt with sign of pituitary adenoma , visual defect: bitemporal hemianopia
🍒 pt came with low libido and low activity BMI 40 ,refer to endocrinologist FSH LH TSH T3
low ,MRI brain show 2.5 cm pituitary adenoma what's most likely dx?
A. morbid obese B. non functioning pituitary adenoma✅

.
🍒high tsh, normal t4, high ESR ? Subclinical thyroiditis
‫ اﻗﻮل ان ﻫﺬا اﻟﺸﺨﺺ ﻋﻨﺪە‬ýÓ‫( ﻣ‬subclinic hypo) ‫ اذا „ﺎن‬TSH ‫ﻦ‬Ä‫ وﻟ‬، ÐŽ‫ ﻋﺎاا‬T4 Or T3 ‫ وﻟ¨ﺲ ﻣﻨﺨﻔﺾ‬þŽ~‫ﻃﺒ|ﻴ‬
ŒŽ • ‫ﺲ‬7 ‫ اﻟﻔﻜﺮة‬، ÐŽ‫ او ﻋﺎ‬TSH ‫ ﻳﺘﺎﺛﺮ ﺑ|ﻨ•ﺎ‬ÐŽ‫ ﻫﻮ ا‬T3 .4 4 Ó
Ž ‫ ﻧﻔﺲ ﻣﺎ‬ïy‫ ﺗ‬. 🍒🍒
Subclinical hypothyroidism occurs when TSH levels are slightly elevated but T3 and T4 are
normal.
🌹 Subclinical hypothyroidism Asymptomatic , everything is normal :
A. start treatment B. wait abd reevaluate after 6M✅

🌹Patient taking levothyroxine. Labs showed patient still hypothyroid. What's your
management
A. Increase dose and follow after 3 week
B.increase the dose and follow after 6 weeks ✅✅
‫ﻂ‬Ï‫ = ر‬thyroid =‫ اﺳﺎﺑﻴﻊ‬٦ ‫ﻌﺪ‬u ‫ﻌﺔ‬u‫ ﺣﺮوف =ﻣﺘﺎ‬٦
....................

🍒Young patient low BMI and short with no hair growth or breast budding. Which type of
pubertal delay? A- chromosomal. B- constitutional✅🍒
‫و|ﻤﺎن‬. ‫ﻈﻬﺮ اﻟﻄﻔﻞ ﻗﺼ_^ اﻟﻘﺎﻣﺔ‬w‫ﻤﻞ ﻃﻮل ﻓ‬À‫ ﺗﻮﻗﻒ اﻧﻬﺎ ﺗ‬، ‫; ﻧﻤﻮا ﻟﻠﻌﻈﺎم ﻧﻔﺴﻬﺎ‬: < ‫ ﻫﻮ ﺗﺎﺧﺮ ﻣﺆﻗﺖ‬، ‫; ﻧﻤﻮا وﻃﻮل اﻟﻄﻔﻞ‬: < ‫ﻫﺬا اﻟﻨ»ع ﻣﻦ اﻟﺘﺎﺧﺮ‬
‫ ﻋﻼﻣﺎت‬puberty ‫ﺗﺘﺎﺧﺮ‬
Constitutional delay : (CGD), the most common cause of short stature and pubertal delay, typically have retarded
linear growth within the first 3 years of life. .
………………….
diabtic and macrocosmic child and has deformity which kind of deformity (hand )baby has :
= Erbs palsy
..............
Pt with oseoarthritis and with cervical degenarative dos and did cervical discectomy now
he has (bladder uncontrol ) = what is the cuses ??
cervical discectomy ( Not quda eqina Bec case not say there is pain radiat to leg , loss
senstion , loos stool ...etc )
. . . .. .
🌹Benigh prostatic hyperplasia = Alpha block🌹

Hyperprolactinemia
🌹Pt with history of thyroid nodules also she have milky discharge from the breast and
amenorrhea since 6 m lab results show that TSH high t 4 high What to do next?
A.MRI ✅ ( To exclud pitotary adenom) B. US thyroid
🍒 Patient had TSH high, T4 high what images to do next?
A. MRI brain ✅pitotary gland tumor B. thyroid ultrasound

🌹 Married 3 years ago with irregular menses and milk expressed manually .. next step:
A- b-HCG B- prolactin✅✅ 🌹irregular menses and milk expressed = prolactine level 🌹
🌹 pt complain of erection, Thyroid hormone normal
A- hyper pituitary C- macro adenoma prolactinoma D- micro adenoma prolactinoma✅
🍬Micro adenoma > hyperprolactin > erectile dysfunction
🌹 female patient care with bitemporal hemianopia and obstructive symptoms
(prolactinoma) *definitive* management A- transsphenoidal resectionÖ B. bromocriptine
🌹Patient on management for hyperprolactinemia .. when to indicate brain mri=
A. Blurry vision B. Double d value of prolactin C. Bilateral breast discharge
🌹Pt with symptoms and sign of hyperthyrodism and prolactenemia . Labs show elevated
TSH and T4 . Next step in mx :- Answer is: Brain MRI 🌹
🍒Pt with history of thyroid nodules also she have milky discharge from the breast and
amenorrhea since 6 m lap result show that TSH high t 4 high Prolactin level 6000 very high
What is the most appropriate management?MRI

Adrenal disease
🌹Long scenario with low BP and hyperkalemia + hyponatrima dx = addisons diaeas (
adrenal insuffincy)🌷
🌹Long scenario with High BP and hypokalemia + hypernatrimia dx = primary
hyperaldosteronism🌷
🍒Male patient medical free , BP 150/90 Lab: normal na , low k ask about Dx ?
A-Primary HTN B-Primary hyper aldosterone ✅. C-Addison disease
🌹Female complain of fatigue and she has scar turned hypo pigment (dark) , Addison case
he ask what test to do= synth Test
🌹woman who had a wound , started to get very dark. Hypotension too First test : A.
synacthen B. cortisol C. dex suppression
🌹Female patient who has dizziness when standing and noticed a scar on the back of the
hand that is getting darker( only these symptoms) , how to confirm ?
A. Cortisol level B. synacthen test ✅ C. suppression test
🍒 Female with hypotension ,hyper k+ , hyponNa , dark line in hand scar what next?
A-dexamethasone suppression test B-cortisol level C-syntchan test D- Morning serum
cortisol levels
_Best initial test == Morning serum cortisol levels: < 3 μg/dL.
_Confirmatory test == ACTH stimulation test (cosyntropin test

🌹Addison disease = is adrenal insufficiency = (hypoadrenalism) = all function of adrenal


decrease (low cortison = presention = hypotension, hyponatremia + hyperkalemia +
Hyperpigmentation = Treatment= Replace steroids with hydrocortisone.🌹
Diagnosis mainly by= ( cosyntropin test) .
🌹 The most specific test of adrenal function is the cosyntropin test.
🌹 Acute adrenal crisis : presents with profound hypotension, fever, confusion, and coma.
Cosyntropin Stimulation Test
🌹 2 cases of cong adrenal hyperplasia Acidic, low glucose, low NA, high K What to give in
tx? glucocorticoids & mineralocorticoids (Dexteros + steroid )
🌹 Female elderly dm and asthmatic on steroid undergone surgery and after surgery
admitted to icu hypotension low na high k hypoglycemia what is the cause:
A. adrenal insufficiency B. DKA
🌹 Female pt asthmatic using steroids in ICU, labs given Na low= Adrenal insufficiency
🌹A woman who is known to have steroid dependent bronchial asthma undergoes surgical
resection of colorectal cancer following the surgery and she is shifted to the ICU due to
instability labs were given potassium and sodium levels glucose I potassium was 6 sodium
was 130= adrenal insufficiency
🌹Patient presents abdominal pain was investigated by ABD CT 2cm adrenal adenoma
what to do next : -Metanephrines and cortisol level.✅
Increased metanephrine and normetanephrine levels are found in patients with pheochromocytoma
🌹Na Low, BP low, K high, surgery hx? A- adrenal insufficiency
🌹female child with breast tissue engorgment, congenital adrenal hyperplasia
excluded, has adult like features, what investigation? A- pelvic us
🌹Before adrenal gland operation, what should you give the patient? Hydrocortisone ✅
🌹 Child had recurrent vomiting, skin turgor muscle wasting The external genetalia is
abnormal of size of the clitoris pediatrician، Labs showed na120 started on sodium
chloride What is the next management: (Congenital adrenal hyperplasia)
A-K5 B-NA Hco3 C-Hydrocortisone✅✅ D-Ceftaxime
🌹 Signs and symptoms of classic CAH In females=enlarged clitoris or genitals
🌹 Non functioning adrenal adenoma, 5cm and causing vague abdominal pain how will u
manage? Adrenalectomy ✅ Observation
🌹 Which of the following in incidentally found in radio imaging?(regarding
adrenal gland ) A. nonfunctional adenoma B. adenocarcinoma
🍒Female do ct scan and accidentally found adrenal mass asymptotic what is the diagnosis
? nonfunctioning adenoma ‫🍒 ﻣﻦ اﺳﻤﻬﺎ‬
🌹Child with hyperpigmentation + petechia hyperkalemia + hyponatremia what to give? -
Cortison
🌹 Brown discoloration of gum and stias, high K and low Na what is the ttt?
A. Hydrocortisone daily

🍒htn and abdominal swelling and palpitations and inv has high catecholamines (2100)=
alpha blockers
🍒HTN don't respond to medication adrenal mass = alpha blocker 🍒
🍓pt with hypertension 220/110 not responding to home medications,
abdominal CT show adrenal mass , catecholamine is very high , asking about
management : A. alpha blockers ✅✅= pheochromocytoma
🍒patient with hypertension, hight catecholamines and *adrenal mass* on US, what is the
treatment: A- alpha blockers ✅pheochromocytoma. B- beta blockers C- CCB
‫ﻂ‬Ï‫ = ر‬adrenal
🍒 (lethargy, hyperpigmentation). Confirmation of addison disease?
A)Low-dosed exame tha son suppression test. B)_ Urinary cortisol C)_ Synacthentest(ACTH
stimulation test) ✅
🌹Orthostatic hypotension, weight gain 5kg per week dizziness, scar in her hand, dark scar
in examination, what investigation confirm diagnosis?
A.Low dose dexa test B.Cortisol level C.24 h free cortisol, urinary D.Synchian test ✅
Cushing disease
🌹 Refractory hypokalemia?
A. Asses aldosterone and renin ratio B. Start spironolacton
🌹 20y old athletic gain 25 kg in the last 4 month with highly muscolanized upper
limb n acne ? steroid anabolic
🌹A 32-year-old woman presents with irregular menses for the last several years. She has
gained 18 kg over the past 2 years and feels that most of the weight gain is in her face,
neck, and abdomen. She also notes bruising without significant trauma, and difficulty
rising from a chair. She was diagnosed with type 2 diabetes and hypertension a year ago.
Which of the following is the first-line investigation for this patient condition?
A.24-hour urinary free cortisol ✅✅🌹
🍒A patient presents with signs of Cushing disease, Labs: ACTH low, Cortisol 8 AM high CT
pelvis: 5 CM mass in adrenal gland What is the most appropriatemanagement?
A. Surgical resection✅ B. Mitotane therapy C. Pituitary transsphenoidal surgery
🍒Preoperatively what to give a patient known case of Cushing syndrome? Steroid
🍓Cushing with 24h cortisol high. ACTH was low. Next
A. MRI B. CT pelvic✅
If ACTH high > go for high dose dexa to differentiate between ectopic Cushing or pituitary
If low > CT adrenal
‫ ر`ﻂ =اﻟﻐﺪة ﺗﻘﻊ ﺗﺤﺖ‬lower .
🌹Pt with cushing's signs and symptoms, positive 24h urine cortisol , lab shows
low ACTH, your management:
A. brain MRI(if high ACTH) B.Abdominal CT C. overnight dexamethasone test
‫ﻂ‬Ï‫=ر‬ACTH low=‫ ﻣﻊ‬Lower ‫ ﺗﺤﺖ‬ýŽ • ‫ =•ﻌ‬abdomen
‫ ﻟﻮ ﻗﺎل‬high ACTH = ‫ ﻓﻮق =ﻧﻌﻤﻞ ﻟﻪ‬ýŽ • ‫ •ﻌ‬brain

🌹Cushing syndrome acth low what is next A. Us abdomen B. Ct brain


🌹central obesity moon face = ACTH level = Low dose dexamethasone supression test or
24h urinary cortisol 🌹
🌹Pt known cushing's syndrome stop cortisone 1 year then he has symptoms of
cental obesity, stria ACTH low, one gram dexametha suppress low what most
appropriate to do? A. Mri brain. B. Ct abdomen. C. Transsphenoidal resection
D. Sample from nose to pituitary
Hypercortisolism (Cushing syndrome) Presentation:
• Fat redistribution: “Moon face,” truncal obesity, “buffalo hump,” thin extremities,
increased abdominal fat
Diagnostic Tests : Establish the Presence of Hypercortisolism
‫ون وﻻ ﻻ؟‬²ƒ § ‫ﺾ ﻣﺼﺎب ب ارﺗﻔﺎع اﻟ–ﻮرﺗ‬wv‫ﻞ اﻟﻤ‬¹ ‫ء‬ÔÕ ƒ
o ‫ﺸﻮف اول‬ï îo ‫“ﻌ‬
🍒best initial test is(24-hour urine cortisol)
If this is not in the choices, then answer is the 1 mg overnight dexamethaso suppression
Establish the Cause of Hypercortisolism
ƒ
‫اﻻن اﻟﺨﻄﻮة اﻟﺜﺎﻧ}ﺔ اﺟﺘﺎج اﻋﺮف ﻣﺎ‬، ‘o‫ون ﻋﺎاا‬²ƒ § ‫ دﻣﻪ ﻛﻮرﺗ‬mo ‫ﺪﻧﺎ اﻧﻪ ﻋﻨﺪە‬é‫ﺎ {ﻌﺪ ﻣﺎ ﻋﺮﻓﻨﺎ اﻟﺸﺨﺺ وﺗﺎ‬¹‫ﺬا ﻣﻌﻨﺎ‬¹
ƒ ƒ
‫ ﻏﺪة‬mo ‫ وﻻ ورم‬، ‫ﻨﺎل‬w‫ اﻟﻐﺪة ادر‬mo ‫ﻮ ورم‬¹ ‫ﻞ‬¹ ‫ون ؟‬²ƒ § ‫ﻮ ﺳ!ﺐ ارﺗﻔﺎع اﻟ–ﻮرﺗ‬¹ pitotary ‫؟ اﺣﺘﺎج اﻋﺮف ﻣﺼﺪر‬
‫ون ؟‬²ƒ § ‫ارﺗﻔﺎع اﻟ–ﻮرﺗ‬
ACTH testing is the best initial test to determine thecause (source) or location of
hypercortisolism.
Treatment: Surgically remove =. Transsphenoidal surgery is done for pituitary sources
whereas laparoscopic removal is done for adrenal sources.
...........
🍓Cushing's sx with new onset DM. Best next step? A- Pituitary MRI B- cortisol level✅✅
🌹Pt with psoriasis treating in with steroids and presenting with s & s of cushings what is
the cause: A. Iatrogenic = à • •
Ž £ ‫و–ﺪ ﺧﺎر‬Òç‫–ﺾ اﺧﺬ ﺳﺘ‬À‫ ﻫﻮ ان اﻟﻤ‬ñç‫ﻮﺷ‬Ä‫ﺐ ﻣﺮض اﻟ‬Þ‫ ﺳ‬ýŽ ‫•ﻌ‬

🌹Cushing case what is the initial test to diagnose: dexamethasone suppression test or
24h urine cortisol
....
🌹Cushing case what is the confirmatory test: 24 urine cortisol or dexamethasone
suppression or late night salivary cortisol
....
🌹A 60-year-old female presented with hypertension, proximal muscle weakness, weight
gain especially in the upper body and purple striae on the lower abdomen. Which of these
tests gives a clue about the etiologyof the most probable diagnosis?
A. High-dose dexamethasone suppression test ✅💐💐🌹
B. 24-hour urinary free cortisol
C. Low-dose dexamethasone suppression test
D. Midnight salivary cortisol
‫ﻂ =ﻋﺸﺎن ﻧﻌﺮف ﺳ!ﺐ اﻟﻤﺮض‬w‫ ر‬etiology = ²§‫ﻴ‬Û‫ = “ﺤﺘﺎج {ﺤﺚ ﻛﺜ‬High dose .

🌹🌹Case about resistant glucose tolerance with high glucocorticoids level, what is the
diagnosis? Cushing Syndrome

.... .... .....

1Heptic dis
🌹30 yo Female patient medically free suddenly start feeling fatigue and
jaundice. She denied drug use. Liver examination not remarkable. Laps
show ALT + AST more than (2500) “sure” , Albumin low All liver
enzymes very bad What to do ?
A. send her home B. admit + Iv fluid + analgesic C. observe in ER D. refers her to liver
transplantation
🌹 30 Y.O female with jaundice and yellow sclera started 2 weeks ago. No flapping tremor
or stigmata of liver failure. LFTs: High AST(1400), ALT(1300), GGT (70?), Bilirubin High
direct and. Indirect. US: biliary strictures and dilatation. Next step in management? A.
discharge with F/U. B. admit and start iv dextrose. C. order PT & INR
D. refer to liver transplant

🌹Young female has jaundice , leathergy , arthralgia , on labe result high LFt and alkaline
phosphatase What is the diagnosis A. Autoimmune hepatitis B. Primary biliary cirrhosis

🌹 Pt came for routine clinic known case of Chronic hep B pt US shows


coarse texture liver ( i think means liver cirrhosis ) his LFT mild
elevation pt is stable and asymptomatic
A. No further management just follow up. B. Liver transplant. C. entecavir
First line treatment for hep B is entacavir or tenovir or peglyated interferon
If peglyated interferon and entacavir in choices go for entacavir

🌹Non-alcoholic hepatostatehorrea what would be ur best advice


A. Stop smoking B. Reduce wt C. Give him statin

🍓HBsAG negative, Anti-HBc & Anti-HBs positive:


A)_Immune because of vaccines of HBV B)_Immune because natural infection✅
C)_Chronic infection. D)_Acute infection
🍓what is the meaning of the following liver serology:
HBsAg *negative*, anti-HBc positive and anti-HBs positive:
A- immunized due to vaccination B- Immunized due to natural infection ✅
C- acute infection D- chronic infection
🍓what is the meaning of the following liver serology:
HBsAg *positive* , Anti-HBc positive IgG: A)_ acute infection B- chronic infection ✅
(HB*s*Ag po*s*itive = chronic infection .
HBsAg *negative* = natural infection)

● The first marker that indicate acute hepatitis B infection is ?
A. HBsAg. B. HBeAg. C. Anti HBc IgG

Nurse who treat the pt with hepatitis B and get hurt by the needle what to do ?
A. IVIG and hepatitis b vaccine

🌹Acute chronic hepatitis b infection marker? = Core antibody (IgM and IgG)🌹

🌹Core + igG and E positive. Is this?
A. acute hepatitis B. Vaccinated and recent exposure C.Chronic hepatitis = IgG
.........
🌹Anti Hbs: postive Anti hbC : pstive Antigen Hbs : positive Igm : negative
=== chronic infection
🌹patient with positive antibody to HBV but negative HBV DNA(Hepatitis B virus) What to
do?A. Nothing✅✅. B. Repeat test. C. LFT. **DNA = negative= Nothing**

🍓hepatitis c type 4 management = Sofosbuvir / Ledipasivr 🍓


S= C
🍓First indicator of acute hepatitis B infection: A. HBsAg✅ B. HBeAg
..
🌹Probably A, hepatitis A is transmitted by fecal-oral route not blood, and it was the only
option with IgM.
..
🌹 Patient is caucasian and symptoms started when he was introduced to regular food
Patient has no history of drug use or alcohol drinking, presented with vomiting, diarrhea
and jaundice. Which of the following indicates her infection? A. HAV IgM✅ B. HAV IgG

The patient is known to have hepatitis B. The nurse is known to be vaccinated and a full
responder. What will you give the nurse? A. Ivig vaccines✅ B. Booster vaccines .
‫¨ اﺧﺬت‬ v v %
& ‫ء •ﺸﻮف اذا‬u
& ‫ﺐ اول‬S‫ ﻃ‬vaccin ‫ء ال‬u
& ‫ اذا ﻣﺎ اﺧﺬﺗﻪ ﻓﻨﻌﻄﻴﻬﺎ اول‬، ‫ﻗ•ﻞ ﻛﺬا او ﻻ‬vaccin ‫ ﺧﻄﻮة ﺗﺎﺧﺬ‬6& ‫اذا اﺧﺬﺗﻪ ﺛﺎ‬
Ÿ% G nª‫اﻣﻴﻮن ﺟﻠ‬
….
🌹acute hepatitis acter restorant possible cause= A. . HAV✅. B. . HBV. C. . HCV. D. . HDV
......
🌹 Female HCV antibody positive HCV RNA negative, Healthy asymptomatic all
labs and LFTs normal= - no need for further testing

🌹Chronic Hepatitis B, symptomatic, laps show high AST ALT and US shows
micro-changes. How to manage?
A. peglyted interferon✅ for chronic Heb B and c B. observation and follow up C.Lamividrin
🍒pt known case of chronic HBV present with elevated LFT and direct and total bilirubin
What most appropriate treatment: A. Entecavir B. Pegylated interferon✅✅ C. Lam
🌹Pt with positive hepatitis C has lesion on liver = hepatocellular carcinoma
🌹Hepatocellular carcinoma screening for patient with Ascites:
A. Alphafetoprotin ( if not present us ) B. Triphasic CT
🌹Hepatocellular carcinoma Tx: Surgical resection

🌹liver nodules what is the cause?


A. Aflatoxin B. Lead toxin C.Hepatitis b ✅✅ D. Wilson disease
🌹Young male c/o fatigue, nausea, vomiting, loss of appetite, jaundice hx of eating from
outside what is the initial investigation:
A. Serum iGM anti-hAv antibody HebA ‫ﺎﺗﻴ÷ﺲ‬x‫ﻴ‬¹ A B. Serum IGg antibody HeBA C. Amylase
🍓pt with HCV ,, developed abd pain, CT done and show HCC which is 6cm =
A. Radiotherapy B. chemotherapy ✅ C. excision
..
🌹Patient referred due to positive HCV She has no history of blood transfusion or drug
abuse her investigations : Elisa positive anti HCV , and HCV RNA is negative , LFT is normal ,
what to do next ? A-Repeat test after 6 months B- Reassurance
.....
🌹 HCVAb+ and negative HCV pcr = Recoverd from infection 🌹 ‫ﻛﻦ اﺳﻤﻬﺎ ﻧ~ﺠﺎﺗ~ﻒ ﺧﻼص راح‬

🌹 Girl we completely free history of HCV ( no injection-blood transfused-no sexual contact


...) , She has HCV.Ag+ ..... Serology RNA appears-ve.. what is your action? Repeat the test

🌹Health worker had needle prick + risk of having HCV = 3% ‫ﺎﻟﻤ~ﺔ‬u ‫ﺛﻼث ﺣﺮوف ﻣﻊ ﺛﻼﺛﺔ‬
🌹 HCV and the contraindications of breastfeeding = If crackles in the nipples🌷
🌷hepatitis c drug = sofobuvir/ledipasvir = if not on choice select = Interferon🌷
🌹 Case of HCV Ab +ve but RNA -ve, not drug abuser pr blood transfusion:
. No need further Mx .
🌹 Most common type of genotype of HCV in saudi arabia ? 4 ( HCV = ‫ﺛﻼث ﺣﺮوف =ﻣﻊ‬
‫ﻌﺔ‬Ï‫) ﺟﻴﻨﻮاﺗﺎ•ﺐ ﺳﺎرت ار‬
🌹 known case of HCV with ascites + neutrophil 400 & other symptom , ttt:
A- IV antibiotics B- Paracentesis
spontaneous bacterial peritonitis = ‫ اﻋرﻓوا ﺗﺷﺧﯾص اﻟﺣﺎﻟﺔ‬250 ‫ ﻓوق‬neutrophill ‫اذا ﺷوﻓﺗوا ال‬
🌹Patient known to have liver cirrhosis and ascites, developed fever and abdominal pain +
high WBC , Next step ? A- antibiotics B- diagnostic laparoscopy C- urgent laparotomy
🍓Case of ascitis with liver cirrhosis lab : wbcs > very high No more information in this Q
Ask about management ? Antibiotics✅(Bacterial pertoinitis)
🌹 case with abdominal fluid and High SAAG what will u do?
A. Tapping B. antibiotics C. diuretics ‫ﻄﻦ‬y‫ﺎﻟ‬u ‫ ﻣﺘﺠﻤﻌﺔ‬ÐŽ‫ﻋﺸﺎن ﻧﻄﻠﻊ اﻟﺴﻮاﺋﻞ ا‬
🌹 Alcoholic patient with Cirrhosis have multiple hypodense lesions in the liver?
A-HCV B-Hepatocellular carcinoma ✅ ✔
HCV+Cirrhosis+Hepatocellular carcinoma * *‫ اﯾش ھم ؟‬، ‫ﻓﯾﮫ ﺛﻼث ﺗواﺋم ﯾﺟوا داﺋﻣﺎ ﻣﻊ ﺑﻌض‬
F* G ‫ﻌ‬2 ‫ ﺗﻠ"ﻒ‬A‫ﺪ ا‬.-‫ ﻣﻊ اﻟﺰﻣﻦ ﻳﺘﺤﻮل اﻟ‬، ( )
* ‫ﺼﺎب ب ﻓﺎﻳﺮس‬2 ‫ﺪ ﻟﻤﺎ‬.-‫(ء اﻟ‬
* ‫ ﻃ"ﺐ اول‬Cirrhosis ‫ ف‬، ‫ﺸﻔﻴﻨﺎ‬Q‫ﻌﺎﻓﻴﻨﺎ و‬2 O ‫ﺪ‬.-‫ﻃﺎن اﻟ‬M A‫ﻃ"ﺐ ﻟﻮ ﻣﺎ ﻋﺎﻟﺠﻨﺎ ﻫﺬا اﻟﺘﻠ"ﻒ ﻳﺘﺤﻮل ا‬
‫ﻘﻮﻟﻚ اﻟﺴﺆوال اﻧﻪ اﻟﻤ[\ﺾ ﻋﻨﺪە ﺗﻠ"ﻒ‬2 Cirrhosis A‫ ﻓﻤﺎ ﻋﺎﻟﺠﻨﺎ اﻟﺘﻠ"ﻒ ﺗﺤﻮل ا‬، (
* ‫ ﻻن ﻣﺎﺟﺎء ﻟﻪ اﻟﺘﻠ"ﻒ اﻻ ﻣﻦ ﻓﺎﻳﺮس‬، (
* ‫ ﻣﺎﻳﻨﻔﻊ اﺧﺘﺎر ﻓﺎﻳﺮس‬Hepatocellular carcinoma
، multiple hypodense lesions ‫ وﺧﺎﺻﺔ ﻋﻨﺪە‬xy‫ﺪون ﺗﻔﻜ‬v ‫ﺸﻮﻓﻮﻧﻪ‬u ‫ اول ﻣﺎ‬cirrhosis ‫ﻣﻌﻨﺎﻫﺎ‬

🌹 pt with hCV present with hx of weight loss , CT done shows irregul'r nodule th't occupies
70% of liver , wh'tʼs your next step ?CT MRI. Endoscopy. Perctounous biopsy ✅
. biopsy ‫ ﺛم ﻧﻌﻣل‬c.t ‫ ف اول ﺷﻲء ﻧﻌﻣﻠﮫ‬، liver cancer‫ ﺛم ﯾﺗﺣول اﻟﻰ‬liver cirrosis ‫ﻓﺎﯾرس ﺳﻲ ﻣﻊ اﻟوﻗت ﯾﻌﻣل‬
🌹 What improves mortality in liver cirrhosis with esophageal varices and
bleeding : A. octreotide. B. IV ceftriaxone!@
🌹 Patient k/c of HepC came with esophageal varices , ten asicitis and
drowsy . Which drugs will reduce mortality?
A. Pantoprazole infusion. B. Octreotide C. IV ceftriaxone D. Blood transfusion
🌹 Pt with sign & symptoms of liver cirrhosis + upper GI bleeding. HGB 8.5.
After stabilize the pt, What is the appropriate management?
A. Blood transfusion. B. Upper GI endoscopy. C. BB

1) what is absolute contraindication of liver transplantation:


a.acute hepatitis b.liver cirrhosis with active alcoholism✅
C.liver cirrhosis with encephalopathy
‫ ﯾﻌﻧﻲ اﻟﻛﺑد اﻟﺟدﯾد ﻛﻣﺎن راح ﯾدﻣرھﺎ‬، ‫طﺑﻌﺎ اﯾش اﻟﻔﺎﯾدة اﻧﻲ أﺟﯾب اﺣد ﯾﺗﺑرع ﺑﺎﻟﻛﺑد واﻟﺷﺧص ﯾﻛون ﻣﺎزال ﯾﺷرب ﻛﺣول‬
‫ ﺧﻠﻲ اﻟﻛﺣول ﯾﻧﻔﻌﮫ‬، ‫😂 ف ﻣﺎﯾﺳﺗﺎھل اﺑدا ﻧﻌطﯾﮫ زراﻋﺔ ﻛﺑد‬.
....... .......
🌹 pt elevated liver enzymes, jaundice, no hx of iv drug abuse. What is the investigation you
will order? A. HAV igM. B. HAV igG. C. HCV antibody. D. HCV antigen

🌹 negative blood or drug hx , jaundice and high liver enzymes all others normal ?
A. HepA igG. B. hepA igM. C. HBeAG. D. HCV antibodies

🌹 Pt with ascites when we take ascitic fluid sample for examination we found E coli what is
the cause? perforation
...
🍓pt with ascites and abdominal pain , E-coli was found in ascitic fluid , what’s mechanism
of bacteria to found in peritoneum “ no perforation” : = translocation

🌹Detecting HBV +ve pregnant women is the type of prevention?
A. Premodal B. Primary C. secondary
• ç ‫ = „ﻠﻤﺘ‬scondry
‫ﻂ‬Ï‫ = ر‬hepatitis B = ñ
...
🌹 hepatitis related symptoms with hep b serology findings (hep b surface antigen
positive and anti-core IgG positive) asks about his hep b status:
A. Chronic infection B. Previous exposure C. Immunization (Core + IgG = Chronic. )
🍒Hep B, surface Ag negative, CORE AB and surface AB positive
A.Immunized due to resolved Infection✅ B.Immunized due to vaccination C.Acute
🌹Hepatitis B acute infection marker? A. HBsAg B. HBeAg C. Anti-HBs
NB: HBeAg for activity and if positive indicate acute infection.
*‫ﻂ =ﺣﺎد =ﺻﻌﺐ =ص‬Ï‫ =ر‬S
....
🍒Pt with 11 m hx of abdominal pain fatigue presented with 2 wks hx of jaundice ? what to
ordi ? A-HAV lgm B-hHBV IGG ✅. C- HCV AB

🌹What type is Hepatitis B vaccine? A. inactivated B. live attenduated C. recombinan

🌹Pt chronic hepatitis B, came with jaundice and RUQ pain, lab show high liver
enzymes (ast>alt), most appropriate next step: Hepatitis D antigen/antibody
.....
Husband k/c of hepatitis B the wife is hep B -ve , she worries about sexual relationships:
What you tell her? 1) wear condom before ejection✅✅ 2 ) use OCP 3) reassurance

🌹Detecting hep B pregnants types of prevention?
A.Primary B.Primordial C.Secondary D.Tertiary
b‫ ﺷﻛﻠﮫ ﻛﺎﻧﮫ وﺣدة ﺣﺎﻣل‬B ‫ ﺣرف‬. A. B .c = scondry . ‫ ﺛﺎﻧﻲ اﻟﺣروف‬B ‫رﺑط = ﺣرف اﻟﺑﻲ‬
hepatitis B serology with positive anti HBc (igg) and HB surface antigen
A-Chronic Hepatitis B ‫ ✅ﻣﻜﺮر‬B-Acute hepatitis B
🌹hepatitis b serology with positive =Chronic Hepatitis B🌷
….
🌷hepatitis c drug = sofobuvir/ledipasvir = if not on choice select = Interferon🌷

Pt with Chronic hepatitis C and ascites ttt? A. spironolactone and durtics. B. Tapping
..
Schistosoma mansoni case, cardiac or pulmonary complication but can't recall the details.
Best diuretic for ascites? • Loop✅✅. • Thiazide. • Osmotic

🌹Spironolactone what type of diuretic? A. loop B. aldosterone


‫ﺎ“ﺪ ﻣﺎن =اﻟﺪوا‬x‫وا =ﺳ‬²§‫ ر`ﻂ =ﺳﺒ‬aldo ‫ﻠﻬﻢ “ﻘﻠﺪوە‬á .

🌹Hepatic cirrhosis due to hepatitis B + ascites + lower limbs edema + basal lung
crepitations + slight hypoalbuminemia + good renal function. Management?
A.Furosemide + spironolactone
🍒 long case on medication “insulin, ACE,spironolactone” patient have high potassium what
causes of high potassium? spironolactone ( drug induce hyperkalemia)
🌹Case typical with nausea , lethargy , vomiting , Lab are : HTN , high potassium,
low Na , Hypoglycemia what is the most appropriate step :
A. IV fluid with insulin B. cortisone with something C. flantadin (Addison dis)

🍒 Chronic hep B came with jaundice highly elevated liver enzyme it was about 500 600,
most appropriate next step
A. Alpha interferon. B. Biopsy. C. No antiviral drugs in the choices D- Request anti delta

17 years old male known IV drug abuser, came for a check up, asymptomatic. What will you
screen for? A. HIV B. Hepatitis B

🌹Nurse get infected with hepatits b , she is HBs ag And HBV rNA negative , what is your
next step ? A. no further investigation B. us. C. aminotransferase test
If HBs ag positive >C If HBs negative>A

- nurse get infected with hepatits b , she is HBs ag Incomplete And HBV rNA negative , what
you've next step ?
- no further investigation - us - aminotransferase test D- follow up closely with LFT’s

🌹jaundice patient with: + HBS antigen +anti hepatits b C IgM What the Dx? Chronic
infection

🌹known case of chronic Hep B presented with jaundice and fatigue Labs showed
worsening ALT + AST and protein what will would you do?
A.Initiate Alpha interferon ✅ ‫ ﻋﻼج‬B. Liver biopsy (No CT) .

🍓Pt had hepatitis B cirrhosis that is stable and they incidentally found a mass in the liver
asking what's the treatment ? A. anti viral B. liver transplant ✅ C. chemo

🍓autoimmune hepatitis lab= elevated lgG🍓

🌹 screening the mother for HBs what type of prevention? a. 1ry b.2ry c. 3ry d. primordial
🌹 2ry prevention of diabetes= a. healthy lifestyle (primary prevention) b. screening for
cases✅ c. immunization (primary prevention) (screen = scondry )


🌹Hep c and cirrhosis. How to tell this is progressing to hepatocelular carcinoma
Ultrasound 🌹
🌹While the obestitratian close the cesarian incision , Pt developed bleeding . What is the
cause ;- A.Liver heamangioma ✅🌹 B.Spleen anyresm C.Intestinal anyresm
🌹 wilson’s diseases, ttt?
A. Iv Defroxamin. B. penicillamine ✅ penicillamine chelates copper

ŒŽ ‫ •ﻤﺴﻚ‬copper ‫ و–ﻄﻠﻌﻪ ﺑﺮة اﻟﺠﺴﻢ‬.
Kayser-Fleischer Ring associated with? Wilson’s disease ‫ „ﻠﻤﺔ اﻟ† ﻟﻤﺮض‬4 Ž
Ser = Son
🌹 anaemia is seen in copper deficiency= Sideroblastic🌷

1Skin
🌹skin lesions in the back with some of them being crusted. +bilateral infiltrates = give
antibiotic 🌷

Patient known to have cardiac arrhythmia on amiodarone. Developed red, sharply


demarcated, red, tender lesion below right knee. Diagnosis?
A_Erythema nodosum
B- Amiodarone-induced ‫ﻌ~ﺪ ﻋﻨﻬﺎ‬u ‫ﺎﻟﺴ|ﻨﺎر–ﻮا‬u ‫ ﻓﺎﻟﻮﺻﻒ‬ýŽ • ‫ﺎﻟﻮﺟﻪ ﻟﻮﻧﻪ ﺑ‬u ‫ﻐﺔ‬y‫ﻫﺬا •ﻌﻤﻞ ﺻ‬
C- Erysipelas✅. D-DVT
🌹on amiodarone. Developed red, sharply demarcated, red, tender lesion below right
knee.=Erysipelas🌷
🌹Patient diabetic and in amiodarone, she developed skin lesion red demarcated,
tender, pt is febrile A. erysipelas B. amiodarone induced C. necrotic lipoma diabetic
🍓Child with lower eye lid swelling and no photophobia and no field affection and no
fever and not respond to Abx :• Cellulities. • Eye allergy • Orbital ischemia
🍓Child with eye swelling and photophobia, eye movement limitation and proptosis ?
• cellulitis ✅✅ • eye allergy
🍓Mass in the labia majora with black color? A- Basal cell cancer. B_Melanoma✅✅

🍓Rash in male age 40 want management antibiotic , steroid . Platelt supporive care ✅

🍓Most common location for Basal cell carcinoma : A. Face B. breast C. lower limb
🍓pt have rash after use herbal medication -allergic contact dermatitis?
Not contact since it’s medication
🍓 24 yr old female uses herbal cream on her vulva noticed redness itchiness and
tenderness around the area of application: - Allergic contact dermatitis.
🍓elderly pt with swelling of the back of neck with multiple opening and discharge and
redness , what’s your diagnosis: A. carbuncle✅ B. furuncle C. abscess D. cellulitis
🍓Female with progressive pain and redness on left hand with History of knife prick :
A. cellulitis✅. B. gas gangrene C. necritizing fasiaitis

🍓 Honey colored crusted lesion=impetigo


🌹Vesicles in all mouth ( in posterior and soft palate ) this is = herpengia caused by cocxaki
🍒Patient with significant hx of painless ulcer. Now presented with rash. =
A. trypanum pallidum

🌹Describe pigmentations on axilla and neak = Acanthosis nigr..🌹

🌹The most associated with nicotine/ outcome related = addiction 🌹



🌹Femal young with vaginal bleeding and brusis = sexul child abuse 🌹

🌹Lady screened for HBV negative , her husband positive on treatment :protcted sex with
condom 🌹

🌹Pt with psoriasis.. best regarding bathing? A. No soap ✅. B. coconut oil better than olive
oil
Psoriasis use ointments and moisture-rich soaps
[• ‫ﻌ~ﺪ ﻋﻦ ﻣﻮاد ﻛ~ﻤ~ﺎﺋ~ﺔ ﻻن راح ﺗ‬u ‫ ز–ﻮت ﻃﺒ~ﻌ~ﺔ‬¤‫•ﻔﻀﻞ اﺳﻨﻌﻤﺎل اﻧﻮاع ﻣﻌﻴﻨﺔ ﻣﻦ اﻟﺼﺎﺑﻮن ﺗﺤﺘﻮي ﻋ‬
🍒Definitive diagnosis psoriasis?biopsy🍒
🍒Primary diagnosis psoriasis= clinical🍒
🌹psoriasis management = steroid 🌹
🌹Cause of psoriasis (pathophysiology) = epidermal hyperproliferation
‫ﺎﻟﺠﻠﺪ ﻓﻤﻦ اﺳﻤﻬﺎ‬R ‫ة‬xu‫ﻜﻮن ﻣﻠ;ﺎن ﻗﺸﻮر ﻛﺜ‬K ‫ﺴﺲ‬Š‫ﺳﻮرا‬
🍒🍒Vesicles and ulcers appears on anterior mouth (gingivostomatitis) what is the
causative agent ? Herpes 🍒🍒
🍒 Vesicles appears on soft palate and posterior oral cavity? Herpangina
Vagiana psorises : pt present with well demarcated lesion suspected to be psoriasis , how to
🍒Dx. ? A-Biopsy ( if ask abot Definitive indicantion). B-Clinical feature is enough ✅
🍓calcium pyrophosphate crystals = pseudogout
🍓negatively birefringent = Gout
🍓Patient came with joint pain, needle like crystals, negative birefringent, (its acute
attack). What med NOT to give? Allopurinol
🍓 Medication contraindicated in acute crystal-induced arthritis
○ NSAIDs. ○Colchicin ○ Allopurinol ✅
The medicantion for acute gout use = NSAID OR Colchicine , but the Allopurinol its not usfal
in acute phase .
🍓 case about hyperuricemia and negative birefringent urate crystal, how to manage?
Probenecid ✅ ✔ is a medication that increases uric acid excretion in the urine. It is
primarily used in treating gout and hyperuricemia.
🍓 Diabetes patient with severe unilateral knee joint pain, on exam it's erythematous,
swollen, warm. patient has fever, joint aspiration shows I think 55.000 wbc? Culture was
negative , crystals still pending. Diagnosis? A- Gout B- Septic arthritis
🌹Typical case about gout (big toe inflamed and tender), what is the name of deposited
material? monosodium urate
🌹 calcium pyrophosphate crystals = pseudogout.
🌹 Patient presented with feature of gout attack first line ?
A-Indomethacin✅ its NSAID. B-Probenecid

1Bacteria
Brucellosis
🌹Treatment on brucellosis? doxycycline and rifampin for 6-8w.
🌹Brucellosis Treat? A.Doxycycline and clindamycin B.Doxy and streptomycin(first line)
.🌹Brucellosis in joints only. Doxycycline treatment for how long? A. 6 weeks B. 6 moth
🌹Neurobrucellosis on rifampin and (another AB I think doxycycline).., ask about duration
of Tx: 6 month
🌹Nurobrucellousis treatment durantion = 6month to 12 month
🌹If it is without neuro manifestation it’s = 6-8 week
🌹Man presented with fever, headache, and back pain, with right sacro-illiac joint pain,
what is the infecting organism: A. Syphilis B. Brucellosis ✅
🌹Case of diarrhea or something came from India, Most common causative org?
A. Rota B. E. Coli. C. Cols. Perfrigins
Pt with hx of rheumatic fever came to hospital after 19 days with fever?
A. Recurrent Rheumatic fever. B. Bacterial endocarditis. C. Mitral valve flotter

🌹 Mollascum contagisum with picture and symptoms


Treatment = A. Antibiotics. B. Antivirals C- Suppurative !@

🌹An elderly man surgically free with fever and new systolic murmur and signs of
infective endocarditis. What is the organism? staph aureus

🌹Infective endocarditis post prosthetic valve one month? staph. Epidermidis


🌹Native = staph aureus. IV abusers = staph aureus
🌹Prosthetic:
Early (within60 days)= staph.epidermidis. Late= Strep.viridans. Dental or oral = viridans

🌹Most common cause of Gastroenteritis after eating steak from local restaurant?
E Coli.✅🌹
● 1-6 hours —> (staph. aureus)
● 12-48 hours —> (salmonella, e.coli)
● 48-72 hours —> (shigella) campylobacter
● > 7 days —> (giardiasis, amoebiasis)

🌹patient with macules in the cheek and oral , O/E : lymphadenopathy Dx ?


A-EBV ✅. B-Herpes zoster. C-HSV. D-HPV
🌹Best Advices to traveled man
1- just eat in clean resurant✅😀👍 2- just drink goverment water 3- aviod rice !@
🌹Young patient after travelling, presented to ER with non-bloody watery diarrhea, no
other details were given, most likely abx you will give:
A. Metronidazole ✅✅‫ ﻣﻜﺮر‬B. Ciprofloxacin C. Other Beta lactams abx
Same Q what’s the Dx : Giardia
🌷after travelling = non-bloody watery diarrhea = Metronidazole = diagnosis ?= Giardia🌷

🌹Patient was of clindamycin and another antibiotic came with watery diarrhea
asking about management = Metronedazole ...
🌹case in women white discharge +psudophyte in microscopic? Cndida
. white ‫(= ﺑﺷرﺗﮭم ﺑﯾﺿﺎء‬candida) ‫ اﻟﻧﺎس اﻟﻲ ﻣن ﻛﻧدا‬: ‫رﺑط‬
🌹Trichomonas, and ask about ttt? -Metronidazole✅ ‫وا‬ÒÓ‫ﺗﺮا™ﻤﺎت =ﺗﺮﻛﺾ =وراء اﻟﻤ‬
🌹prosthetics valve repairs after 4 c'me with picture of IE what bacteria '?
A-st'ph 'ureus B-staph epidermis ✅✅. C-strep vird'n
P.......‫ ﻣﻊ‬P ‫ ﻧﺣﻔظﮭﺎ ﺣرف‬، e*p*idermis ‫ ﻏﺎﻟﺑﺎ ﯾﺟﻲ ﻣﻌﮫ ﺑﻛﺗﯾرﯾﺎ‬p*rosthetics valve* ‫اﻟﺻﻣﺎم اﻟﺻﻧﺎﻋﻲ‬
🌹Patient 1 month after valve replacement, what organism?
A-Step varidans. B-S.Epidermatic✅ C-Staph auras

🌹Generalized cervical lymphadenopathy +‫و‬mild tenderness + low grade fever. What's the
most likely diagnosis: - Viral Infection .✅‫ﺻﺦ‬

Travelers diarrhea came back with watery diarrhea + trophozoites?


A- Ameba B- Gardia ✅
‫ = ﺟﺮد‬gardia= ‫ ﻓ~ﻪ ﺗﺮاب ووﺳﺎﺧﺔ =ﺗﺮاب‬Òç‫ اﻟﺴﻔﺮ وﻏ‬ŒŽ • ‫= ﺟﺮدل اﺳﻬﺎل وﺳﺦ‬:trophoz ‫ﺗﺮاب‬
.......
🌹Child had fever, vomiting and rash on 2nd day became all over the body?
A. Meningococcemia B. Rocky Mountain fever =¦ ƒ § ‫ﺎﻟ‬wv‫ رﻛﻮا ﺑ‬²n‫ =دﻓ‬8‫رو‬.
ƒ § ‫¦ =ﻳﻮﻣ‬
o
C. Kawasaki D. Measles
🌹E.coli uti allergy to penicillin sulfa and shellfish what you give?
A. tmx/sul B. Ampicillin C. ceftriaxone D. Nitroforgen

🌹15 year old girl presents with lower abdominal pain, diarrhea and fever (38°C). She has
mucus and blood mixed with her stool. The most likely organism is?
A. amebiasis B. Taenia saginata C. Ascaris D. One more worm

🌹 “staccato cough”, organism :Chlamydia pneumonia*


ýŽ Ó ‫اﺳﻜ‬. Òç‫ﻮت =ﺳﻜﻮت =„ﻼم ﻛﺜﻴ‬Ù‫ﻂ =ﺳﺘﺎ‬Ï‫ر‬.
Chalamydia = ‫„ﻼم‬
.....
🌹Pt had an RTA later developed s & s of gangrene, what is the organism: C.bifergins

🌹 19 years married came wit symptoms of Lower UTI Dx:= Honeymoon cystitis
Staccato cough >> chlamydia pneumonia
staccoat🌹 ‫ اﺳﻛﺗوا 🤐🤫 ﺳﺗﺎﻛوت‬chlam pnum= ‫رﺑط = ﻛﻼم وﻧﻣﯾﻣﺔ‬

🌹 case of febrile neutropeni' wh't next?cluture form sputum,urine,blood 'nd Iv


antibiotics = Febrile neutropenia can develop in any form of neutropenia, but is most
generally recognized as a complication of chemotherapy when it is myelosuppressive
🌹 Elderly case of endocarditis with fingernail splinter hemorrhages, what’s the causative
organism: A. Strep species✅. B. Enterococcus faecalis C. Klebsiella
🌹 patient with splinter hemorrhage and other sign on Endocarditis, no previous
operations, what is the most common organism: A- staph. Aureus B- strep. Veridans ✅
🍓pt just came from South Asia with diarrhoea what is the infection he had:
Answer is :E.Coli
Bloody diarrhoea> Shagella
‫ﻪ و–ﺠ~ﺐ دم‬Ï[• ‫ﺷ~ﻘﺎﻻ =ﺷﻘﺎوة =ﻧ‬
Watery diarrhoea> E.Coli
¸Ž á ‫ ا™ﻞ =„ﻠﻨﺎ ﻧﺎ™ﻞ ﻋﺎدي =واﻻﺳﻬﺎل ﻣﺎ‬ÐŽ‫ﻛﻮﻻي =ﻛﻮ‬
🌹 Impetigo caustive organism, (honey crusted lesions )? Staph aureus 🌹
🌹Pt presented with urosepsis, urine culture revealed E.coli >100000. TTT? Ceftriaxone (
colli > third generation cephalosporin. ) 🌹
.. . . . . . .‫ﻧﺎﺧذ ﺳﯾف وﻧﻘطﻊ ﻓﯾﮫ اﻻﻛل‬cef = 🔪 ‫ ﺳﯾف‬Coli =.🥖🥐🍞 ‫رﺑط = ﻛوﻟﻲ‬
🍓Case of patient came from africa with large LN high fever :
A. Corona B. Ebola C. EBV✅ N.B Ebola can cause it but, not very common

🌹Elderly with catheter with positive culture: Check urine catheter daily 🌹
🌹Elderly pt with lower abd pain and couldnt urinate for many what management?
foley catheter, if acute urinary retention 🌹
🍓 Fever, pharyngitis , maculopapular rash in trunk , lemphadenothathy , dx?
A. Infectious mononucleosis ✅ B. CMV C. Hodgkinlymphoma D. dighteria

🌹 Young female presented with fever, pharyngitis, generalized lymphadenopathy, DDx?


A. diphtheria B. Hodgkin C. mononucleosis D. CMV

🍓 Watery diarrhea , came from egypt, dx? E.COLI = Egypt

🍒Adult with positive kernig sign what is the causative agent :Sterpt🍒
….
🍒🍒scenario about impetigo, asking about organising? Staph aureus 🍒🍒

🍓pt came from Egypt with leg tender red lesion in rt leg and she had penicillin allergy,
you give her Ab , in second day she developed N/V and diarrhea watery=Wt the Dx
A. salmonella B. Ab allergy C. worsen gastroparesis D. C diffeclis✅
🌹 50 yo Female came back from Egypt, known case of penicillin allergy ,presented to the
ER because of painful red lesions on the anterior shin of the right leg, she received
antibiotics, next day developed abd pain, fever and diarrhea. What’s the cause of her
symptoms? A. antibiotics allergy. B. Clostridium difficile✅. C. Salmonellosis
💐💐Patient with C difficle, what's the cause = omeprazole 💐💐
🍒Patient on clindamycin developed diarrhea, stool analysis: c.diff Toxin, management?
A.Continue same medication B.Azithromycin C.Metronidazole✅
No vanco on choice

🌹Patient had positive cryptococcus “multiple unproductive sex”= HIV serology🌹



Patient eat Steak 🥩before 4 hours and now presenting with vomiting which
organsm can cause it : B.E.coli✅ ‫ﻘﺮ‬x‫ﻣﻊ ﻟﺤﻢ اﻟ‬
🍒History of URTI 2 weeks ago and now presented with rash? Pneumococcal🍒
🍒🍒lobar pneumonia in adult organism? Strep.pneumoniae🍒🍒
..
🍓Case of seizure followed by diarrhea? A. Shigella✅ B. H. Pylroic ‫ﺷ~ﺠﺎﻻ =ﺷﻘﺎوة‬
🍓Most common cause of OM in pedia : A. Viral B. Bacterial✅ C. Fungal
🍓2 weeks old with conjunctivitis bilaterally , 2 weeks later chest x ray show lung interstitial
lung infiltration: A. trancumnus B. Streptococcus Answer is viral or chlamydia✅

1Drugs
🌹 Furosemide SE: = hypokalemia
🌷 Pt with signs of Hypocalcemia , chovestic sign : A. oral Ca B. IV Ca
🌷 acetaminophen ingestion they mother says that the bottle had a small
amount and he spilled some of it on his clothes after 4 hours
observation in ER he is still stable A. Watch for another 4 hours
🌹 Chronic pain medication?A- Acetaminophen b-Opioid c-Ibuprofen
‫ ﻧرﻛز ﻗﺎل‬chronic ‫ وﻟﯾس‬acut ‫ﺑﺎﻗﻲ اﻻدوﯾﺔ ﻣﺎﯾﻧﻔﻊ اﺳﺗﻌﻣﻠﮭﺎ ل ﻓﺗرة طوﯾﻠﺔ ﻟﮭﺎ اﺿرار ﻣﻧﮭﺎ ﻋﻠﻰ اﻟﻛﻠﻰ واﻟﻣﻌدة‬.
🌹 1)_Cyclosporine use for? Steroid sparing agent
‫ﻂ ﺣﺮف‬Ï‫ ر‬S ‫ ﻣﻊ‬steroid ‫ و‬sparing
....... ..... ....
🌹 High K = give ca gluconate K = Ca
🌹On chemo then develop fever = culture +iv ABx
🌹sinus infection with haemophilus influenza treatment ?
A. steroid inhalation abx decongestant B. steroid anti inflammatory
🌹Most effective method to decrease stone formation in patient with hypercalcemia?
A. decrease protein B. thiazide C. allopurinol D. decrease calcium intake

🌹management high ca = normal saline


‫ﻂ =اﻟﻤﺎء •ﻐﺴﻞ اﻟﺪم و–ﻄﻠﻊ اي ﺗﺮا™ﻤﺎت ﻓ~ﻪ‬Ï‫ر‬
🌹Pt known to have advanced stage cancer "I think lung cancer" came complaining of back
pain, his oncologist told him he would not live more than a year, labs found his calcium
high "~3 folds", mx? A. IV fluid B. Furosemide C. Morphine

🌹A lady who lost her close friend in grief and complaining from trouble sleeping Which is
the best treatment ?
A. Imipramine. B. Chlorpromazine. C. Lorazepam✅ ( trouble sleeping) D. Fluoxetine
‫ﻂ =ﻟﻮرا =ﻧﻮم =اﻗﻔﻞ اﻟﻨﻮر =ﻋﺸﺎن اﻧﺎم ز–ﺎدة‬Ï‫ ر‬.
........
🌹What medications should not be taken with Plavix(clopidogrel)?
A. omeprazole✅. B. statin. C. SSRI
🍒 pt with MI ( give imfomtatlon and vitals signs with increase JVP ) , what is the tx of shock
-A-Abx B-doptanim ✅
🍒 pinpoint pupil treatment = Nalxon

🍒 nitanolo toxicity : glucagon


🌹 absence seizure treatment = Ethosuximide
🍒MRI was given diagnosed with toxoplasmosis, What is the ttt?
Pyrimethamine + sulfa
🍓Leukotriene receptor antagonists: montelukast🍓
🍓pt have sepsis, what are the things you will not need it :
A. Iv antibiotic. B. inotropes. C. iv fluids. D. corticosteroids✅
🍓 Case of that girl who ingested 2 tablet of paracetamol after one day she go to hospital
with N/V and RUQ pain what to give :
A. acetylcysteine✅ B. activated charcoal C. don't give any thing in this time
*activated charcoal* = at *1-3* hours post-ingestion, after that no need.
Serum paracetamol level peaks *4 hours after ingestion* If he came 4 hours or more after
ingestion order serum paracetamol level immediately.
If he came 7 hours after ingestion with symptoms or history suggests toxic dose start
antidote immediately !!!!
because hepatic injury usually starts 8 hours post-ingestion otherwise patient may lose his
liver while you are waiting for the results.
........
🍓 garlic smell : organic phosphorus🍓
‫ﺟﺎرﻟ}ﻚ =ﺟﺮدل =ﺟﺮدل ﻓ}ﻪ ﻣﻨﻈﻔﺎت ﻋﺸﺎن ﻧﻨﻈﻒ ﻓ}ﻪ‬

🍓Low grade fever ,o2 sat 88 , hypotension 90/50 what is the initial step of management?
A. Iv fluid B. O2✅ C. Antipyretic .......ABC management...

🍒SLE on prednisone with neurological manifestation what add ?


Cyclophosphamide ( for nurologic manifistantion )🍒

🍒DM on metformin 1g not control? combined with oral hypoglycemic agents🍒
🌹Female BMI was 40 what give her for DM ? A- metformin B- exercise C- Diet

🍓After Chemotherapy for cancer , reading of electrolyte disturbance=Hypocalcemia



🍒 candidate for asymptomatic UTI screen?
Pregnant ‫ﺎﻋﺮاض اﻻﻟﺘﻬﺎب ﻓﻼزم ﻧﻌﻤﻞ‬u ‫ ﺣﺎﺳﺔ‬4
Ž ‫ ﻣﺎ‬4
Ž ‫ﻻن ﻣﺎﺑ~ﻌﻤﻞ اﻋﺮاض وﻛﻀﺎﻋﻔﺎﺗﻪ اﻧﻪ ﻣﻤﻜﻦ •ﻌﻤﻞ اﺟﻬﺎض و‬
‫–ﻦ‬À‫🍒 ﻟﻬﺎ ﺳﻜ‬
.....
🍒A middle age company worker with mild depression, and he has interpersonal conflicts
with colleagues? Psychotherapy CBT🍒

🍒 symptoms next appropriate step? gluten free diet🍒
….
🌹Young patient T1DM on rapid and long acting insulin with hypoglycemia, what’s the next
appropriate management? decrease level of rapid and long🌹
..
🍓 management of open wound with fracture? irrigation and debridement is the first step
in any open wound 🍓
🍓hyperkalemia management = ca gluconate 🍓
🍒 Pt diabetic + CKD > k+ high > first line of management ? ca gluconate
Case of heroin withdrawal which was treats, now as a rehabilitation of heroin addiction
doctor should use a heroin. substitute which is:
A. NaloxoneB. Papaveretum C .methadone✅
🌹 A man with multiple sexual partners (Hx of unprotected sexual encounters) has
decreased LOC and other CNS symptoms. Toxoplasmosis is confirmed (they
gave the results) and asking what is the best treatment =
(Trimethoprim/sulfamethoxazole)
🍓Vomiting in every flight more than I hr, tx: scopolamine
46 years old female was on OCPs in last 12 years presented with upper abdominal pain On
the investigation The was hepatic adenoma What is the appropriate next step?
A. Surgical resection B. Stop OCPs ✅✅
🍒Women took OCP for 12 years came with RUQ pain imaging revealed mass 5 cm on the
right hepatic lobe how you will manage?
A.Stop the OCP B/ Excistion ✅ ‫ﺣﻞ ﺟﻠﻮري‬
In general Stop OCP's follow up in 6-12m But since she is symptomatic and more tnan 5 cm
= I will go with Resection = surgical resection is recomme
...........

🌹man wants to stop smoking after his father getting a lung cancer, he is a known case of
seizure disorder compliant on carbamazepine, which one from the following he can't use?
A. Electronic cigarettes B.bupropion ✅ ‫ﺸﻨﺠﺎت‬â ‫ ﻳﺰود‬C. NRT D. varenicline
# wants to stop smoking + has Epilepsy = Dont give bupropion 🚫
🌹Pregnant lady known asthmatic was on salbutamol only before pregnancy and was
controlled, During pregnancy, she started experiencing daily symptoms and once a week
night symptoms, Which of the following treatments would you give?
A. ICS with short acting bronchodilators C. ICS with long acting bronchodilators
D. ICS with long acting beta 2 agonist✅
🌹Poorly controlled asthmatic patient developed hypertension which drug is CI:
A. beta blocker B. CCB ‫ﻠﻮك ﺑ~ﻌﻤﻞ‬u ‫ ﺑ|ﺘﺎ‬bronchoconstruction
Over dose of beta blocker asking about antidote? Glucagon
🍓patient presented with decreased reflexes, numbness and vitiligo (B12 def) what to
give: A. oral B12. B. Im B12✅
🍒Thiazide helps in decreasing urinary calcium > calcium stones in kidney
🍒Allopurinol > decrease risk of uric acid stones in kidney
.........
🌹pt with penicillin allergy what abx to give him : clarithromycin or ciprofloxacin

.........
🌹Asymptotic UTI after delivery : No need🌹‫ ﺧﻼص وﻟﺪت ﻣﺎ &ﺤﺘﺎج‬،

🌹Leukotriene Receptor Antagonists? Mountelukast .


..........
🌹causative organism of viral conjunctivitis Adenovirus 🌹
..............
Case of car accident which of the following keep it low: ‫ﻣﻮ ﻣﻔﻬﻮم وﻫﺬا ﺣﻞ ﺟﻠﻮري‬
A-insulin ✅ B-hypoglycemia C-prolactin

🍒🍒Pt happen to her chemical burn during resuscitations they remove her close and
found chemicals powder what next step? swiped the powder. 🍒🍒
🌹Female patient has chemical burn all back area what to do in ER ?
A.sweep chemical powder✅ B.wash by clean water for 30 min (if lqiuid)

🍒active status epilepticus= benzodiazepine🍒


🍒DKA. His Glucose is 34 mml.= Decrease glucose by 3 mml per hour🍒 If very high
glucose should be reduced at least 10% in 1st hr.

‫ = ر`ﻂ‬stat = ‫ 😂 =ﺧﻼص ﻋ}ﺐ‬²§‫ﻼم اﻟ–ﺜ‬e‫وا =ﻓ}ﻪ زن و ازﻋﺎج {ﺎﻟ‬²ƒ ƒ ‫ﻢ =ﺑ‬wv‫ ﺳﺘﺎت ﺣ‬Epi

🌹Patient couldn’t tolerate ACE due to cough? switch to ARBs🌹


..
Treatments of facial palsy urgent ttt = 🍒steroids ‫ي وﺟﻬﻚ‬ÒÓ‫ =اﺳ‬ÒÓ‫و–ﺪ =ﺳ‬Òç‫ﻂ =ﺳﺘ‬Ï‫ ر‬.


🍒Amiodaron Side effect = bradycardia, photoxicity, thyroid toxicity, lung fibrosis 🍒
..
💐Allergy after CT contrast=Epinephrine💐

Paracetamol toxicity presented after 1 day with symptoms of RUQ pain and nausea next:
= N-Acytocloin

🌹case of young girl medically free symptoms of migraine what give her?
NSAID 🌹
...
💐External ear infection ttt?? Topical neomycin (Not orall)💐
🍒Young female after swimming pool, painful red ear pinna?Topical neomycin
🌹Patient with inflammation of external ear with hx of swimming management:
A. oral amoxicillin B. topical neomycin
...

💐 nephrotic syndrome just dx in clinic Ask for ttt ? start steroids💐



🌹Heart failler with crepitantion = ACE Inhiptor + diarotic 🌹
..
🌹low HDL + high LDL + high triglyc + high total cholostrol = Atorvastatin 🌹
...
🌹druge decrease the affect of Clopidogrel = omeprazol 🌹
🌹What is the medicantion (DDT4 hypoglycimia druge ) not need for evaltion kiedny
function when we use it == Lingliptine 🌹
🌹pt with heart disease with statin + atenalol + metaformin came now ( orthostatic
hypotenstion ) what the druge from all cuases it ?? Beta block (atenalol )
🌹pt with acute attack of gout what Medicantion (NOT) Give to hime ??
Allopurinol 🌹
......
🌹edema in lips and face whats druge cuses it ? ACEI
. . .. .
🌹DM obese what's best advice = exercise with low carbohydrate intake 🌹
....
🌹Alcoholic with pancriatic clacifide and fat on stool = first tell.him to decreas fat intake (
If failled use lipase with each meal) 🌹
.....
🌹Streptokinase antidote: aminocaproic acid antidote 🌹

🌷pt with massive hemorrhage what to give to stop the action of streptokinase=
A-aminocaproic acid B - vitamin K
.‫ اﻣﯾن = رﺣﻣﺔ ﷲ ﻓﯾﮫ ﺷﻲء ﯾوﻗف ھذا اﻟدواء‬amino = ‫رﺑط = اﻣﯾﻧوا‬....

...... .....

🌹Dry mouth after chemo? Hypocalcemia🌹



🌹Patient is having sensitivity from penicillin what is the absolutely contraindicated abx?
.Pipracilline/tazo

🌹 Hypertension antihypertensive + low K , Which of the following will lead to treatment
failure ? Pseudonephedrine


1Antibiotic
🌷-candida manage? Trecoazole cre'm topic'lly
🌹 SLE flair with UTI what drug not to give to this patient:
A. Cephalexin. B. Amoxicillin C. Nitrofurantoin D. Sulfamethoxazole
🌷Antibiotics contraindications in CKD? Nitrofurantoin & Fosfomycin🌷!@
🍒 patient k/c renal impairment cannot give him? . nitrofurantoin
🍒 Pt with CKD( chronic kidney dis) and UTi ttt?
A)_Nitrofr ‫ﻠﻮي‬6‫ ﻣﺮض اﻟﻔﺸﻞ اﻟ‬¤‫ﻫﺬا ﻣﻤﻨﻮوووع ﻋ‬
B)_ Cipro ✅ ✔ ‫ ف اي اﺣﺪ ﻋﻨﺪە ﻓﺸﻞ „ﻠﻮي و–ﺤﺘﺎج ﻣﻀﺎد ﺣﻴﻮي‬، ‫ﺪ‬yÄ‫–ﻖ اﻟ‬À‫ﻫﺬا اﻣﻦ ﻻن ´ﺴﺘﻘﻠﺐ ﻋﻦ ﻃ‬
‫وﻟ~ﻜﺲ‬Ò£‫ﻧﻌﻄ~ﻪ دواء ﺳﻴ‬
🌷 Which drug can safy taken in ckd ( Chronic kidney disease ) stage 3 :
- Metformin - nitrofurantoin - Lithum - Wafrarin
🌷 when to stop metfromin in di'betes if GFR is? Less than 30 (stage 4)
🍓Man with flank pain , dysuria - UTi what you’ll give ? Nitro
🍓patient known to have an allergy to penicillin and sulfa group present with
UTI which antibiotic will choose ?
A. Nitro ✅ UTI = NIT. B. trimethoprim/sulfamethoxazole i C. cephalexin D. ampicillin
🌹Otitis Media signs and symptoms asking about the treatment: Amoxicillin/Clavulanic
🌹Patient presented with diarrhea for two weeks. Stool showed trophozoites (giardiasis)
What is the treatment:= Metronidazole
‫وا‬ÒÓ‫ اﻟﺤﻖ ارﻛﺐ اﻟﻤ‬ýÓ‫–ﺖ ﺧ‬À‫ﻂ =ﺟ~ﺎردا =ﺟ‬Ï‫ =ر‬metro.
.......
🍓Which antibiotics can cause prolonged QT interval: = Erythromycin
Infectivev endocarditis with valve involves what Ab to give:. Vancomycin + ceftriaxone✅✅
🌹 pseudomembranous colitis management = vancomycin 🌹
‫ﻂ =ﻣ¼†وب ﻓﻤﺘﻮا‬Ï‫ = ر‬vanco = ‫ﺎت ﻫﺬە‬Ï‫ اﻟﻤ¼†و‬¤‫ﺬب واﻻﺷﺎﻋﺎت ﻋ‬Ä‫ اﻟ‬Òô‫ﺎﻟﻘﻮﻟﻮن وﺗﻬﻴﺞ =ﻳﻮە •ﺎ ﻛ‬u ‫•ﻌﻤﻞ اﻟﺘﻬﺎب‬
psudo .
🌹 Treatment of clostridium difficile: IV Vancomycin ✅
‫ = ﻣﻊ اﻧﮫ ﺻﻌب ﻧوﻗﻔﮫ ﻻن طﻌﻣﮫ‬close = ‫رﺑط = ))ﻓﺎﻧﻛو = 🥤🥤 ﻓﯾﻣﺗوا = ھذا اﻟﻣﺷروب ﻣﺿر ﺟدا ﻻزم ﻧوﻗﻔﮫ‬
))‫ =ﺻﻌب‬difficilt =difficult ‫ﺣﻠو‬
🌹old man tender abd bloody stool take amoxicillin from 3 wks dt UTI sigmoidoscopy
inflamed mucose with plaque like lesions?
A. ischemic colitis B. pseudomembranous colitis✅

🍓Infective endocarditis treatment = Vancomycin and ceftriaxone as empirical tx


🌹Case of infective endocarditis asking about empirical antibiotics =
vancomycin + gentamicin ‫ﻋﻠﻰ ﺣﺳب اﻻﺧﺗﯾﺎرات ﻟﻛن ﻛﻣﺎن ﺟﯾﻧﺗﺎ ﯾﻧﻔﻊ ﻣﻌﮭم‬
🌻Empiric abx for Native valve endocarditis: penicillin G + gentamicin
🌻Patients with a history of IV drugs: nafcillin + gentamicin

🌹meningitis Strpt .P = Ceftriaxone + Vancomycin 🌹


🌹Infective endocarditis scenario splinter hemorrhage rx ? vanco + ceftriaxone ✅
Patients with history of prosthetic valves : Vancomycin + gentamicin + rifampin
🌹Case about treatment of Staph aureus resistant: Vancomycin
🌹 Case Septic arthritis, start on a cephalosporin and culture showed staph and resistant
to that drug, best Mx? A. Arthroscopic washout B. Start vancomycin
🌹 treatment of Staph aureus resistant: Vancomycin
after appendectomy a patient got abdominal infection by enterococcus faecium. He is
allergic to penicillin what are you going to give him: ==vancomycin✅
🍒pt with MRSA wanted to be treated by Vancomycin. Immediately pt developed rash in
the trunk. next step =continue with slow infusion = (Red man syndrome is an infusion-
related reaction to vancomycin )🍒

🍒Pt 60 year old, wants prophylaxis for meningitis?ciprofloxacin


‫ﺎر اﻟﺴﻦ ﻻن ﻋﻘﻠﻬﻢ ﺣﻜ~ﻢ‬y‫ ﻛ‬¤‫وا ﻋ‬Ò£‫وا =اﺻ‬Ò£‫ﻂ =ﺳﻴ‬Ï‫ر‬

🌹40s man with CAP Empirical Abx?


A. Cefatizidme B. Meponim C. Tazobactam/piprcillin D.Cipro

🌹pt come from indonisia present with bloody diarrhia diagnosis with ( camylobactiria ) =
Azithromycin 🌹
🌹 Campylobacter jejuni treatment ?
A. Ceftriaxone. B. Azithromycin ✅ C. Metronidazole D. Fluconazole
‫ﻂ =„ﺎﻣ~ﻠﻮا =ﺟﻤﻴ~ﻠﺔ =ازﻫﺎر‬Ï‫🌻🌼 ر‬azethro

🌹Prophylax on.meningitis in adult = ciprofloxin🌹


....
🌹Prophylax on.meningitis in child = Rifampacin 🌹
. . ..

….
🌹Spontaneous bacterial peritonitis treatment? IV ceftriaxone

🍒baby on otoscope exam there’s no mobility of TM, he is breast feeding,no fever, no
pacifier, it’s unilateral what will you give? A. amoxicillin ✅✅. B. amoxicillin-clav
..
......
🍒🍒H pylori case on ppi and ....azole what you add ? Clarythromycin 🍒🍒
..
Empiric Abx for infective Endocarditis = Vanco and Gentamicin


🌹sever allergey to vancomycin = stop vancomycin then give cefatriaxon 🌹
🌹just rash happen due to vencomycin = continue with vancomycin but with slow infustion
rate ( Bez its mild allergy not need to stope vanco cont with slow infustion 🌹

‫اﺳﺎﻟﺔ ﻓﯿﮭﺎ ارﻗﺎم وﺣﺴﺎﺑﺎت‬


🌹 Q about minimum muscle strength per week = 2
🌹-Long scenario of diabetic patient and some renal problems when to stop metformin
regarding the GFR?
A-60 to 45. B-45 to 30. C-less than 30 ✅✅ = GFR = 3 ‫ ﺣﺮوف‬D-less than 15
🌹 when u stop metformin? -less than GFR 30 ✅

🌹The same scenario asking about stage or phase of paracetamol toxicity, she was
complainig of abd pain,nausea and vomiting = 1✅✅ 2. 3 4

🌹Mean Arterial Pressure is 55 , intracranial pressure 15. What's the cerebral perfusion
pressure? 40 (55_15)
🌹Dyslipidemia screening age MAN: A. 18 B. 25 C. 35 D. 50
🌹Dyslipidemia screening age femal:45
🍓 How many quality years does smoking take from a somker?10
🌹 You have a population, 800 have liver disease and 800 dont have it. The test shows 200
of those with the disease to be positive. What is the sensitivity? A. 25%✅B. 33%C. 50%
🌹 What is the percentage of male smokers in Saudi arabia: 25%

🌹Smoking risk of MI increases by= 2. MI = 2 ‫ﺣرﻓﯾن‬


🌹Start screening of HPV:. 20-24 years 🌹
🌹 Screening of HVB every: 3 years ‫ ﺛﻼث ﺣﺮوف ﻣﻊ ﺛﻼث ﺳﻨﻮات‬.
🌹Screening for HTN in low risk people ? A. 19 y B. 25 y C. 35y D. 50y
🌹minimum daily exercise for children:60min
🌹 How to calculate cerebral perfusion pressure MAP-ICP = CPP
🍓How many hours of sleep are associated with weight gain: < 6 hours✅
🍓child that is dehydrated and his glucose was also low the question was what would you
give him ? A. 20ml/kg NS B. 10% dextrose in 1/2 saline✅ C. Abx
🍓Child 10 kg had 5% dehydration calculate the fluid requirements ml/ day:
A. 1000 B. 1250 C. 1500✅. D. 1750
How to calculate maintenance :
100‫ ﻛﯾﻠو اﺿرب ب‬10 ‫اول‬
50‫ اﺿرﺑﮭم ب‬10 ‫ﺛﺎﻧﻲ‬
20‫ اﺿرﺑﮭم ب‬10 ‫ﺛﺎﻟث‬
🌹 Young came with dehydration (hypotension + tachycardia)what give first :
- IV bolus NS
......
🌹When to screen for hyperlipidemia in low risk males? 40🌹
🌹Which cancer to be screened in general population: : Colorectal cancer 🌹
🌹Osteoporosis screening: 65 🌹
🍒🍒How many years does the average smoker loses = 10years 🍒
💐Lung cancer risk increases with smokers for how many folds? 20 💐
🌹Risk of lung cancer in smokers =20🌹
🍓How the risk of smokers when to campare of non smokers :
A. 2 ✅ B. 3 C. 4 D. 5 heart disease:by 2-4 folds.
🍒 Osteoporosis and colon ca screening : osteoprosis: 65 , Colon Ca: 50 🍒
🍒 screen of colon cancer in low risk group? 50y
. ‫ ﺳﻧﺔ‬50 = ‫ ﺑﺎﻟﻌرﺑﻲ‬٥‫ ﻣﺛل ال‬O ‫ ﺣرف‬colon ‫رﺑط‬

🍒Screening age for osteoporosis ?64-70🍒


🍒 Minimum time of effective aerobic exercise? 20 min 🍒
🍒N.B: no 30 min in options, It’s 150min/week, 30mins in 5 days/w so as daily it will be
20min per day.🍒
🌹Minimal workout activity for adult A. 20 per 6 days B.30 per 5 days ✅✅
🌹minimal exercise min in small session: A. 5. B. 10 C. 15. D. 20 ✅
🌹If they ask about what increase walking capacity answer is supervised exercise
program
🌹Osmotic gap of stool vibrio cholera :
A. Osmotic gap 40 B. Osmotic gap 10 C. Stool osmolality 400 !@
‫ ﺧﻠﯾﻧﺎ ﻧﻐﻠﻲ ﺳﻌر اﻟوﺟﺑﺔ‬، 🍜🥗🥘 ‫ ﷼‬٤٠ ‫ = ﻛوﻟﯾرا = ﻛوﻟﻲ = اﻛل = اﻏﻠب اﻟوﺟﺑﺎت ﺳﺎرت ب‬cholera= ‫رﺑط‬
‫ﻋﺷﺎن ﺧﺎطر اﻟرﺑط‬
.....
🍒Indicate remission of nephrotic syndrome?Urine albumin less than 4mg/m2/h for 3
consecutive early morning sample🍒
....
🍒 When stop metformin in CKD patient? GFR < 30🍒

🍒 Fluid correction in DKA to avoid complications?
give slow rehydration over about 48 hours, avoid hypotonicity and ofunnecessary alkali
therapy.🍒
.......
🍒Risk for hep C after needlestick? 3% (1.8-3%)🍒

🌹How many days per week is recommended to do muscle exercise?
🏝about 150 minutes ( 5 days ) of moderate exercise
🏝 75 minutes (2 days ) of minimum exercise per week.🌹
ƒ § ‫ ﻣﺮﺗ‬ÞÕ‫ﺎﺿﺔ واﻟﻤ‬wv‫ﻢ {ﺎﻟ‬e‫“ﻼ ﺷﺪوا ﺣ}ﻠ‬
‫ اﻻﻗﻞ‬-‫ع ﻋ‬ö‫¦ {ﺎﻻﺳﺒ‬ o
.....
🌹pt with paracetamol overdose now she has Right upper Quadrate pain + nuseia +
vomting = whats stage = 2 🌹
‫ﻠﻤﺔ‬á ‫ﺸﻮﻓﻮا‬ò ‫ﺲ‬ñ ‫¦ ﻻن‬ ƒ § ‫ ﻟ}ﻪ ﻗﻠﻨﺎ اﺛﻨ‬upper right pain ‫ء‬mn‫ ﻧﻔﺲ اﻻﻋﺮاض ﺗﻌﺐ و‬١ ‫ ﻻن‬2 ‫ ا‘ ﺣﺪدت ﺳﺘﺎج‬:
o o o
‫ﺲ {ﺪووووون اﻟﻢ‬ñ ‫وﻏﺜ}ﺎن‬
🌹Hepatitis B acute marker? anti HB core IgM (Acute marker)
..
🍒What is the good character of screening test == Sensitive🍒(Not specific)
🍓highest level of evidence to determine exercise effect on quality of life:
A. cohort B. clinical trial C. case control D. observational study ✅

..... ........
. . .. . . . .
🌹the goal of diapetic pt with coronry artery dis = LDL Less than 70 🌹
….
🌹pt with Bronze skin and hepatomegaly = 24 h urinary Iron (hemochromatosis)
🌹Case of hemochromatosis normal LFTs but high ferritin at about 860, how to manage:
A. Start Phlebotomy✅ B. follow up ferritin after 6 months C. deferoxamine
🌹Hemochromatosis asymptotic, liver enzymes within normal only increased in ferritin
level 750 Whats appropriate next step?
A. regular phlebotomy every month✅✅ B. follow up after 6 months .
🌹Hemochromatosis=phlebotomy: weekly or q2wk then lifelong maintenance
phlebotomies q2-6mo🌹

🌹When to use metformin in renally impaired patient: GFR Above than 30 🌹



🌹POST MI when we do ELECTIVE surgery: 6 weeks = ‫ﻂ =ﻋﺪد ﺣﺮوف‬Ï‫ ر‬post MI = ٦ .
.......
🌹What will increase the prevalence when there is no change in incidence ?
A. cLose of follow up✅ B. better diagnosis method C. poor community

1CNS
🌹Elderly with positive bilateral rinne's test and Weber test is better on the
right ear. What is the diagnosis?
A. Conductive hearing loss of right ear B- left sensorineural hearing loss.
🌹patient with sinusitis did scope that injures the nerve that caused loss of sensation of
lower eyelid and superior lip what is the nerve injured ? Infraorbital
🌹 Old pt with hx of recent travel , came with difficulty getting aroused/awaken, he
report multiple falls, examination shows no head wounds and skull intact , most likely
diagnosis:? A. post concussion syndrome B. chronic subdural hematoma
🌹- 50 yo pt with 3wk H of fatigue and tiredness and tonge and lower limb
fasciculation . Dx?A.myasthenia gravis C.motor and neuro disorder✅ D.mono neuropathy
🌹 A woman presents with left sided facial twitching and spasticity throughout her body;
she also has unilateral tongue fasciculation. diagnosis is A. myasthenia gravis .b B.
Myasthenic syndrome. C. Mononeuropathy. D.Motor neuron disease
🌻Spasticity > upper motor neuron
🌻Fasisculation> lower motor neuron
🌻both features upper and lower > amyotrophic lateral sclerosis

🍒Case of stroke came in 4 hours what is the most important next step ?
A-asprin B-ct scan ✅✅ C-mri brain ...
🍒Female patient presented with lethargy Ct scan shows area of hypodensity in the
frontotemporal lobe A. brain abscess. B. herpes encephalitis ✅
¤‫]ﺲ =ﺷﺎف اﺳﺪ وﺿﻊ •ﺪە ﻋ‬À‫ ﻫ‬frontal ‫وﻫﺮب‬
🍒 Pt has change in behaviors what’s the Dx, = Frontotemporal dementia

🌹 patient c/o 4 months of hearing loss now come with 7 nerve palsy
A. rumsy hunt B. brain tumor C.bell's palsy
https://www.ncbi.nlm.nih.gov/m/pubmed/17300646/
🌹pt with paraplegia and high ca = immobilization
🌹Management high ca = A. normal saline B. albumin and normal saline C. forsmid iv

🌹pt with transient visual loss=TIA


🍓Patient with neck pain radiate to the left shoulder and numbness of the arm . Long
thoracic nerve✅
🌹 patient develops hyperthermia, generalized muscle rigidity, altered mental status,
tachycardia, hypertension. labs show elevated creatine kinase. He started
metoclopramide two weeks ago to treat diabetic gastroparesis. Which of the following
drugs can also cause these symptoms? Fluphenazine🌹
. . .....‫ وﺗرﻓﻊ ﻧﺑﺿﺎت اﻟﻘﻠب واﻟﺿﻐط‬alternated mental = ‫ = ﻓﻠوس = اﻟﻔﻠوس ﺗﻠﻌب ﺑﺎﻻﻋﺻﺎب‬fluphen = ‫رﺑط‬

🍓pt with facial asymmetry , moth deviation, absence of forehead wrinkles , what’s the
nerve affected: A. ll B. V C. Vll✅ D. X

🌹 Double vision, when he look straight the lt eye deviated to nose side, Lt. eye cannot look
laterally? A. Rt. 6 CN palsy. B. Rt. 3 CN palsy. C. Lt. 6 CN palsy✅. D. Lt. 3 CN palsy
🌹Case of dm pt with sx of 3rd nerve palsy (ptosis, double vision when looking laterally)
Dx? Dm 3rd nerve neuropathy
🌹Pt with 6th nerve palsy can't abduct his left eye with diplopoa : Left 6th nerve plasy
🌹) _ ( Left eye unable to abduct) , when looking forward there’s squint towards the nose
which nerve affected: - left 6th

🌹right eye. Pupil reflex normal in both eyes and all other cranial nerves are
normal diagnosis? A. 3rd cranial nerve palsy B. cavernous sinus thrombosis

🌹Patient has sudden onset unilateral facial paralysis. Which medication is proven to
reduce duration of symptoms? -Steroids ✅
sudden onset unilateral facial paralysis = ((Bell palsy)) = treat by = steroid .
‫ اﻟﻮﺟﻪ‬fg‫ﺪ =ﺳ‬h‫و‬fm‫ﻐﻄﻴﻬﺎ =ﺳﺘ‬h‫ وﺟﻬﻪ و‬fg‫ﺴ‬d ‫ﻞ واﺣﺪ وﺟﻬﻪ ﻧﺼﻪ ﻣﺸﻠﻮل =ﻻزم‬V‫ﺎﻟﻤﺮض =ﻧﺘﺨ‬O ‫ﻂ اﻟﺪواء‬JI‫ﺗﻌﺎﻟﻮا ﻧ‬

🌹Lewy bodies dementia: hallucinations with dementia and sometime parkinsonism.🌹


🌹 Parkinson's + early dementia + fluctuating cognition + visual hallucinations?
A. lewy body dementia B. – Parkinson C. - alzheimer D. - corticobasal dementia
🍒 in CTmultiple atrophy? A. vascular dementia B. alzheimer✅
🍒70 year male HTN with progressive decline in cognitive state Mri shows Periventricular
white matter Hyperintensities what is the Dx:
A. -vascular dementia✅ B. -Alzheimer dis C. -Normal pressure hydrocephalus ...

🍓elderly k/c of HTN with gradual loss of cognitive function, especially executive function
on imaging there is *diffuse* periventricular white matter hyperintensity what us the
cause: B. Vascular✅ C. Alzheimer.
🌹old male patient came with his brother, brother complaining of change in
personality,mild forgetful, MRI show cortical atrophy : Answer:Alzheimer disease ✅
🌹Long scenario about Alzheimer then MRI show: atrophy of hippocampus
and temporal lobes.
🌹Vascular dementia: history of major atherosclerosis risks DM, HTN, Smoking,
hyperlipidemia🌹
🍓Recent memory loss and behaviour changes /multiple brain infarct: Vascular Dementia
🍒elderly hypertensive and has cognitive changing , MRI brain showing: periventricular
hyperintensity : vascular cognitive impairment
🌹Elderly with unsteady gait, later he developed forgetfulness
A. Alzheimer disease B. Frontotemporal dementia
C.Normal tension hydrocephalus✅🌹 D. Cruetzfelt jakob disease
🌹Old M, drowsy and forgetful, Hx of femoral embolization 12 hrs ago. Ddx?
A- Alzheimer's, B- Lewy body dementia, C- multiple infarct dementia✅
‫ﺎﻟﻤﺦ ﻓﺴﺎر‬B ‫ ﻣﻮت اﺟﺰاء‬9‫ﻪ ﺟﻠﻄﺎت ﺻﻐ_^ة وادت ا‬w‫ﺪ راﺣﺖ ﻟﻠﺪﻣﺎغ وﻋﻤﻠﺖ ﻓ‬w©‫ﺎﻟﻘﺪم ﻣﺘﺤﺮﻛﺔ وﻻﻧﻬﺎ ﻣﺘﺤﺮﻛﺔ ا‬B ‫ اﻟﺠﻮاب ﻻن ﺟﺎت ﻟﻬﺎ ﺟﻠﻄﺔ‬±: } ‫ﻣﻨﻄ‬

🍒 in CT ventricular enlargement? normal pressure hydrocephalus


🌹Poorly controlled diabetes with orthostatic hypotension (no change in HR) =
Autonomic neuropathy 🌹
🍓 pt have difficulty feeding can not put fork to his mouth and have traffic accident and
where is lesion = cerebellum ‫ﻔﻤﻪ‬u ‫ﻣﺎﻓ~ﻪ ﺗﻮازن ﻣﺎ•ﻘﺪر •ﻀﻊ اﻟﺸﻮﻛﺔ‬

parkinsonism

🍒🍒Symptoms Of Parkinson diagnosis: Resting tremors 🍒🍒


🍒Case of Parkinson dis. Came with right hand tremor. Best drug ? Levodopa🍒🍒
🌹 Pt diagnosed with parkinsonism, her only complain is right hand tremor that disturbs
her manual activities. What medication to use?
A. Propranolol B. levodopa/carbidopa C. Amitriptyline
🌹Clear case scenario with Mask face, bradykinesia, tremors whats the dx? Parkinsons 🌹
🌹patient complaining of progressive tremors interfering with writing and eating, head
bobbing, the treatment is:Propranolol🌹(Not Levodopa)
🌹pt with bilateral hands tremor that interfere with her writing. What is the best
medication for these symptoms? A. levodopa/carbidopa B. Propranolol ✅✅
‫ﻘﻮﻟﻚ‬2 ‫ ﻻن ﻟﻤﺎ‬bilatar ;: < ‫ ﻓﻜﺮوا‬٦٠ ‫&ﺾ اﻗﻞ ﻣﻦ‬%‫ وﺧﺎﺻﺔ ﻟﻮ ﻋﻤﺮ اﻟﻤ‬hyperthyroidsm ‫ﻠﻮك ﺑ?ﻨﻤﺎ‬B ‫ﺎﻧﻮل =ﺑ?ﺘﺎ‬Ê‫^ ب ﺑﺮو‬É‫ وﻋﻼﺟﻬﺎ اﻟﻬﺎﻳ‬parkinson
‫; اﺗﺠﺎە واﺣﺪ ﻣﻦ اﻟﺠﺴﻢ‬: < ‫ و&ﺎﺛﺮ‬٦٠ ‫ ﻓﻮق‬Ì
: É2
🌹Scenario of Parkinson’s and asked about the diagnosis? Mask face, bradykinesia,
resting tremors ....
🌹Parkinson’s case and asking about the site of the pathology: Substantia nigra
🍓Shuffling gite , tremor what is the diagnosis? parkinson
parkin‫ ﺑﺎرﻛﯿﻨﺞ‬shuffli ‫رﺑﻂ = ﺷﻮﻓﻠﻲ‬
🍓 Parkinson + hallucinations, what is the most important indicator he will develop
dementia: forgets appointment✅✅
…………………….
🍒pt have difficulty feeding and can’t put fork to his mouth and have traffic accident and
where is the lesions ? Cerebellum 🍒
💐15 years old had her menstruation 2 months back, she's irritable, not cooperative with
the examiner= Behavior💐
🌹woumen with abnormal beheviar and agitantion = frontotemporal dementia .🌹
🌹Case of multi neurological symptoms and in deffernt parts of body ( Multiple sclerosis ) .
Ask what is the best diagnostic test :- A. EMG B. NCV C. MRI brain and spain ✅
🌹 Case known multiple sclerosis what to give: A. interferon B. IV corticosteroids
🌹Pt with cranial n ocular manifestations and lower limb neurological manifestations and
ask about investigation : A. brain and spinal cord MRI🍒 B. thoracic spine CT
🌹 Female complaining of decreased ability to walk the same distances, feels pain in legs,
after some distance she reaches her limit and cannot walk, she feels better when walking
while leaning forward, or when walking uphill. diagnosis?
A. Spinal stenosis B. Disc herniation

🌹 Lumbar Spinal stenosis, mx?


A. Lumbar laminectomy. B. Biofeedback. C. Physiotherapy D. Epidural analgesia.

🌹 Patient with nephrotic syndrome , abdominal pain what is the complication that
happened? Peritonitis (Peritonitis is a complication in nephrotic syndrome)

Meningitis
🌹 Meningitis with pic showed pateci ask what to give his brother? Rifampin 🌹
🌹 pediatric Prophylactic antibiotic for relative of child with meningitis =-Rifampicin
🌹80 old with meningitis , what you give people contact with him:
Ciprofloxacin ( Dont choice Rifampin) bez he is old .
‫ ﻋﻠﻴﻬﻢ‬Ò£‫ ﻧﺼ‬Ò£‫وا =ﺻ‬Ò£‫ﺎر اﻟﻌﻤﺮ ﻧﺨﺘﺎر ﺳﻴ‬y‫ﻛ‬
🌹 What to give Pilgrims against african meningitis ? cipro
🌹 15 years old case of meningitis LP to be done what to do:
A-Take consent form parents and assent from patient ✅ Òç‫ ﺳﻨﺔ ﻟﺴﺔ ﺻﻐ‬١٥ ‫ان‬
B-take consent from parents and donʼt assent patient D-consent patient only
🌹 15month with b'cteri'l meningitis tx? Vncomycin + ceftrixone ‫ﻓﻤﺘﻮا و ﺳﻔﻦ =ﻋﺸﺎن ﻧﺘﻌﺮف‬
‫؟‬ñ• ç ‫ =ﻣﻨ‬ñ
• ç ‫اﻧﺖ ﻣﻦ ﻓ‬
🍓22 years old pt e typical meningitis culture show gram + cocci in clusters. ask about ttt?
A. ceftra, vanco and steroid B. ceftra and vance
🍓Gram negative coccobacilli in meningitis: A. Ciprofloxcine B. Ceftraxione ✅
🍒drug for meningitis + endocarditis= Ceftriaxone + vanco For both
🌹 3 day old with sx meningitis and csf analysis (Beta hemolytic and catalase +ve ) ,
empirical therapeutic : Ampicillin

🍓Late complication of meningitis: A. Ataxia B. Seizure C. Deafness ✅


🍓what is the most common bacterial meningitis in elderly? Listeria✅

‫–ﺎ =ﺳﻦ‬Òç‫ (اﻟﺴﻦ = )ﻟ¨ﺴﺘ‬ŒŽ Òç‫ﻂ =ﻛﺒ‬Ï‫ر‬
🌹 Meningitis protector + pnumonia = Droplets percution
• ç ‫ﺴﺎل ﻣﻨ‬E‫رﺑط = اي اﺣﺪ •ﻜﻮن (ﺣﻖ ﻧﻤ~ﻤﺔ و‬
‫ﺴﻮي ﻟﻪ دروب وﻧﻜﺴﻠﻪ ﻣﻦ اﻟﺤ~ﺎة‬D)= Òç‫ ﻛﺜ‬ñ
🌹 pt with signs of meningitis, LP show gram negative diplococci , what’sthe precaution :
A. standard B. droplet✅meningococcal C. contact D. airborne
🌹Bacterial meningitis case, asking what you expect to see on CSF:
Low glucose, High protein, Neutrophil predominance
🌹 Pediatric patient presented with meningeal irritation sx, CSF showed: high WBC,
mildly elevated protein, normal level of glucose, your most likely dx: (no other details, A.
Diabetic coma B. TB meningitis ‫ ﻟﻮ اﻟﺠﻠﻮﻛﻮز „ﺎن ﻣﻨﺨﻔﺾ‬C. Viral meningitis ✅
🌹 man from India has meningitis symptoms with high protein, high lymphocytes and low
glucose in CSF. What kind of meninges? T.B meningitis.
🌹 Pediatric patient presented with meningeal irritations sx, they have done CT scan
which showed: low attenuation of the temporietal area, I don’t remember they provided
CSF results, your dx: A. Bacterial meningitis B. Brain Abscess
C. HSV meningitis✅ Bez =temporietal area.
🌷menigitis with low attenuation of the temporietal area,=HSV meningitis🌷
🌹with symptoms of meningitis, how many hours after beginning antibiotics before
discontinuing isolation: = 24h 🌹 🍓Isolation for meningitis : 24h 🍓
🌹meningitis CSF methicillin resistant staph aureus which type of antibiotics to use MRSA
treatment Vancomycin
🍒case of bacterial meningitis, With high protein, low carbohydrate, predominant Cells?
neutrophils
🍒case of tuberculous meningitis with high protein and low glucose , predominante Cells?
Lymphocyte
🌹Pilgrims from Africa came for Hajj, vaccinated by meningococcal vaccine, what other
prophylaxis on arrival? A. Isolation 6 days B. isolation 10 days. C. 500 mg ciprofloxacin oral
🌹Child (days) confused, irritable for 2days Then rash developed
A. measles B. Meningococcemia
......
🍒51 yrs ( classic case of meningitis ) organism: A. listeria B. streptococcus pneumoniae
🍒Infant with meningitis what is the most likely causative organism= Group B Strep
(ñ• ç ‫ = ﻣﻨ‬meningitis) ‫ ورﺿﻴﻊ ؟ ﺟﺎﻟﻪ ﻣﻦ‬Òç‫=ﺟﺮوب ﺑ|ﺘﻪ( ﺟﺎء ﻫﺬا اﻻﺗﻬﺎب ب ﻣﺦ ﻃﻔﻞ ﺗﻮە ﺻﻐ‬group beta)
‫ﻻن „ﻞ ﻣﺠﻤﻮﻋﺔ ﺟﺎ•ﺎ ﺑ|ﺘﻪ وﺟﺎﻟﺲ •ﻤﺴﻜﻪ‬
🍒 meningitis organism in adult: Strep Pneumonia

1Shock
Old age female DM HTN admitted to ICU due to urosepsis and septic shock Vitals low PB
SpO2 84 temp 38 HR 120 given O2 and not improve Cardiac examination is clear High WBCs
X Ray shows bilateral lung infiltrate what is the case of her respiratory problem ?
A. ARDS. B. Cardiogenic shock. C. Hypovolemic shock D. Fluid overload

female after RTA, congested neck veins and muffled heart sound, what else would make
the prognosis worse: A- BP 88/50 ✅✅ == (tamponade)

🌹Septic shock least effect management ?A. Steroid✅. B. Inotropes. C. IV fluid. D. Ab


.. . .
🌹Elderly female Underwent surgery, got tachycardia and hypotension in operation, no
more details. Why? A- Hemorrhage✅. B- Cardiogenic shock

🌹Pt after repair of abdominal aortic aneurysms. Despite fluid resuscitation vitals still
unstable. There’s negligible urine output since he is on Foley catheter one
hour ago. Periphery was warm to touch. BP: 90/.., HR:120, RR:38, temp:36.6, type of
shock: A. Septic B. Hemorrhagic C. Cardiogenic D. Anaphylactic
🌹After repair patient had got Shock iv fluid resus failed warm extremities and hypotension
which type? Septic.
🌹chest pain did heart surgery, in shock with high cardiac output and decreased
peripheral resistance? A. Septic ✅✅ B. Cardiogenic C. Hypovolemic D. anaphylactic
🌹long scenario about SIRS with labs?
A. sepsis B. severe sepsis C. SIRS. D. bacteremia
‫ ال‬SIRS ýŽ • ‫ •ﻌ‬Systemic inflammatory response syndrome = ‫ﺐ‬Þ‫ﺴ‬7 ‫ „ﻞ اﻟﺠﺴﻢ ﺣﺼﻞ ﻓ~ﻪ اﻟﺘﻬﺎب‬ýŽ • ‫•ﻌ‬
‫ اﻧﻔﻜﺸﻦ او ﺣﺼﻞ‬burn ‫ ـﻊ‬Ï‫ وﻟﻬﺎ ار‬criatira :
🌻fever >38.0°C or hypothermia <36.0°C
🌻tachycardia >90
🌻tachypnea >20 breaths
🌻WBC= >12,000/l or low WBC leucopoenia <4,000
‫ اﻗﻮل اﻧﻪ ﻋﻨﺪە‬în‫ ﻃ}ﺐ ﻣ‬Sepsis ‫؟ اذا ﻓ~ﻪ ﻧﻔﺲ ال‬critira ‫ﺎﻟﺴ|ﻨﺎر–ﻮا ﻗﺎﻟﻚ ﻋﻨﺪە‬u+ ‫ﻨﺎﻫﺎ‬é‫ ﻛﺘ‬ÐŽ‫ ا‬infection .
‫ اﻗﻮل ﻋﻨﺪە‬în‫ ﻃ}ﺐ ﻣ‬sever sepsis ‫؟ ﻧﻔﺲ ﻋﻼﻣﺎت ال‬sepsis ‫ﺲ ﻗﻠﻨﺎ‬7 ‫ﻂ‬y‫ﺎﻟﻀ‬u sever ‫ﻏﺸﺎن اﺣﺪ اﻻﻋﻀﺎء‬
• • ç ‫اﺗ|ﻨ‬ÒçÄ‫ ﻗﺎﻟﻚ ﻣﺜﻼ اﻟ‬ý•‫•ﻌ‬
‫ وﻇﺎﺋﻔﻬﺎ‬ŒŽ ‫ﺎﻟﺠﺴﻢ ﻓﺸﻞ‬u (organ dysfunction) ‫ او ﻗﺎﻟﻚ‬¤6‫ ﻣﻌﻨﺎﻫﺎ ﺣﺼﻞ ﻓﺸﻞ اﻟ‬ÐŽ‫ ﻋﺎ‬ñ Ž

‫ﺪ‬yÄ‫ اﻟ‬ŒŽ ‫ ﻓﺸﻞ‬ýŽ • ‫ﺪ ﻋﺎﻟ~ﺔ •ﻌ‬yÄ‫وﻇﺎﺋﻒ اﻟ‬.
‫ اﻗﻮل ﻋﻨﺪە‬în‫ ﻃ}ﺐ ﻣ‬septic shock ŒŽ • ‫ﺲ ﻧﻀ~ﻒ „ﻠﻤﺔ وﺣﺪە اذا ﺷﻮﻓﺘﻮﻫﺎ اﻋﺮﻓﻮا دﺧﻞ‬7 ‫ ؟‬shock 4 Ž ‫و‬
Hypotenstion .
.....
🌹 type of shock in patient Hit by car and after 3 hour suddenly has distended abdomen?
A.hypovolemic ‫ﻒ‬wÙ‫✅ ﻧ‬
🌹mva case with tachy + hypotention + low pulmonary wedge presser = type of shock =
hypovolemia 🌹
🌹pulmonary wedge pressure: low,
high systemic vascular resistance Type of shock? A. Hypovolemic B. Cardiogenic

🌹Signs of cardiogenic shock after 2 hours low bp and HR?


A. Dobutamine ✅🌹. B. atropine
In patients with cardiogenic shock due to decompensated heart failure, dobutamine decreases left
ventricular end-diastolic pressure and raises blood pressure by increasing cardiac output
.......
🌹tachy + hypotention + noraml air entary + rise JVP = Cardiogenic shcok 🌹

🌹 IHD & DM pt e week pulse. What type of shock you suspect = Cardiogenic
‫ﺾ ﻣﺼﺎب ب‬wv‫; ﻻن اﻟﻤ‬ n
o ‫ ﻣﻨﻄ‬iscemic heart dis .
🌹heart failure , septic and uroseptic shock develop hypoxia , bilateral crepitation Giving
fluid and inotrop still have hypoxia on mechanical ventilation = Cardio-pulmonary 🌹
ICU paitent with diabetic , heart failure , septic and uroseptic shock develop hypoxia ,
bilateral crepitation Giving fluid and inotrop still have hypoxia on mechanical ventilation?
I think what is the cause of this hypoxia??
a-Ards b-Cardio-pulmonary ✅✅ c-Volume overloud
‫ﺤﺼﻞ‬#‫ ﻓ‬، ‫ﻢ‬°‫„ اﻟﺮﺋﺔ ﻓﻴ©‹ا‬6 4 ‫ ﻣﻮﺟﻮد‬56‫ﻞ اﻟﺪم ا‬¢ ‫ﻄﻠﻊ‬ž‫ﺾ و‬Ik‫ﻢ اﻟﺴﻮاﺋﻞ )ﺎﻟﺮﺋﺔ اﻟﻘﻠﺐ ﻣﻮ ﻗﺎدر ﻳ‬°‫ﻪ ﻳ©‹ا‬#‫ﺤﺼﻞ ﻓﺸﻞ ﻓ‬7 ‫ اﻧﻪ ﻋﻨﺪە ﻓﺸﻞ )ﺎﻟﻘﻠﺐ وﻟﻤﺎ‬، ‫ ﺳﺎر ﻣﻌﻪ‬56‫ا‬
4 Œ ‫ ﻟﻮ ﻋﻜﻴﻨﺎ ﻟﻪ اوﻛﺴﺠ‬³©‫ ﺣ‬، ‫ﺎاااﻧﺔ ﺳﻮاﺋﻞ‬#‫ﺪ ﺟﺪا )ﺎﻟﻨﻔﺲ ﻻن اﻟﺮﺋﺔ ﻣﻠ‬7‫ﻟﻪ ﺿﻴﻖ ﺷﺪ‬
‫¶ﺪة )ﺎﻟ¯ﻼم اﻟﺤﺎﻟﺔ‬µ‫ اﻟ‬، ‫ﻪ ﻣﺪرات ﺗﻄﻠﻊ اﻟﺴﻮاﺋﻞ‬#‫¬ اﻟﻌﺎﻟﻢ ﻣﺎ ﻫﻴﻨﻔﻊ ﻻزم ﻧﻌﻄ‬
‫ ﻫﺬە ·ﺴ¸ ب‬cardio _ pulmonary ‫ ﻣﺸ¼ﻠﺔ ﺣﺼﻠﺖ )ﺎﻟﻘﻠﺐ أﺛﺮت ﻋ• وﻇﺎﺋﻒ اﻟﺮﺋﺔ‬³6 4 ‫ﻌ‬7 .
🌹Case of cardiogenic shock .. ?decrease cardiac output✅🌹

🌹 Scenario about old female with cardiogenic and ischemic heart disease,, asking about
type of shock she had? cardiogenic shock ✅ ✔
🌹 other Scenario about trauma and hemorrhage,asking about type of shock?
Hypovolemic shock ✅ ✔
Spinal shock occurs during the acute phase following spinal cord injury and is a
Neurogenic shock, part of spinal shock, consists of severe bradycardia and hypotension
🌹Paintnt MVA,hypotensive tachycardia tachypnia can shrug the shoulder but cant
move elbow or lower extremities
the cause of this hypotension = upper spinal injury 🌹
1Psycatric
🌹Patient parents complain of signs and symptoms of schizophrenia in their son
What treatment option is appropriate:
A. Oxcarbazepine << bipolar. B. Amisulpride << treat Schizo
🌹Psychiatric patient you ask him “ do you think you have mental illness “ this is
Called = insight
🌹Patient with insomnia worsening sad mood, low self esteem, hopelessness for 2 year ?
A. Depressive disorder B. Organic depressive disorder C. Minor depression D. Dysthymia
🌹family leave him with his friends in party, then returned back home with SOB and
swelling. What is the cause = (Food allergy) give Adrenalin🌹
🌹 Female come to ER complining from her neighbor ( abuse her) and talk with hem
even he doesn’t in ER Dx -Dulsions and hallsuntion
🌹 Sleep disorder:
• Stage 1 > easily awake. • Stage 2 > deeper stable diff to awake. • Stage 3 > deeper, GH.
• Stage 4 > dream 🌷stage 4 = dream 🌷
🌹 O male, lost his partner, he is in bad & depressed mood and for 2 months. Dx?
A- Minor depression. B- Major Depression. C- bereavement✅✅
‫ ﺷﻬﻮر‬٣ ‫–ﻦ او‬À‫ ﻣﻦ ﺷﻬ‬ÒôÙ‫ ﻟﻮ „ﺎن ا‬، ‫ﺘﺌﺎب‬Ù‫ ﻫﺬا ﻣﺎ ﻧﻄﻠﻖ ﻋﻠ~ﻪ ﻣﺮض ا‬، ‫ﺘﺌﺎب‬Ù‫ ﺣﺰن وا‬ŒŽ • ‫~ﺪ ﺑ~ﻈﻞ‬Ù‫ ا‬Œ•‫واﺣﺪ اﺑﻮە ﺗﻮ‬
‫ ف ﻟﻮ ﺣﺎء ﺳﺆوال وﻓﺎة‬، ‫–ﻦ‬À‫ ﺣﺴﺐ اﻟﻤﺼﺎدر ﺷﻬ‬¤‫ وﻓﺎة او ﻓﻘﺪان ﺷﺨﺺ ﻋ‬ŒŽ • ‫ة اﻟﺤﺰن‬ÒÓ‫ﺘﺌﺎب ﻻن ﻓ‬Ù‫ﻧﻘﻮل ﻋﻠ~ﻪ ا‬
‫ة ﺣﺰن‬ÒÓ‫ﺴﻤ~ﻪ ﻓ‬D ‫–ﻦ‬À‫ ﺷﺨﺺ وﻇﻞ زﻋﻼن وﻣﻜﺘﺌﺐ ﻣﺪة ﺷﻬ‬bereavement .
🌷lost his partner he is bad & depressed mood for 2 month == bereavement🌷

🌹 Women after peaten by her husband and came to police , what is the most common
cause of domestic violencersfq q ‫? ﻋﻨﻒ ﻣ‬
- cultural background - Socioeconomic state ✅ ±: } ‫ =ﻣﻨﻄ‬Ï:‫ ﻣﺴﺘﻮى اﻗﺘﺼﺎدي واﺟﺘﻤﺎ‬- Beliefs about the
right of the other gender
🌹 7 y Child with vomiting dx as schools phobia and GAD tx? CBT + SSRI

🌹) Patient presented to you with loss of apathy and delusions what’s the treatment
A) TCA. B) TCA + cognitive behavioral therapy C) cognitive behavioral therapy ✅
‫ﻠﻤﺔ‬6‫ ﻫﺬە اﻟ‬apathy ‫ﺎﻻە و‬y‫ ﻣﻌﻨﺎﺗﻬﺎ ﻻ ﻣ‬cognitive ‫ﻂ ﺑ|ﻨﻬﻢ‬ÏÀ‫ﻓﻤﻦ اﺳﻤﻬﺎ ﻧ‬. ‫ ﻣﻌﻨﺎﻫﺎ ادراك‬.
Dx is Delusional disorder , Delusional symptoms + apathy = schizophrenia most likely which
treated by antipsychotic not antidepressants
🌹)_ Pt thinks that aliens are in the backyard he knows they don’t exist, but is
overwhelmed by the idea. He feels he’s going crazy dx: Obsessions✅

🌹Patient complains of sleeping difficulty, being irritable, anxious, problem with


concentration, DDX? “mood disorder not otherwise specified"
A. generalised anxiety disorder. B. anxiety with depression
🌹 30 yo, 3-months history of talking to aliens‫ﺎﺋﻨﺎت ﻓﻀﺎﺋ}ﺔ‬á. Next step?
A. anti-psychotic B. anti-depressant C. bereavement console (Mot sure about spelling)
🌹Patient saying that he has a disease, and all the doctors he visited told him he didn’t have
any thing and the tests are normal , but still he is saying that he has, diagnosis?
A. Somatization B. Something pain disorder D.Hypochondriasis
Somtization : symptom. Maingering :gain attention. Hypochodriasis :think have cancer

🌹14 years old was investigating his father's death then he said that he saw his dead father
then he walked naked suddenly when asked why doing so said that his father told him to do
so and his proud of him his symptoms was for 3 days What is the diagnosis of this patient?
A. Schizophrenia B. Schizoform C. Schizoaffective D. Acute psychosis
‫»ء ﺣﺎد وﻣﺆﻗﺖ‬¼
Ž ‫ﻴﻮت‬Ù‫ ا‬.

🌹Patient came to ER recurrently with palpation and fearing of die (no sweating
or loss of consciousness) A. Panic disorder✅ B. Agoraphobia

🌹Patient with schizophrenia on antipsychotic medication ( I forget the name ) he develop


oro-facial abnormal movement what the diagnosis? A. Tardive dyskinesia B. Catatonia

🌹 out come of 45 y/o pt with recurrent schizophrenia on maintenance therapy :


A. complete remission B. 70% will have good quality of life.
C. 50% will have impaired quality of life D. One third of pateint will have good prognosis

🌹 OCD case clear Lady check doors and oven and kids school things many times, can't
sleep if did not do that ?
A. anxiety B. OCD‫وﺳﻮاس ﻗﻬﺮي‬

🌹Patient with recently depressed after being promoted to manager, what to do?
A. TCA B. SSRI C. Psychotherapy
🌹pt with RA, diagnosed with something, doctor to want to start steroids Pt afraid of side
effect of steroids , what should you do ? A. Enforce the need of steroids
B. tell him you will die if you did not take them C. tell him the SE are not serious
Answer is : C By exclusion plz check last answer in the exam

🌹 depressed pt and doctor ask the pt (do you think that's you have mental illness) the
doctor assess? A. insight B. judgment C. guilt
🌹 Male with asthma has sx of social phobia he said his sx aggravated or increased with his
inhaler ? -sertraline 🍒
🌹‫؟؟‬ñ • ç ‫ﻄﻞ ﻓﻬﻮا ف اي ﻣﺮﺣﻠﻪ دﺣ‬y‫ﺾ ﻗﺮر وﺧﻄﻂ ﯾ‬QÀ‫ و„ﻞ ¼» واﻟﻤ‬ñ
• ç ‫ﺣﻠﻮ ﺧﻄﻮرة اﻟﺘﺪﺧ‬è¼ ‫واﺣﺪ اﻟﺪﻛﺘﻮر‬
Ž
A_Precontemplation B_contemplation C_Preprantion

🍒ADHD child What'll we do? A. parent training program B. behavioral technique✅


Less than 6y : CBT. More than 6y : CNS stimulants
🍒 15 Y. O girl had a Problem with her friend. Next day when she meets her friend she go
away from her : A. Denial. B. Avoidance✅
🌹 Patient after being told by physician that he have lung cnacer he respondes : “is it a
mistake?” What kind of response is this ? A-denial ✅✅ B-bargaining

🌹 Elderly with depression. He is on 30 mg amitriptyline taken at night. In the morning, he


feels dizziness. What will you do?
A. switch him to SSRI B. advice to take it before meals
C. change dose to 10mg three times daily✅ D. advice to take it in the morning
🌹20 years old girl, BMI 20, inactive for 5 years:
A- behavior eating disorder B- Chronic depression✅ C- IV drug user D- Obesity

🌹 Women sabling die cry for 5 day not sleep for 2 days, what short course management?
A. Fluxatine B. Imytriptand C. Clonacolam D. Flurazepam ✅
🍓 Auditory hallucination + think of food poisoning.. Dx ?
A. Delusion B. auditory hallucination
vote: delusional disorder doesn't cause psychosis, it's Fixed unshakable beliefs
🍒 other is paased away , he said she is present and no one can see her
A. delusionB. visual hallucinations✅
🍒pt with feeling of grandiosity + hallucination + delusion, What he has !?
A. neurosis. B. psychosis✅. C. personality disorder

🍒A patient presents with auditory and visual hallucinations for the *first time*. most
appropriate treatment? A. Olanzapine B. SSRI. C. CBT. D. Antipsychotic plus CBT ✅
🍓Girl depressed as she feels abandoned by her therapist , admitted , on admission hears
voices then denies that she did , caused conflict between doctors: (answer is type of
personality disorder ): Borderline personality.

🍓 Student with dulosin , return to normal without treatment?


Brife psychotic disorder = þŽ~‫–ﻒ ﺛﻢ رﺟﻌﺖ ﻟﻠﻄﺒ‬À‫ﻣﻦ اﺳﻤﻬﺎ ﺑ‬
🍓px his father died and he devolved sadness, delusion and disorganized behavior and
speech for 3 days. After that he became normal
a. schezo effectiveness b. Schezoform. C- brief psychotic disorder
🍓His daughter wants to join sport class ,confrontational communication?
why you're upset about this?
🌹Patient came to the clinic for a routine follow up. He is a smoker but has no plan to quit
smoking. What are you going to do next
A. Set a "stop date" B.Tell him about your opinion about smoking ✅
C. Prescribe nicotine patches D. send him to smoking quit group
...........
🌹Old pt with signs of loss of interest, suicidal ideation, wt loss, sleeps less for 2 months?
A. Major depression ✅‫ﺎﻻﻧﺘﺤﺎر‬u ‫ ﻻن ﻓﻜﺮ‬B. Minor depression C. Bipolar
....... .........
🌹Ebola case want to leave hospital A.Call security ✅ B. Contact health committee
🌹91 y/o male pt with multiple diseases developed adenocarcinoma? comfort care mean =
((Palliative care ))
🌹Elderly pt 91 yrs, a known case of dementia. He has adenocarcinoma, its
primary source cannot be determined, there is multiple hepatic lesions,
and lesions in other different organs I do not remember (widespread metastasis as I
understood). How do you manage this pt?
A. Palliative systemic chemotherapy. B. Comfort care. C. Biopsy of the lesion
‫ﺎﻟﻌﻤﺮ‬u ‫ة‬Òç‫~ﻤﺎوي ﺟﺪا ﻛﺒ‬Ä‫ﻻن ﻣﺎﻫﺘﺘﺤﻤﻞ اﻟ‬

🌹child c/o school days abd pain = positive reinforcement 🌹


🌹What’s it the positive reinforcement ? support Behavioral therapy
‫ر`ﻂ =ﻋﺸﺎن ﺗﻜﻮن ﺷﺨﺺ ﻗﻮي‬force ‫ و ﻣﺘﻔﺎﺋﻞ‬postiv ‫ﻚ ﻣﻊ اﻟﻨﺎس‬á‫ ﻻزم “ﻜﻮم ﺳﻠﻮ‬behivior ‫ﺲ‬iö‫ﻛ‬
.........
🌹 Very old pt in ICU with carcinoma and other trouble in breath.. surgeon want to do sm
surgery to improve breathing but not sure if the pt will survive during the surgery.. what
do? A.let the pt sign very high consent B.No need to do it cuz there is no sure
improvement D- discuss with family members
….
🌹Ratio of prevalence? Predective value 🌹. Prevalence = predictive
( Don’t choice Odd ratio ×)
........
🌹Elderly with pneumonia become sever agitated?
A. Give him haloperidol ✅ B. Put him in a dark room
C. Let his family to stay with him
Answer is: Delirium best to treat the cause. and try to do anything to re-orient the patient,
so C is the answer, if severe agitation > haloperidol .
🌹worst prognostic sign of inhalation injury? A.Tachycardia B. Hoarseness of voice✅
C.O2 sat = ‫ﻛ~ﻤ~•ﺎل =ﻫﻮرﻧ¨ﺲ =ﺣﺮ =ﻣﺎدة ﺣﺎرﻗﺔ‬
🌹Pt with strong hx of cancer what is the best advice to give?
A.Keep being active ✅✅‫ﺸﺎﻃﻚ‬D ¤‫ ﺣﺎﻓﻆ ﻋ‬B. Lose weight
C. Nutrients only from food D. Something unrelated
🍒Being active at recommended levels and keeping to a healthy weight may reduce the risk
of certain cancers coming back. Being physically active may reduce your risk of some of the
late effects of cancer treatment.
🌹Natural history stages, when best to do screening ?
A. pre disease stage B.Pre clinical stage C. Pre clinical disease stage
D. Pre recovery, disability, death stage

Vaccine
🌹vaccines contraindicated for immuncompramised baby? live attitude🌹
🌹Pediatric Pt with seizures what vaccine not to give? : DTaP 🌹 ‫ﻌﻤﻞ «ﺸﻨﺠﺎت‬: ‫ﺔ‬S¬‫ﻣﻦ اﻋﺮاﺿﻪ اﻟﺠﺎﻧ‬
🌹Dtap vaccine “least” risk factor A. Fever 38.4 B. Redness C. Rash
🍓New born you want to give vaccine , the mother told you that she had a new born died
after vaccination due to immune dif, what you will do :
A. Give BCG B. Dont give BCG✅ bez its live vaccin. C. Give immunoglobulin
🍓Adolescent has missed the second dose of varicella vaccine, what will you do now:
A. measure varicella antibodies B. give double the missed dose C. give the same dose✅
🌹57 year with CAD which vaccine? influenza ( Dont choice hemophila influnza , there is
diffrent
🌹Hepatitis b post exposure prophylaxis? IVIG + vaccines if un immunized.
🌹Pretem baby 1.7 kg asking about vaccines?? delay HBv🌹
🌹Note: Premature infant should be vaccinated as full term with exception of HBV vaccine
if his / her weight less than 2 grams ( give HBV vaccine after one month from birht )🌹
🌹SLE patient on immunosuppressive, which vaccine should be taken by the patient:
Annual influenza
🌹Mother has gastroenteritis and breast feeding her child (4months old) Coming for
vaccines: A-delay vaccines B-give hep b only C-give OPV only D-give them all ✅
🌹What type of food we must ask about sensitivity before vericella vaccine?
A-milk. B-egg ✅✅ C-chicken D-rice
🌹Egg sensitivity he should not get which vaccine? yellow fever ‫ﻟﻮن اﻟﺒ~ﺾ اﺻﻔﺮ ﻓﻤﻦ اﺳﻤﻬﺎ‬
🌹Patient known SLE on immunosuppressive agents which vaccine is recommended?
Annual influenza ✅✅ ‫ﺻﺢ „ﻞ ﺳﻨﺔ •ﺎﺧﺬوﻫﺎ‬
🌹Child brought by father complained of Measles, you knew from history that he didn't
take Vaccines? A- educate father✅ B- treat child and ignore father
🌹Trauma patient in hospital . When to give him vaccine of PCV ( Pneumococcal conjugate
vaccine ) : A.2days. B. one week. C.2weeks✅ Tow = Truma ¸Ž Ó ‫¸ ﻣﻊ‬Ž Ó ‫ﺣﺮف‬
🌹Which vaccine contraindicated in immuncompromise patient == varicella
🍓pregn'nt 't third tri she is chrons pt on azithropine+biological agent ask about her child
vaccines ? A-give 'll. b-del'y till 3 month C-delay till 6month✅
🍓1year vaccine include: OPV, MMR, MCV , PCV
🍓 Patient known SLE on immunosuppressive agents which vaccine is recommended?
A-do not give live attenuated vaccine ✅ ✔ B-Varicella C-MMR
🌷Small box vaccine is contraindicated in patient with psoriasis and contact dermatitis🌷
‫¸ ﻻن راح •ﻌﻤﻞ ﻟﻪ‬Ž • ‫ ﺷﺨﺺ ﺛﺎ‬ŒŽ • ) Òç‫ ﻟﻮ (ﺑﻮﻛﺲ ﺻﻐ‬ýÓ‫_ ﺣ‬
Ž ‫ع ﺗﻌ‬c‫ﻂ =ﻣﻤﻨ‬Ï‫ر‬
‫ح‬À‫ﺎﻟﺠﻠﺪ و ﻗ‬u ‫ ﻣﺸﺎ™ﻞ‬dermtitis and psorisisi
🌷crohn's pt on azathioprine + biological agent want child vaccines = delay till 6month🌹
🌷A 6 year old who never received vaccination except BCG at birth he is hepB+ve ,what
vaccines should he receive now? A- Polio. B- hib. C- dtp✅✅. D- mmr
🍓 HBV nurse vaccine what to give her ? - immunoglobulin ✅
🍓 6m old baby k/c of epilepsy, on anticonvulsants, regarding vaccinations:
A-Stop all vaccinations B-Switch opv to ipv C-Don’t give dtap✅✅D-Don’t give live vaccines
🌷pregnant with crohn's on biological drug and steroids= dely live vaccine 6 months🌹
🍓 Contraindicated vaccines in pregnancy A-•meningococcal. B-•varicella ✅✅ C-•influenza
🌹Hep B = recombinant vaccine = Hepatitis B Vaccine (Recombinant)
🌹URTI with lymph node enlargement for child who missed vaccine:
A- Strept. Pneumonia. B- Pertussis D- Diphtheria✅
🌹Pneumonia vaccine in elderly?
• PPSV23 only • PCV13 first, followed by a dose of PPSV23 ²§‫ﺎ اﻟ–ﺒ‬¹‫ {ﻌﺪ‬²§‫ء اﻟﺮﻗﻢ اﻟﺼﻐ‬ÔÕ
o ‫اول‬
• PPSV23 first, followed by a dose of PCV13 • Or 13 then 23
🌹-child have vesicles and rash and have immune deficiency what to do for his brother?
a-Immunoglobulin ‫ ﻋﺸﺎن •ﻘﻮي اﻟﻤﻨﺎﻋﺔ‬b-Avoid skin contact
….
🌹 What is the fovarble place for AV fistula?
A-brachial artery basalic vein B-brachial artery cephalic vein
C-radial artery basalic vein D-radial artery cephalic vein ✅✅✅
🌹 Q about renal impairment with arteriovenous anastomoses or shunt
A- arteriovenous B- brachiocephalic C- radiocephalic ✅✅
🌹55 man hypertensive and has renal insufficiency. He is evaluated for AV fistula , no
history of trauma or AV malformations in his non dominant hand , what type of fistula
should be offered for him first=A-Brachiocephalic B-radiocephalic C-baslocephalic.
Patient diagnosed with CKD now he is coming in outpatient clinic and you decided to start
him on hemodialysis, his crt is 10, what best method of access
to start hemodialysis?
A. Arteriovenous graftÒç‫ ﺟﺮاﻓﺖ ﻣﺸﺎ™ﻠﻬﺎ ﻛﺜﻴ‬B. Arteriovenous fistula
C. Tunneled venous catheter D. Non tunneled venous catheter

🌹2 Pics of child’s hand with distal radial fracture with and angulation and Deformity with
small lesion. What will you do: Closed reduction with below elbow cast
.....
🌹 Patient had thenar muscle atrophy however his sensation was intact over that area,
what nerve injury is most likely? A. Radial B.Median ✅✅ C. Axillary D. Ulnar
.........
🌹 Hypothenar atrophy but sensation intact? Ulnar injury

🌹weak of dorsoflection of hand , which nerve ? Radial 🌹

HIV
🌹 27 year old with history of confusion for 2 days and seizure for 5 hours . he is not
oriented to time place person . he has history of unprotected sex for 6 years ,serum
cryptococcal antigen positive , what is appropriate test to do :-
A. HIV serology B. csf toxoplasmosis C. csf syphilis D. herpes
● HIV with splinter Hge ? Endocarditis
🌹Business man having unprotected sex previous 6 months presented with seizures, cough
..etc. Investigations show cryptococcus neoformans. What's the patient having? HIV ✅
🌹unprotected sex = cryptococcus neoformans = HIV🌹
🌹Active HIV pregnant woman, advice:
A-Safe to breastfeed while taking medications
B-Advise for c/s to reduce chance of baby getting infected ✅✅
🌹pt result came back HIV +ve who came prior for infertility assessment:
A-Tell the patient✅ B-tell the wife
🌹HIV patient developing CNS problem what to give? sulfadiazine and pyrimethamine
🌹positive cryptococcal antigen ,with hx of unprotected sex : A. hiv B. syphilis
🍓business man used to travel to east complaining of cough and lymphadenopathy and
oral candida what is dx:
A. toxoplasmosis
B. HIV✅✅Yeast infections in the mouth, oesophagus and other mucous members are
common in people who are infected by HIV.
🌹 Man come to your clinic complaining of oral thrushes and generalize fatigue and
lymphadenopathy for 2 weeks on forthar history he said the he traveled for several time
what is the most likely diagnosis?
A.hiv ✅ B. measles C. malaria D. syphlis
🌹Painless penile ulcer= syphilis

1Bone
🌹Sharp, shooting pain of neck radiated to shoulder and jow dx? Cervical disk prolapse
🌹 Patient shoulder pain radiate to arm and tingling loss of sensation and
no reflexes: A. cervical disk prolapse. B. MS. C. rheumatica myalgia
🌹A 40-year-old “veterinarian” man :) coming in with headache neurological
signs and symptoms, agitation and mood changes and irritability. Tenderness in the right
iliac fossa.Remembers having fever two weeks prior to onset of symptoms. What is the
most appropriate management?
A. tuberculin test B. X-ray right iliac joint C. Amphylline test (or something similar)
....
🌹Elderly Female with back pain, dexa scan shows: -1.9 lumbar spine, -2 hip, CT
shows compress fracture:..
A. Osteoporosis B. Osteopenia
🌻Dexa findings goes with osteopenia <1.0->2.5 but there’s compression fracture gose with
osteoporosis (dexta should be <2.5)
(Normal Dexta >1.0)
....
Elderly Pt with back pain, Bone density tests result Spine - 2 Peripheral - 1.9 What is dx A.
Osteoporosis B. Osteopenia✅ (‫)ﺗﺼﺤﻴﺢ‬
....

🌹 Paper like cells of bone marrow ? gaucher disease= ‫ =ﺷﻖ اﻟﻮرﻗﺔ‬²§‫ ﺟﻮﺷ‬.
🌹doctor ask pt to face wall lean bend forward and let his arm unsupported, examining
what? A. scoliosis B. child abuse C. rectal prolapse
🍓 Case with sarcoma in thigh how to now the metastasis: C.T for chest
Sarcoma metastatic commenly to > Polumnar . Sarcoma = chest
🌹Old Patient with cancer complaining of sudden onset back pain , best next step:
A - MRI B- steroid +MRI

🍓Female pt complaining of back pain increase when she walking downstairs?


Spinal stenosis
🍒65 year old patient known to have spinal stenosis. He presents with central back pain.
His pain starts in the morning and usually resolves after 30 minutes with no treatment. Yet,
his symptoms improve with acetaminophen. Upon examination, you found mild paraspinal
muscle spasm. Which of the following is the most appropriate management?
A. Biofeedback B. Physiotherapy✅ C. Spinal laminectomy D. Stronger pain killer

🍓 70 years with lumber stenosis treatment : Physiothyrphy > NSAID > Surgey

🍓 Vit D deficiency = rickets 🍓


🍒Military With hx of prolonged standing c/o Flat feet and pain in medial foot. What
tendon involved? spring ligament It is called also calcaneonavicular ligament
🌹While examining the knee, tibia Above/anterior to foment what ligament is injured?
A. Anterior B. posterior C. tibial,..
🌹Female patient complains of medial knee pain that increases with activity , tenderness
on medial knee, Diagnosis: 1-tendonitis 2-osteoarthritis
🌹Adolescent patient with tenderness all over the tibial tuberosity he recently had a
growth spurt what is the most likely diagnosis:
A/Stress fractur B/osgood-schlatter✅ C/Perthes
‫ﻂ‬Ï‫ =ر‬osgood = ) ‫ (رﻛﺒﺘﻚ‬¤‫ اﺳﺠﺪ =ﻋ‬tibial tuberosity ‫ﺬە ﻣﻮﺟﻮدة ﺗﺤﺖ‬¹ knee
🌹osgood-schlatte =tenderness all over the tibial tuberosity.
🍓Pt k/c of RA with knee pain on cortisol not improve wt to add:Methotrexate🍓

🍓 Elderly man with bilateral knee pain increases at night and with rest on examination
there's no effusion, erythema, swelling. What is next investigation is: A. Bilateral x ray ✅
B. MRI C. bone density
🍓 Patient in his 30s with right knee swelling Joint aspiration showed Yellow, turbid,
positive mucin clot test, 15 WBC, 90% PMN (no mention of crystals) What’s the most likely
diagnosis? A. septic arthritis B. Rheumatoid arthritis C. Gout D. Pseudogout
🌹 case of Young pt rt knee pain erythema lines I don’t recall lesion swelled up and hot
(case of cellulitis probably), What’s the organism ? A. bacteriod. B. S pyogen ✅✅

🌹 Patient with osteoarthritis on NSAIDs came with symptoms of perforated


ulcer asking about diagnosis? NSAID induce PUD

🍓Another one old women with knee pain in the morning the relief with rest and increase
with walking = osteoarthritis .
OA > Releafe by Rest and worse by activitys
RA> Releafe by Activity and worse by Rest.
🌹66 yo Patient has a history of polyarthralgia with pain in distal and proximal
interphalangeal , no hx of morning stiffness + medically free no active arthritis ?
A. Methotrexate B. Finger splints✅✅ C. Cyclophosphamide
Osteoarthritis Inexpensive splints worn nightly can reduce the pain of hand osteoarthritis.
🌹 -Hip pain in elderly, x ray showing osteophytes and narraowing:
A-Osteoporosis B-Osteoarthritis✅✅
....
🌹Pt with compressive fracture with Dexa scan result what’s the diagnosis?
A. Osteomalacia B. OsteoporosisC. Osteopenia D. paget
🌹 81 female Patient complaining of back pain after cough, x ray showed compression
fracture; what is the initial step in management?
A. Intranasal calcitonin B. Paracetamol C. Admit for surgery D. Iv bisphosphonate
🌹Cervical osteoarthritis not improved will on NSAID drugs ?
A. Soft cervical collar Physiotherapy> nsaids> soft cervical collar> ice> Injectible Steroids.
🌹 Pt 75 y/o with hip pain increases with activity +mainly severe before sleeping , but she
has morning stiffness for several hours :A. Osteoporosis✅ B. Osteomyelitis C.
Osteoarthritis
🌹Elderly Pt with back pain, Bone density tests result Spine - 2 Peripheral - 1.9 What is dx
A. Osteoporosis ✅ B. Osteopenia

🌹elderly pt with significant height loss whats the investigation?


A. bone density✅✅ B. osteoporosis C. cervical and spine
🌹 Elderly k/c of DM, HTN, PUD is diagnosed with osteoarthritis and complaining
of pain, what analgesic to give? A. Ibuprofen B. Another NSAID C. tramadol D. Steroid
🍒Patient known case of HTN, DM , with chronic joint pain on paracetamol with minimal
improvemt what to give him?
A-Celecoxib B-lbuprofe n C-Steroid D-Tramadol= opiod pain meducantion

🌹 Old px with osteoarthritis and cervical laminectomy for degenerative


myelopathy ... now c/o gait instability, urinary incontinence for 1 month ,
most likely diagnosis is ?
A. cauda equina syndrome B. Transverse myelitis
C. MS D. Recurrent Cervical degenerative disease
🌹 old patient with osteoarthritis and with cervical degenerative disease did cervical
discectomy .. now has uncontrollable bladder what is the cause ?
A. cauda equina syndrom B. MS C. transverse myelitis D. cervical discectomy
🍒 pharmacological agents for use in osteoporosis ad malignancy= denosumab🍒
🍒 most approved drug by FDA to treat osteoporosis that result from PTH peptide related
tumor? Denosumab
🌹lung ca complaining of pain lab showed high ionised ca Management ?
A. IVF then Denosumab = used to treat bone metastases.🌹
🍒Old lady with hip pain, increases with walking? osteoarthritis🍒
🌹Old lady with PIP and DIP, case of OA, not controlled with nsaids, what to add
Methotrexate
🌹Elderly with lumber fracture how you will manage?
Oral alondernate = is a bisphosphonate medication used to treat osteoporosis and Paget's
disease of bone
....
🌹- 77ys old man with hypertension on atenolol 100 with hearing loss in noisy places + air
conduction better than bone =A. atenolol toxicity B. problem with cochlear hair cells
‫ﺎر اﻟﻌﻤﺮ‬6‫• ﻣﻊ ﻛ‬
O • K ŽO‫ﺬا [ﺴ]ﺐ اﻟﺼﻤﻢ ا‬V
Elderly with sensory neural hearing loss we should consider presbycosis > it’s cochlear or
sensory neural hearing loss, cochlear hair cells will be affected
....

🌹Long case : Rash , - ANA , fever , arthritis and arthralgia , hyperferritinemia ddx ?
A. SLE B. RA C. IE D. Adult Still disease
‫ ﻣﺮض‬adult still ‫ ﻫﻮ ﻣﻦ اﺣﺪ اﻧﻮاع‬juvinal arthritis ‫ •ﺤﺼﻞ ﻣﻌﺎﻫﻢ‬rash + arthrits + fever
....
🍓 5As for smoking: Ask, Advise, Assess. Assist, and Arrange
🌹27 ys old female 10 ys smoking oral lesion like ulcer the same from 4 month
Stain as initial = Biopsy to confirm
🌹a man will visit an area endemic for onchocerciasis for one week chance to be
infected? B.mild✅ C.moderate D.high
……
Duodenal perforation x. Ray :

….
🌹Which test is appropriate to study 5 different types of exercise given to obese ladies over
a period of time and follow their BMI
A. cohort B. cross sectional. C. case control. D. clinical trial !@
🌹Admitted patient suddenly develops fever and low blood pressure, Central line site
shows erythema and swelling what's your action:
A. draw blood for culture change the line and resume dialysis
B. draw blood for culture remove the line and start abx
🌹 If incidence is the same, what would increase the prevalence of a disease?
A. more accurate diagnostic tests. B. better coping with the disease C. Loss of follow up
🌹 pt walks around saying inappropriate stuff to people, what’s his problem?
A. preservation. B. train of thoughts!@
🌹 What indicates adequate Resuscitation?!@
A. Normalize BP. B. Central venous pressure 12. C. Normalize H
…..

You might also like